Download - Ευκλειδης Β 69

Transcript

ΕΚΔΟΣΕΙΣ ΠΑΤΑΚΗ www.patakis.qr·

πρωτοπορία στnν εκπαίδεvσn

Άλγεβρα Α' Λυκείου

Θ. Τσιούμας, ι Σιάχος

Μεθοδολογία Άλγεβρας Α' Λυκείου

Μ. Ευσταθίου, Ε. Πρωτοπαπάς

Μεθοδολογία Άλγεβρας Β' Λυκείου

Γεωμετρία Β' Λυκείου

Γενικής παιδείας

Ε. Πρωτοπαπάς

Γενικής παιδείας

Ε. Πρωτοπαπάς

Μαθηματικά r· Λυκείου

Αρχέs οικονομικήs θεωρίοs

Αρχέc; οικονομικής θεωρίας Γ Λυκείου

θετ. και τεχν. κατεύθυνσης Μάθημα επιλογής για όλες τις κaτευθιjνσrις

(2 τόμοι) Δ. Ι. Παναγιωτόπουλος ι Σιάχος, Θ. Τσιούμας

ΝΕΑ ΔΙΕΥΘΥΝΣΗ: ΠΑΝΑΓΗ ΠΑΛΔΑΡΗ (ΠΡΩΗΝ ΠΕΙΡΑΙΩΣ) 38, 104 37 ΑΘΗΝΑ, ΤΗΛ. 2 I 0.36.50.000, 2 Ι 0.52.05.600, 801 .I 00.2665

ΘΕΣΣΑΛΟΝΙΚΗ: Ν. MONAHHPIOY 122,563 34. ΤΗΛ. 2310.70.63.54 web site: http://www.patakis.gr • e-mail: [email protected][email protected]

Νέες Εκδόσεις της

Ελληνικής Μαθηματικής Εταιρείας

� �

fΞΜΗΝιΚΗ ΜΑΘΗΜΑfΙΚΗ ΕΤΑΡΕΙΑ

,,;,,, .,, ,.,, , .•. ,., Χ. Δ. Artιnpόvrnς και S. Κ. Ch8krab8rtl

Ελληvιοι.ή Μαθι1ματοιιή Εταιρε:ία

Κεvτρ κή διάθεση Ελληνική t\ΑΑθημaτική Εταιρεία

Πανε1 ι.ιίου 34 - Αθήνα

ΕΛΛΗΝΙΚΗ ΜΑΘΗΜΑΤΙΚΗ ETAIPEIA Τ ε ύ χ ο ς 69 • Ι ο ύ λ ι ο ς • Α ύ γ ο υ σ τ ο ς • Σ ε π τ έ μ β ρ ι ο ς 2008· Έτο ς λη' • Ευρώ: 3,50

e-mail: [email protected] www.hms.gr

ΜΑΘΗΜΑΤΙΚΟ ΠΕΡΙΟΔΙΚΟ ΓΙΑ Τ Ο ΛΥΚΕΙΟ

ν' Οι Ολυμπιονίκες στο Ευρωκοινοβούλιο

ν' Θεόφιλος Καϊpης και οι Αριθμοί ν' Μαθηματικοί Διαγωνισμοί

ν' Homo Mathematicus

Μαθηματικό Α' Τάξης ν' Άλγεβρα: Ταυτότητες και Απόλυτα ν' Γεωμετρία: Ισότητα και Ανισοτικές Σχέσεις

Μαθηματικό Β' Τάξης ν' Άλγεβρα: Τριγωνομετρία

ν' Γεωμετρία: Θεωρήματα διχοτόμων και ομοιότητα ν' Κατεύθυνση: Διανύσματα

Μαθηματικό Γ' Τάξης ν' Μαθηματικά Γενικής Παιδείας ν' Μαθηματικά Κατεύθυνσης: Μιγαδικοί αριθμοί ν' Ένα θέμα Πανελλαδικών εξετάσεων ν' Αντίστροφες συναρτήσεις

• • • • • • • • • • • • • • • • • • • • • •

ν' Ο Ευκλείδης προτείνει •••

ν' Τα Μαθηματικά μας Διασκεδάζουν

2 3 9

31

35 40

43 50 54

60 63 69 70

75 79

Γράμμα της Σύνταξης Αγαπητοί μαθητcς και συνάδελφοι, Με ευκαιρία την έναρξη της σχολικής χρονιάς, σος

ευχόμαστε υγεία, χαρά και επιτυχία στις προσπάθειές σος.

Γιο την αναβάθμιση του επιπέδου του περιοδικού απαι­τείται η συμβολή όλων μας. Θα θέλομε οπό τη θcση αυτή, να απευθύνουμε και στους μαθητές και κυρίως στους συναδέλφους την έκκληση να μας στέλνουν όχι μόνο ασ­κήσεις ή λύσεις στις προτεινόμενες ασκήσεις, αλλά και άλλο ενδιαφέροντα άρθρα. το οποία λείπουν οπό το περ­ιοδικό με αποτέλεσμα να αναλαμβάνει το έργο αυτό κατά μεγάλο μέρος η Συντακτική Επιτροπή. Θα είναι μεγάλη η χορό για μας, αλλά και σημαντική προσφορά στο περιο­δικό μας η αποστολή όρθρων γιο το επόμενο τεύχη του Ευκλείδη Β' με τελική ημερομηνία για το 4ο τεύχος την 15η ιaναυαρίου 2009. Ευχαριστούμε εκ των προτέρων γιο την όποια συνεργασία.

Με συναδελφικούς χαιρετισμούς Ο Πρόεδρος της Συντακτικής Επιτροπής Γ.Σ. Τοσσόπουλος Ο Αντιπρόεδρος της Συντακτικής Επιτροπής Β. Ευσταθίου

Σύνθεση εξωφύλλου: Η 49η Διεθνή Μαθηματική Ολυμπιάδα 2008

Η Ελληνική Ομάδα με τον Ισπανό gυlde

Στιγμιότυπα απ6 την τελετή βράβευσης των μαΘητών αη6 τους Ισπανούς επισήμους στην 49η Διεθνή ΜαΘηματική Ολυμπιάδα •••••••••••••••••••• • • ••••••••••••••••••••••••••••••••••••••••••••••••••••••••

, ΕΚΔΟΣΗ ΤΗΣ ΜΑΘΗΜΑτΙΚΗΣ ΗΑΙΡΕΙΑΣ ΠΑΝΕΠΙΣΤΗΜΙΟΥ 34 106 79 ΑΘΗΝΑ 1'1}'.:2103617784 -3616532 Fax: 2103641025 Εκδότης: Αλcξανδρής Νικ6λαος Διευθυντής: Τυρλής Ιω6ννης

επιμέλεια 'ΕκδοΗης: Τασσ6πουλος Γιώργος

Ευσταθίου Βαγγέλης

Εκτελεστική Γραμματεία

Πρόεδρος: Τασσ6nουλος Γιώργος

Αντιπρόεδρος: Ευσταθίου Βαγγέλης

Γραμματέας: Χριατ6nοuλος ΓΙcΜιγιώrης Μέλη: Αργυρ6κης Δ.

Δρούτσας Π. Λουρίδας Σ. Ταπcιν6ς Ν.

Κωδικός ΕΛ.ΤΑ.: 2055 ISSN: 1105- 7998

Αθανασόπουλος fεώρyιος Αναστασίου fιάννης Ανδρουλακάκης Νίκος Αντωνόπουλος Νίκος Αργυράκης Δημήτριος Βακαλόπουλος Κώστας Δρούτσας Παναγιώτης Ευσταθίου Βαγγέλης Ζαχαρόπουλος Κων/νος Ζώτος Βαγγέλης Κακκαβάς Απόστολος Καλίκας Σταμάτης Κανέλλος Χρήστος Καραγκούνης Δημήτρης Καρακατσάνης Βασfλης Καρκάνης Βασiλης Κατσούλης fιώργος Κερασαρfδης fιάννης Καρδαμίτσης Σπύρος Κηπουρός Χρήστος Κλάδη Κατερίνα

Συντακτική επιτροπή Κόντζιας Νίκος Κοτσιφάκης fιώρyος Κουτρουμπέλας Κώστας Κυριαζής Ιωάννης Κυριακόπουλος Αντώνης Κυριακόπουλος Θανάσης Κυβερνήτου Χρυστ. Λαζαρfδης Χρήστος Λάππας Λευτέρης Λουρfδας Σωτήρης Μαλαφέκας Θανάσης Μανωλάκου Στοματική Μαυρογιαννάκης Λεωνfδας Μενδρινός fιάννης Μεταξάς Νικόλαος Μπρίνος Παναγιώτης Μυλωνάς Δημήτρης Μώκος Χρήστος Πανουσάκης Νίκος Ρέγκλης Δημήτρης

Σdϊτη Εύα

Σταϊκος Κώστας

Στάϊκος Παναγιώτης

Στρατής fιάννης

Ταπεινός Νικόλαος

Τασσόπουλος fιώργος

Τζιώτζιος Θανάσης

Τριάντος fεώργιος

Τσαγκάρης Ανδρέας

Τσατούρας Ευάγγελος

Τσικαλουδάκης fιώργος

Τσιούμας Θανάσης

Τυρλής Ιωάννης

Φανέλη Άννυ

Χαραλαμποπούλου Λίνα

Χαραλάμπους Θάνος

Χριστιάς Σπύρος

Χριστόπουλος Πανσyιώτης

Ψύχος Βαγγέλης ••• • •••••••••••••••••••••••••••••••••••• • ••••••••••••••••• • •••••••••••••••••••

Τα διαφημιζόμενα βιβλία δε σημαίνει 6τι προτείνονται απ6 την Ε.Μ.Ε. Οι συνεργάτες, τα άρθρα, οι προτεινόμενες ασκήσεις, οι λύσεις ασκήσεων κτλ. πρέπει να στέλνονται tyκαιρα, σrο γραφεία της Ε.Μ.Ε. με την ένδειξη "Για τον Ευκλείδη β'". Τα χειρ6yραφα δεν επισrρέφονται. Τιμή Τεύχους ευρώ 3,50 Ετήοια συνδρομή (12,00 + 2,00 Ταχυδρομικά = ευρώ 14,00) ΕτψJια συνδρομ1j για Σχολεία ευρώ 12.00 Το ανήτιμο yια τα τεύχη που παραyyέλνονται σrέλνεται με απλή εmταyή σε διαταγή Ε.Μ.Ε. Τ αχ. Γραφείο Αθήνα 54 Τ.Θ. 30044 ή πληρώνεται σrα yραφεfα της Ε.Μ.Ε.

Εκτύπωση: ΙΝΤΕΡΠΡΕΣ Α.Ε τηλ.: 270 8760330 Υncιίluνος τunoypαφcfou: Β. Σωτηριάδης

Οι Έλληνες Ολυμπιονίκες ιων Μαβημαιικών σιο Ευρωπαϊκό Κοινοβούλιο τ ις Βρυξέλλες «κατέλαβαν» οι Έλληνες μαθητές από 7 έως 1 0 Σεπτεμβρίου 2008 που έχουν λάβει διακρίσεις σε διεθνείς μαθηματικούς διαγωνισμούς (Διεθνής Μαθηματική Ολυμπιάδα 2008,

Ισπανία και Βαλκανική Μαθηματική Ολυμπιάδα 2008, Σκόπια) Συνοδεύονταν από τον πρόεδρο της Ελληνικής Μαθηματικής Εταιρείας Καθηγητή κ. Νικόλαο Αλεξανδρή του Πανεπιστημίου Πειραιώς. Οι μαθητές είχαν ευκαιρία να ανταλλάξουν απόψεις με μέλη του Ευρωπαϊκού Κοινοβουλίου και να εκφράσουν τους προβληματισμούς τους τόσο στα Μαθηματικά όσο και για το μέλλον της Εκπαίδευσης στην Ελλάδα και στην Ευρωπαϊκή Ένωση .

Η Ελληνική ομάδα που έχει πάρει μετάλλια σε διεθνείς διαγωνισμούς το 2007 - 2008 και συμμετείχε στην αποστολή στο Ευρωπαϊκό Κοινοβούλιο ήταν η παρακάτω : Βαφείδης Αναστάσιος, Γιεχασκιέλ Ηλίας, Μπραζιτίκος Σιλουανός, Παπαδημητρίου Δημήτριος, Ράπανος Νικόλαος, Μαυροειδή Άλκηστις, Ντριάκος Ιωάννης, Σάββας Μιχαήλ, Μπρατζιτίκος Κωνσταντίνος, Λιάμπης Κωνσταντίνος, Ρωμανίδης Νικόλαος, Παγωνάκης Δημήτριος, Τσαμπασίδης Ζαχαρίας, Παντέλα Στέλλα, Καλαντζής Γεώργιος, Λογοθέτης Φώτιος, Λώλας Παναγιώτης. Χαρακτηριστικά φωτογραφικά στιγμιότυπα από τις εκδηλώσεις της Ελληνικής αντιπροσωπείας.

�--------------���

ΕΥΚΛΕΙΔΗΣ Β ' 69 τ. l/2

Ο Θεόφιλος Κα"ϊρης είναι ένας από τους λιγότερο γνω­στούς διδασκάλους του Γένους μας. Για τις θετικές επιστήμες, όμως, ήταν ο καλύτερος πρεσβευτής τους, στο δούλο Γένος, γιατί δίδασκε, κυρίως, μαθηματικά και φυσική , προσπαθώντας να εισα­γάγει τα σκλαβωμένα ελληνόπουλα, στις πολύτιμες φυσικομαθη­ματικές έννοιες, που διδάσκονταν στην Ευρώπη την εποχή εκείνη της μαύρης και βαριάς τουρκικής σκλαβιάς και που ανοίγουν τους ορίζοντες της σκέψης και της διανόησης.

Έτσι, σαν ένας άλλος Ρήγας Φεραίος, προσπάθησε να ενστα­λάξει στα κατάβαθα της ψυχής τους την έφεση όχι μόνο για πε­ρισσότερη γνώση, αλλά και για εκτίναξη του τουρκικού ζυγού,

Θεόφιλος Κα"ϊρης ώστε όλα τα ελληνόπουλα να γεύονται τους καρπούς της ελευθε-ρίας και να απολαμβάνουν τις σπουδαίες κατακτήσεις της επιστή­

μης και της τεχνικής των ευρωπαϊκών ιδεών.

Ο Θεόφιλος Καψης γεννήθηκε στην Άνδρο των Κυκλάδων, το 1784. Έμαθε τα πρώτα γράμματα στη γενέτειρά του και συμπλήρωσε τη μόρφωσή του στις φημισμένες, τότε, σχολές των Κυδωνιών της Μικράς Ασίας, της Πάτμου και της Χίου . Στην τελευταία, μάλιστα, δίδασκε ο δεινός ελληνιστής Αθανάσιος ο Πάριος και ο μαθηματικός Δωρόθεος ο Πρώιος. Το 1801 χει­ροτονήθηκε διάκονος και λίγο αργότερα πήγε στην Πίζα της Ιταλίας, για να σπουδάσει φυσικο­μαθηματικές επιστήμες επί 5 χρόνια. Ύστερα μετέβη στο Παρίσι, όπου ασχολήθηκε με φιλοσο­φικές μελέτες και παράλληλα γνώρισε το διάσημο διδάσκαλο του Γένους Αδαμάντιο Κοραή.

Το 181 1 , ανέλαβε διευθυντής της φιλοσοφικής σχολής της Σμύρνης και από το 1812 έως το 1821, διορίστηκε διευθυντής, και δίδαξε στην Ακαδημία των Κυδωνιών, ως διάδοχος, του δα­σκάλου του και καθηγητού των φυσικομαθηματικών επιστημών Βενιαμίν του Λέσβιου . Από τους νεωτερισμούς της σχολής αυτής ήταν, μεταξύ των άλλων, και η εισαγωγή της αλληλοδιδα­κτικής μεθόδου (οι καλύτεροι μαθητές επαναλάμβαναν την διδασκαλία στους περισσότερο αδύ­νατους), καθώς και η εποπτική και πειραματικά διδασκαλία της φυσικής. Σε αντίθεση προς τις άλλες σχολές της σκλαβωμένης Ελλάδος, όπου την πρώτη θέση κατείχαν τα φιλολογικά μαθή­ματα, την Ακαδημία των Κυδωνιών χαρακτήριζε προοδευτικό και μεταρρυθμιστικό πνεύμα με περισσότερο προσανατολισμό στα μαθηματικά και στη φυσική . Ο Θεόφιλος ΚαΤρης έδωσε, ακόμη, πανεπιστημιακό χαρακτήρα στα μαθήματά του, διδάσκοντας πειραματική φυσική, μαθη­ματικά και αστρονομία, με τη βοήθεια πειραμάτων και εποπτικών οργάνων, πράγμα το οποίο γι­νόταν για πρώτη φορά σε ελληνική σχολή . Τόση ήταν η φήμη της Ακαδημίας των Κυδωνιών κατά την περίοδο αυτή, ώστε συνέρεαν εκεί μαθητές από όλη τη χώρα, με αποτέλεσμα ο αριθ­μός τους να φθάσει τους 600.

Ο Θεόφιλος Καtρης, το 1919, μυήθηκε στη Φιλική Εταιρεία και το 1821 ύψωσε την ελληνι­κή σημαία της επαναστάσεως στην πατρίδα του, την Άνδρο. Τον επόμενο χρόνο, έλαβε μέρος, ως απλός στρατιώτης, στην εκστρατεία του Ολύμπου, στην οποία και τραυματίσθηκε.

Όταν πλέον έπνευσε ο άνεμος της ελευθερίας στην Ελλάδα, και ήλθε ο Καποδίστριας ως

ΕΥΚΛΕΙΔΗΣ Α' 69 τ. Ι/3

----------------Θεόφιλος Κα"ϊρης --------------­

πρώτος κυβερνήτης, ο Θεόφιλος Καtρης ήταν εκείνος που τον προσφώνησε με εμπνευσμένο πα­νηγυρικό λόγο . Όταν δε αργότερα ανέλαβε ο Όθωνας, βασιλιάς της Ελλάδος, απένειμε μεταξύ των άλλων και στον Θεόφιλο Καtρη παράσημο για τους aπελευθερωτικούς αγώνες του. Το πα­ράσημο, όμως, αυτό αρνήθηκε να το παραλάβει. Ακόμη αρνήθηκε το διορισμό του, ως καθηγη­τού της φιλοσοφίας στο Πανεπιστήμιο Αθηνών.

Από το 1 836 έως το 1 839 λειτούργησε στην Άνδρο πρότυπη ανώτερη σχολή, η οποία απο­δείχθηκε ανώτερη εκείνης των Κυδωνιών, ενώ διέθετε ταυτόχρονα οικοτροφείο για τα ορφανά παιδιά, κυρίως των αγωνιστών της ελληνικής επαναστάσεως. Δίδασκε μόνος του και με την αλ­ληλοδιδακτική μέθοδο, ελληνικά, θεωρητική και πειραματική φυσική, μαθηματικά, αστρονομία και φιλοσοφία. Χρησιμοποιούσε δε κατά τη διδασκαλία του και διάφορα όργανα, μεταξύ των οποίων και το πρώτο τηλεσκόπιο που ήρθε στην Ελλάδα.

Δυστυχώς, το 1 939, κατηγορήθηκε από τους αντιπάλους του, ότι δίδασκε στη σχολή της Άνδρου μια νέα θρησκεία, τη θεοσέβεια, και γι' αυτό το λόγο εξορίστηκε από την Ιερά Σύνοδο στη Ιερά Μονή της Σκιάθου . Το 1 842 κατέφυγε στο εξωτε­ρικό, για να επιστρέψει δυο έτη αργότερα, πιστεύοντας ότι ο θόρυ­βος εναντίον του είχε καταλαγιάσει. Οι κατήγοροί του, όμως, πέτυ­χαν να τον παραπέμψουν, το 1 852, στο δικαστήριο, το οποίο τον καταδίκασε σε φυλάκιση 2 ετών. Ένα έτος αργότερα, ο Θεόφιλος Καtρης πέθαινε στη φυλακή, ενώ τον επόμενο χρόνο ο Άρειος Πά­γος αναιρούσε την καταδίκη του.

Οι Ά νδρειοι, τιμώντας τη μνήμη του, ανήγειραν την προτομή του Αδαμάντιο Κοραή

σε κεντρική πλατεία της πόλης τους και ίδρυσαν την ΚΟ:fρειο βιβλιοθήκη, στην οποία υπάρχουν πολλά τυπωμένα και χειρόγραφα έργα του. Ιδιαίτερη σημασία έχουν τα αντίγραφα των χειρογράφων βιβλίων του, τα οποία χρησιμοποίησε για τα μαθήματα της σχολής της Άνδρου και τα οποία σώζο­νται στις διάφορες βιβλιοθήκες της Ελλάδος, ανάμεσα στις οποίες εξέχουσα θέση κατέχουν οι 4 τό­μοι των χειρογράφων του επισκόπου Δαβίδ Φωκίδος, υπομνηματισμένα από τον ίδιο.

Η �τυγγρό�βμiα�ιι'v't Γ!iJ!�) ΘεG'D�rηλσιυ iΚ�t'{Q"prη1 Το διδακτικό έργο του, ο Θεόφιλος ΚΟ:fρης, το συγκέντρωσε σε 4 ογκώδεις χειρόγραφους τό­

μους, τους οποίους ασφαλώς χρησιμοποιούσαν οι πολυπληθείς μαθητές του, της σχολής της Ά ν­δρου. Ένας από τους καλύτερους αυτούς μαθητές ήταν ο Αδαμάντιος Μολοχάδης, που αργότερα έ­γινε επίσκοπος Φωκίδος με το όνομα Δαβίδ, ο οποίος, όχι μόνο αντέγραψε τα 4 αυτά βιβλία του δα­σκάλου του, τον οποίο τόσο πολύ αγαπούσε και σεβόταν, αλλά και τα υπομνημάτισε με υποδειγμα­τικό τρόπο, κάνοντάς τα περισσότερο κατανοητά και με ευρύτητα πνεύματος αναγνώσιμα. Το πρώ­το από τα βιβλία του έχει τίτλο: «Στοιχειώδης Φυσική)) και περιλαμβάνει 654 σελίδες. Ασχολείται με τη «μικροσκοπική)) μελέτη της φυσικής και των στοιχείων αυτής, που δομούν το φυσικό κόσμο. Στο εγχειρίδιο αυτό γίνεται μια ενιαία παρουσίαση των τότε γνώσεων γύρω από τη φυσική (εκτός της Μηχανικής), τη Χημεία, την Ορυκτολογία, τη ζωολογία και τη Βιολογία.

Το δεύτερο βιβλίο του τιτλοφορείται: «Μηχανικόν μέρος της Φυσικής1)) και έχει 6 1 6 σελί­δες. Χωρίζεται σε δύο μέρη, εκ των οποίων το πρώτο περιλαμβάνει τις κινήσεις των σωμάτων,

1 Στην αρχή του βιβλίου αυτού ο Θεόφιλος Κα"tρης σημειώνει: «Ήρξατο η παράδοσις του μηχανικού της Φυσικής εν

Άνδρω 1 837, Νοεμβρίου 26, ημέρα Παρασκευή» στο τέλος δε γράφει: «Ετελείωσεν η παράδοσις της παρούσης, 1 839, Μαρτίου 4» . Είναι φανερό, λοιπόν, ότι η διδασκαλία του βιβλίου αυτού διήρκεσε 15 περίπου μήνες. Επομένως η διδασκαλία της Αστρονομίας (που περιλαμβάνει σχεδόν το μισό βιβλίο) πρέπει να διδάχθηκε σε 7 μήνες περίπου. Στην εποχή μας, για το μάθημα της αστρονομίας, που διδάσκεται κατ' επιλογήν στη Β' Λυκείου, διατίθεται μόνο μια ώρα την εβδο­μάδα. Σήμερα που η ύλη της αστρονομίας είναι πολλαπλάσια και οι απλές, αλλά νέες aστρονομικές γνώσεις πιπιλί­ζονται από τους μικρούς μαθητές του Δημοτικού Σχολείου. Αντί, λοιπόν, να αυξηθούν οι ώρες διδασκαλίας του τό­σο σπουδαίου και σύγχρονου μαθήματος της αστρονομίας στα Λύκεια, αυτές μειώνονται στο ελάχιστο . Ας ελπί­σουμε ότι με την πρώτη μεταρρύθμιση των προγραμμάτων που θα γίνει στο εγγύς μέλλον, θα πάρει το μάθημα αυτό την πρέπουσα θέση του στα Λύκεια της χώρας.

ΕΥΚΛΕΙΔΗΣ Α' 69 τ. l/4

----------------Θεόφιλος Κα"ϊρης

τη βαρύτητα, την ακουστική, την οπτική και τις απλές και σύνθετες μηχανές. Το δεύτερο μέρος περιλαμβάνει την αστρονομία, με την οποία θα ασχοληθούμε περισσότερο στη συνέχεια.

Το τρίτο βιβλίο επιγράφεται «Γνωστική φιλοσοφία - φιλολογία», το οποίο περιλαμβάνει 4 1 7 σελίδες και είχε επανειλημμένες εκδόσεις, ενώ τα άλλα δύο προηγούμενα βιβλία του είναι ανέκδοτα, εκτός από την «Αστρονομία».

Το τέταρτο και τελευταίο βιβλίο του Θεόφιλου Κα.tρη επιγράφεται «Ποσοτική2», δηλ. Άλ­γεβρα, όπως θα λέγαμε σήμερα και περιλαμβάνει 628 σελίδες. Περιέχει όχι μόνο τις στοιχειώ­δεις αλγεβρικές γνώσεις, που διδάσκονται σήμερα στα Λύκεια της χώρας μας, αλλά και αρκετές γνώσεις πανεπιστημιακού επιπέδου, όπως θα δούμε3. Το βιβλίο αυτό, το εξέδωσε σε δύο τόμους το 1 ° Λύκειο Βόλου, το 1 994 τον 1 ° τόμο και το 2003 το 2° τόμο . Επίσης εξέδωσε, το 1 989, την Αστρονομία, που περιλαμβάνεται στο «Μηχανικόν Μέρος της Φυσικής» .

Στη συνέχεια, δεν θα ασχοληθούμε, βέβαια με όλα αυτά τα περισπούδαστα συγγράμματα του μεγάλου δάσκαλου του Γένους, αλλά μόνο με τον τέταρτο τόμο, τα μαθηματικά και λίγο με την αστρονομία.

Ονομάζει ο Θεόφιλος Κα'fρης το βιβλίο του αυτό «Ποσοτική», από τη λέξη «ποσό», επειδή , όπως εξηγεί ο ίδιος στην πρώτη παράγραφο, ασχολείται με τις μεταβολές, τις ιδιότητες και τις αναφορές, όχι πλέον των αριθμών της αριθμητικής, αλλά των γενικών αριθμών, που τους συμ­βολίζουμε με τα γράμματα της αλφαβήτου, δηλ. περιλαμβάνει τα «ποσά» .

Το πρώτο μέρος του βιβλίου, που είναι και το μεγαλύτερο, φθάνει μέχρι την 517 σελίδα και περιλαμβάνει 1 7 κεφάλαια με 5 1 2 (ΦΙΒ') παραγράφους. Θα μπορούσαμε δε να πούμε ότι περι­λαμβάνει τα λεγόμενα βασικά μαθηματικά, ήτοι τις αλγεβρικές πράξεις, τα κλάσματα τις δυνά­μεις τις πραγματικές και φανταστικές ρίζες, τις προόδους, τους λογαρίθμους και τους λογαριθμι­κούς πίνακες, τις πρωτοβάθμιες και δευτεροβάθμιες εξισώσεις, τα προβλήματα τόκου, την α­προσδιόριστη ανάλυση, τις τριτοβάθμιες εξισώσεις και τις εξισώσεις ανωτέρου βαθμού, τις άρ­ρητες ρίζες, τα συστήματα, τις συναρτήσεις, την τριγωνομετρία και τους τριγωνομετρικούς πί­νακες, τη σφαιρική τριγωνομετρία, τις σειρές, τις καμπύλες και τις κωνικές τομές.

Το δεύτερο μέρος καταλαμβάνει μόνο 1 1 1 σελίδες με 1 50 (ΡΝ') παραγράφους και περιλαμ­βάνει στοιχεία απειροστικού και διαφορικού λογισμού, διαφορικές εξισώσεις, μέγιστα και ελά­χιστα, εφαρμογή του διαφορικού λογισμού στη θεωρία των καμπύλων, στοιχεία ολοκληρωτικού λογισμού, ολοκλήρωση αρρήτων συναρτήσεων και ολοκλήρωση διαφορικών διωνύμων.

Το μάθημα της Ποσοτικής ο Θεόφιλος Κα'fρης άρχισε να το διδάσκει τον Ιούλιο του 1836 και τελείωσε τη διδασκαλία του τον Ιανουάριο του 1 939. Δηλ. μέσα σε δυόμισι χρόνια περίπου κάλυψε μια ποσότητα ύλης, αν όχι μεγαλύτερη, όμως, εφάμιλλη προς την ύλη που διδάσκεται σήμερα στα Λύκεια της χώρας μας. Και με τι μαθηματικό υπόβαθρο; Πολύ πιο κατώτερο από το υπόβαθρο των σημερινών μαθητών μας. Γιατί, φυσικά, τότε δεν υπήρχε ούτε η 6τάξια οργα­νωμένη διδασκαλία του Δημοτικού Σχολείο, ούτε η 3τάξια προπαρασκευή του Γυμνασίου . Τα ελληνόπουλα της πρώτης μετεπαναστατικής περιόδου δεν είχαν αυτή την πολυτέλεια της καλής εκπαιδευτικής προετοιμασίας για το Λύκειο, που έχουν τα σημερινά.

Υπάρχουν πολλά σημεία της διδασκαλίας του Θεόφιλου Κα'fρη που θα τα ζήλευε ακόμη και ο σύγχρονος μαθηματικός συγγραφέας διδακτικών βιβλίων4. Ιδιαίτερα θα ζήλευε την ερμηνεία και τη δικαιολόγηση των νέων εννοιών, που εισάγονται, όσο και αν αυτές είναι πρωτότυπες και

2 Τόσο το βιβλίο αυτό, όσο και τα άλλα τρία είναι δερματόδετα και αρκετά καλά διατηρημένα αν και πέρασαν, από τότε που γράφτηκαν, δυο σχεδόν αιώνες Φέρουν δε στη ράχη τους τα χρυσά γράμματα «Δ. Φ .» . Π ρόκειται για τον επίσκοπο Δαβίδ Φωκίδος (Αδαμάντιο Μολοχάδη) , ο οποίος ήταν μαθητής του Θεόφιλου ΚαΤρη στη σχολή της Άν­δρου και λόγιος της εποχής του. Σ' αυτόν ανήκαν τα 4 αυτά βιβλία που βρίσκονται σήμερα στη βιβλιοθήκη του I ου Λυκείου Βόλου, όπου τα δώρισε ο ίδιος, το 188 1 , ως καταγόμενος από το Νεοχώρι Μαγνησίας.

4 Ας πάρουμε για παράδειγμα τα «συνεχή κλάσματα>> . Ποιος θα περίμενε, ύστερα από μια διεξοδική ανάπτυξη, να αποδείξει ότι αυτά συγκλίνουν, τυπικά, σε έναν πραγματικό αριθμό;

ΕΥΚΛΕΙΔΗΣ Α' 69 τ.l/5

----------------Θεόφιλος Κα'ϊρης ---------------

ιδιόμορφες5. Ας σημειωθεί, τέλος, ότι πολλές φορές ο Θεόφιλος Κα:tρης, θέλοντας να βγάλει ένα συγκε­

κριμένο συμπέρασμα ή να επισημάνει μια σπουδαία ιδιότητα, επαναλαμβάνει τη στερεότυπη φράση : «μανθάνομεν λοιπόν ότι . . . ». Με τον τρόπο αυτό επιτείνει ακόμη περισσότερο την προ­σοχή των μελετητών - μαθητών του, για να προσέξουν και να ξεκαθαρίσουν τι σπουδαιότερες από τις προσφερόμενες γνώσεις. Αν σκεφθούμε ότι η επίταση αυτή θα γινόταν οπωσδήποτε πιο έντονη κατά την προφορική παράδοση του μαθήματος, αντιλαμβανόμαστε πόση ιδιαίτερη από­δοση στη μάθηση είχε η ανακεφαλαιωτική αυτή παράθεση .

(J) ισχσλ�Ω·στής lί{!Χ\' iφγσJ)V του Θεόφ�λου !Κ.α1ΊJ1Γi Ο επίσκοπος Δαβίδ Φωκίδος ( 1 805 - 1 887), κατά κόσμον Αδαμάντιος Μολοχάδης, είναι γνω­

στός στην ελλαδική εκκλησία ως ένας πνευματικός ηγέτης, που επί 32 χρόνια ποίμανε ο ποίμνιό του «εν παιδεία και νουθεσία Κυρίου» και επί πλέον βοήθησε αποφασιστικά, ως εκπρόσωπός της, το 1 872, στην αντιμετώπιση του βουλγαρικού σχίσματος από το Πατριαρχείο της Κωνσταντινουπόλε­ως. Δεν είναι, όμως, γνωστός, ως εξαιρετικός σχολιαστής των έργων του Θεόφιλου Κα:tρη, διότι άλ­λα μεν έργα δεν εκδόθηκαν άλλα δε δεν έτυχαν αυτά της ευρείας κυκλοφορίας.

Ο Αδαμάντιος Μολοχάδης αντέγραψε για δικό του λογαριασμό τα 4 χειρόγραφα βιβλία του δα­σκάλου του Θεόφιλου Κα:tρη, σε 4 ογκώδεις τόμους, καλαίσθητα δεμένους, με δερμάτινη ράχη, χρυσά γράμματα και στερεά εξώφυλλα. Ο σκοπός της αντιγραφής ήταν, οπωσδήποτε, για να έχει τα δικά του αντίτυπα, για να τα μελετάει όταν ήθελε, αλλά και για να τα δανείζει σε κάθε ενδιαφερόμε­νο. Επειδή, ακριβώς ήταν δικά του αποκλειστικά αντίγραφα, άφησε στο κάτω μέρος της κάθε σελί­δας ειδικό χώρο, με σκοπό να προσθέσει τις δικές του παρατηρήσεις και τα δικά του σχόλια.

Και πραγματικά έκανε θαυμάσια προσθήκη στο έργο του δασκάλου του. Ο χώρος αυτός είναι σχεδόν γεμάτος με τις σημειώσεις του. Υπάρχουν δε πολλές περιπτώσεις, που ο χώρος αυτός δεν επαρκεί και έτσι χρειάστηκε να γίνει συρραφή πρόσθετων φύλλων, εκτός σελιδοποίησης. Στον τόμο της «Ποσοτικής» παραθέτει 2. 1 1 4 σχόλια, δηλ. πάνω από 3 σχόλια ανά σελίδα, κατά μέσο όρο.

Ας αναφέρουμε μερικά από τα σχόλια αυτά και μάλιστα εκείνα που δεν περιέχουν πολλούς μαθηματικούς τύπους και μαθηματικές εκφράσεις. Στην αρχή της σελίδας 92 π.χ. , η οποία ανα­φέρεται στα ριζικά συναντούμε την εξής υποσημείωση : Ο τίτλος της παραγράφου αυτής και της επόμενης πρέπει να είναι: «Περί της αναγωγής των ριζικών σε απλούστερη μορφή». Έτσι ο ανα­γνώστης κατατοπίζεται καλύτερα για το περιεχόμενο της θεωρίας, που ακολουθεί.

Γενικά με τα σχόλιά του, ο Αδαμάντιος Μολοχάδης απλοποιεί τα γραφόμενα του βιβλίου και τα κάνει προσιτά στον αναγνώστη . Για τη λύση π. χ. του συστήματος 1 ου βαθμού φ - χ = 2, φ - 2χ = -3 , το οποίο λύνεται με το γενικό τύπο, που είχε βγάλει ο Θεόφιλος Καtρης, ο Αδαμά­ντιος Μολοχάδης παρατηρεί ότι: Γίνεται ευκολότερη η εργασία για την εύρεση του χ, εάν αφαιρε­θούν τα μέλη της 2ιις εξίσωσης από τα αντίστοιχα μέλη της ]ης. Για την εύρεση δε του φ, πρέπει να αφαιρεθούν τα μέλη της 2ης από τα μέλη της lης, αφού πολλαπλασιασθούν αυτά επί -2».

Συμπερασματικά μπορούμε να πούμε ότι ο Αδαμάντιος Μολοχάδης με τα σχόλιά του επε­ξηγεί και αναλύει πυκνά νοήματα, εκσυγχρονίζει τα κείμενα, βελτιώνει τις προτάσεις, που δεν κυριολεκτούν, συμπληρώνει τις αναφερόμενες θεωρίες και τα επιχειρήματα, βγάζει χρήσιμα και πρακτικά συμπεράσματα, δίνει τις απαραίτητες πληροφορίες για τους αναφερόμενους επιστήμο­νες, συγγραφείς και εφευρέτες, παραπέμπει σε άλλα συγγράμματα τους ενδιαφερόμενους κ.λπ.

!Η αστρονομίια του Θ�όφ!.λου ΚαΤρη Το βιβλίο της αστρονομίας του Θεόφιλου Καtρη , αποτελεί το μεγαλύτερο μέρος του «Μη­

χανικού μέρους της Φυσικήφ και περιλαμβάνει όλη την ύλη της αστρονομίας, που ήταν γνω­στή μόλις μια ή δυο 1 Οετίες μετά την έναρξη της διαστημικής εποχής, δηλ. από τον Οκτώβριο

5

Αυτό φαίνεται πολύ καθαρά στο ανάπτυγμα του Διωνύμου του Νεύτωνα (SΠΒ' κ.ε .), για το οποίο διαθέτει συνο­λικά 21 σελίδες του βιβλίου του . Μέσα σ' αυτές τις σελίδες εξηγεί τον τρόπο, με τον οποίο βρίσκονται οι συντελε­στές και οι δυνάμεις του κάθε όρου, με αποδείξεις, παραδείγματα, δοκιμές, και πολλά άλλα. Ακόμη διερευνά τις διάφορες μορφές, που είναι δυνατό να πάρει το διώνυμο αυτό και πώς, με τη βοήθειά του, μπορούμε να βρούμε και το ανάπτυγμα πολυωνύμου.

ΕΥΚΛΕΙΔΗΣ Α' 69 τ. Ι /6

---------------- Θεόφιλος Κάϊρης --------------­

του 1 957, οπότε με την πληθώρα των διαστημικών σκαφών ήλθαν στο φως οι νέες κατακτήσεις της αστροφυσικής. Χωρίζεται σε 6 κεφάλαια, τα οποία περιέχουν τα εξής θέματα: Γενικές γνώσεις αστρονομίας (ουράνια σφαίρα, κινή­σεις των αστέρων, παράλλαξη κ.λπ.) , ημερήσια και ετήσια κίνηση του Ήλιου (εκλειπτική, νόμος εμβαδών, ζωδιακό κύκλος, ηλιογήινα φαινόμενα κ.ά.) , η Σελήνη και οι κινήσεις της ( φάσεις, εκλείψεις, παλίρροιες κ.λπ.) , κινήσεις των πλανητών (νόμοι, περίοδοι, απο­στάσεις, δορυφόροι, κομήτες κ.λπ. ), απλανείς αστέρες ( φαινόμενα μεγέθη, κατάλογοι, μεταβλητοί αστέρες κ.λπ.) , περί χρόνου (εξίσω­ση του χρόνου, ο κύκλος του Ηλίου και της Σελήνης κ.λπ.) και περί τηλεσκοπίων (τηλεσκόπιο του Γαλιλαίου, διοπτρικά και κατοπτρικά τηλεσκόπια, με τις διάφορες κατηγορίες τους) .

Ας σημειωθεί ότι ένα ακόμη μικρό κεφάλαιο της αστρονομίας πε- Βενιαμίν του Λέσβιου ριλαμβάνεται στο βιβλίο «Στοιχειώδης Φυσική» με τους αερόλιθους (μετεωρίτες) και άλλο ένα στο βιβλίο «Γνωστική Φιλολογία - Φιλοσοφία», με την κοσμολογία. Η επιστημονική υποδομή του βιβλίου είναι δεδομένη, αφού ο Θεόφιλος Κσ:tρης χρησιμοποιεί π.χ. την «παρατήρηση» του έναστρου ουρανού, για να καθορίσει τις 4 εποχές του έτους. Το «πείραμα», αν και είναι δύσκολο στην αστρονομία, εντούτοις, με τη χρησιμοποίηση του τηλεσκοπίου, που είχε στη διάθεσή του ο μεγάλος αυτό διδάσκαλος του Γένους, καθώς και του κυκλοτέταρτου (ένα αστρονο­μικό όργανο παρόμοιο με τον εξάντα), αποτελούσαν τη βάση της εποπτικής διδασκαλίας του κατά τις βραδινές ώρες. Η «απόδειξη» αποτελεί απαραίτητη εργασία στις θετικές επιστήμες και ο Θεόφι­λος Κσ:tρης τη χρησιμοποιεί για πολλές έννοιες. Ας αναφέρουμε μόνο την απόδειξη ότι η διάμετρος της Γης είναι μηδαμινή μπροστά στην απόσταση Γης - Ήλιου. Δεν συμβαίνει, όμως, το ίδιο για την απόσταση Γης - Σελήνης, η οποία είναι 60πλάσια της διαμέτρου της Γης.

Η επιστημονική αξία της διδασκαλίας του καταφαίνεται και από το γεγονός ότι δεν διστάζει να χρησιμοποιήσει ακόμη και την επίλυση σφαιρικού τριγώνου για τον υπολογισμό των εκλει­πτικών συντεταγμένων και για τις εκλείψεις.

Ακόμη, προσπαθώντας να εξηγήσει το φαινόμενο της βαρύτητας, εισάγει μια δική του εντε­λώς καινούργια έννοια, την έννοια του «ενύλου», το οποίο θεωρεί ότι είναι διάχυτο στο Σύμπαν και συνδέει μεταξύ τους όλα τα ουράνια σώματα. Το «ένυλον» μοιάζει πολύ με το «πανταχηκί­νητο» του Βενιαμίν του Λέσβιου6 ή με τον αιθέρα των αρχαίων Ελλήνων.

Τέλος, η διδακτική μεθοδικότητα του βιβλίου της αστρονομίας καταδεικνύεται από την πρόσκληση του αναγνώστη να συμμετάσχει στις νέες προσφερόμενες γνώσεις, με το να χρησι­μοποιεί το πρώτο πληθυντικό πρόσωπο στις εκφράσεις του : αποδεικνύουμε, γνωρίζουμε, συ­μπεραίνουμε, θα προσπαθήσουμε κ.λπ. Ακόμη χρησιμοποιεί και διάφορες προεκτάσεις για την καλύτερη αφομοίωση των aστρονομικών γνώσεων από τους μαθητές του .

Βιβλωγραφικές αναφοpιές: I. Εγκυκλοπαίδεια «Πάπυρος - Λαρούς - Μπριτάνικα» 2 . Ελληνική Μαθηματική Εταιρεία: Πρακτικά πανελληνίων συνεδρίων 1984 και 1986. 3 . E.E. I.E.T. : Ο ι μαθηματικές επιστήμες στην τουρκοκρατία, Αθήνα 1990. 4. Καρρά Γιάννη : Κα'ί'ρης - Κούμας, δυο πρωτοπόροι δάσκαλοι. 5 . 1° Λύκειο Βόλου : Αναμνηστικό Λεύκωμα 2 11ς 50ετίας, Θεσσαλονίκη 1988 . 6. Μαυρομμάτη Κωνσταντίνου : Η Αστρονομία του Θεόφιλου Καtρη, Θεσσαλονίκη 1989. 7 . Μαυρομμάτη Κωνσταντίνου : Η αστρονομία του Βενιαμίν Λέσβιου, Θεσσαλονίκη 1991. 8 . Κ . Μαυρομμάτη- Δ. Χριστόπουλου : Η Ποσοτική (Άλγεβρα) του Θεόφιλου Κα'ϊρη, τόμ. Ι 0', Βόλος 1994. 9. Μαυρομμάτη Κωνσταντίνου : Η Ποσοτική (Άλγεβρα) του Θεόφιλου Κάfρη, τόμος γ\ Βόλος 2003 . I Ο. Μαυρομμάτη Κωνσταντίνου : Λεξικό Αστρονομίας, Βόλος 2006. 11. Ο.Λ.Μ .Ε . : Λόγος και Πράξη 1985 και 1990. 12 . Πανελλήνιο Συμπόσιο : Θεόφιλος Κα'ϊρης, Αθήνα 1988 . 13 . Σταμάτη Ευαγγέλου : Ιστορία των ελληνικών μαθηματικών, Αθήνα 1980 . 14. Υπουργείο Παιδείας: Διδακτικά εγχειρίδια Α', Β' και Γ Λυκείου .

6 Βλ. «Η αστρονομία του Βενιαμίν του Λέσβιου» σελ. 120 και αλλού.

ΕΥΚΛΕΙΔΗΣ Α ' 69 τ. Ι/7

Οι αριθμοί στα βάθη των αιώνων Θωμάς Γ. Καρανίκας

Η αλήθεια είναι ότι κάθε άρθρο θα πρέπει να έχει μια εισαγωγή, μία εναρκτήρια φράση . . . Η Ιστο­ρία όμως δεν είναι κάτι τόσο τακτικό και νοικοκυρεμένο και η αναζήτηση της πρώτης χρήσης των α­ριθμών είναι ένα ταξίδι στις ομιχλώδεις aπαρχές της ανθρώπινης ζωής και του πολιτισμού . Οι αρχαιο­λόγοι και οι μελετητές προσπαθούν να ανασυστήσουν το πάζλ της προϊστορίας μας από μερικές πήλινες πινακίδες. Οι καινούργιες ανακαλύψεις δεν είναι μόνο καινούργιες ψηφίδες σ ' αυτό το πάζλ, αλλά νέα δεδομένα που μπορούν να αλλάξουν ριζικά όλη την εικόνα του παρελθόντος και την σχέση μας με αυτό

Η παλαιότερη ένδειξη αριθμητικής καταγραφής βρέθηκε στην Σουαζιλάνδη της Νότιας Αφρικής και είναι μία περόνη μπαμπουίνου με 29 εμφανείς εγκοπές που χρονολογείται από το 35 . 000 π.Χ. Μοιά­ζει με τα «ημερολογιακά ραβδιά» που ακόμη και σήμερα χρησιμοποιούν στην Ναμίμπια για να καταγρά­φουν την παρέλευση του χρόνου. Επίσης είναι σημαντικό και άλλα κόκαλα , της νεολιθικής περιόδου, έχουν βρεθεί στην Δυτική Ευρώπη . Μία κερκίδα λύκου που βρέθηκε στην Τσεχία και χρονολογείται από το 30 .000 π.Χ. φέρει 55 εγκοπές σε δύο σειρές ανά πέντε, οι οποίες μάλλον αποτελούν καταγραφή θηρα­μάτων. Ένα από τα πιο ενδιαφέροντα ευρήματα είναι το αποκαλούμενο Κόκαλο Ισάνγκο, που βρέθηκε στις όχθες της λίμνης Εντουαρντς, ανάμεσα στην Ουγκάντα και την Λαϊκή Δημοκρατία του Κονγκό. Έ­χει χρονολογηθεί γύρο στο 20.000 π.Χ. και μοιάζει να είναι κάτι παραπάνω από πίνακας θηραμάτων . Μικροσκοπική ανάλυση αποκάλυψε πρόσθετες εγκοπές , οι οποίες μπορούν να συσχετισθούν με τις φά­σεις της Σελήνης Δεδομένης της σημασίας της πρόβλεψης της πανσέληνου, πιθανόν για λατρευτικούς λόγους αλλά σίγουρα για τον εξαιρετικά πρακτικό λόγο της νυχτερινής ορατότητας, δεν ήταν καθόλου παράξενο που οι νεολιθικοί άνθρωποι έδιναν τόση σημασία στην καταγραφή των μεταλλαγών του μεγά­λου Ρολογιού του ουρανού. Στην πραγματικότητα, μέσω της αστρονομίας , της aστρολογίας ή της κο­σμολογίας , ο ουρανός άσκησε τη μεγαλύτερη επίδραση στην εξέλιξη των Μαθηματικών .

Από την Μεσοποταμία τη γη μεταξύ των ποταμών Ευφράτη και τίγρη , έχουμε γραπτά αρχεία που εκτείνονται μέχρι 3 .500 π.Χ. περίπου Η περιοχή κατακτήθηκε κατά καιρούς από διάφορούς λαούς. Οι παλαιότεροι Σουμέριοι και Ακκάδες έδωσαν την θέση τους στους τεχνίτες του σιδήρου Χετταίους, οι ο­ποίοι υπέκυψαν στους τρομερούς Ασσύριους. Ακολούθησαν οι Χαλδαίοι και ο περίφημος Βασιλιάς τους Ναβουχοδονόσωρ, ώσπου κατακτήθηκαν κι εκείνοι από τους Πέρσες οι οποίοι με την σειρά τους υποτά­χθηκαν στις στρατιές του Μεγάλου Αλεξάνδρου. Το κέντρο εξουσίας ήταν αρχικά στην Ουρ και μετά στην Νινευη και την Βαβυλώνα. Τα κυριότερα μαθηματικά στοιχεία προέρχονται από την Παλιά Βαβυ­λωνιακή Αυτοκρατορία ( 1 900- 1600 π.Χ.) όπου έχουμε σουμεριακές και ακκαδικές επιδράσεις, και από τη δυναστεία των Σελευκιδών του 4ου αι. πχ , όπου είναι εμφανείς οι ελληνικές και βαβυλωνιακές επι­δράσεις. Λόγω της μεγάλης σημασίας που είχε η Βαβυλώνα σε όλη αυτή την περίοδο, τα τότε μαθηματι­κά είναι γνωστά ως Βαβυλωνιακά .Το σημερινό δεκαδικό σύστημα είναι ένα σύστημα θεσιακού συμβολισμού με βάση το I Ο -με άλλα λόγια I Ο μονάδες της μιας αριθμητικής θέσης αντιστοιχούν με I μονάδα της αμέσως ανώτερης . Τα παλαιότερα γραπτά που έχουμε δείχνουν ότι οι Βαβυλώνιοι χρησιμοποιούσαν ένα εξηνταδικό σύστημα με βάση το δηλαδή το 60, το οποίο έχει εmβιώσει μέχρι σήμερα στο μέτρημα του χρόνου.

Τα υλικά τεκμήρια για τα βαβυλωνιακά μαθηματικά είναι σε μορφή πήλινων πινακίδων με σφηνοει­δείς επιγραφές. Η χρήση τους ήταν διαδεδομένη και έχουν επιβιώσει εκατοντάδες χιλιάδες δείγματα, από μικρά θραύσματα έως ολόκληρες πλάκες μεγέθους χαρτοφύλακα. Ο πηλός ήταν άφθονος και για όση ώρα

1,000 11000,000

Αρχαία μέτρηση των λαών της Μεσοποταμίας

έμεινε νωπός μπορούσε κανείς να σβήσει ένα υπολογισμό και να ξαναρχίσει από την αρχή . Μόλις ο πηλός σκλήραινε η πινακίδα ήταν άχρηστη και η την πετούσαν η την χρησιμοποιούσαν σαν δομικό υλικό.

Στην Γεωμετρία είχαν μεθόδους για να βρίσκουν το εμβαδόν επιπέδων σχημάτων και πολλά προβλήματα τα έλυναν αλγεβρι­κά. Τους άρρητους αριθμούς οι οποίοι πα­ριστάνονται με άπειρα δεκαδικά ψηφία , τους χειρίζονταν αριθμητικά περικόπτοντας το κλασματικό εξηνταδικό ανάπτυγμα

ΕΥΚΛΕΙΔΗΣ Α. 69 τ. l/8

Επιτροπή Διαγωνισμών της Ε.Μ.Ε.

Jln ΜΕΣΟΓΕΙΑΚΗ ΜΑΘΗΜΑΤΙΚΗ OnYMΠIAQA 2008 εις μνήμην PETER 0' HOLLORAN

Πρόβλη μα 1 ". Έστω c Α ο παρεγεγραμμένος κύκλος του τριγώνου ABC με κέντρο Ια ο οποίος εφάπτεται της

πλευράς BC στο σημείο D και στις προεκτάσεις των πλευρών AC και ΑΒ στα σημεία Ε και Ζ , α­ντίστοιχα. Αν η ευθεία ED είναι κάθετη προς την πλευρά ΑΒ (την οποία τέμνει στο σημείο Η) και η

ευθεία ZF είναι η κάθετη από το Ζ προς την ευθεία Ηlα (την οποία τέμνει στο σημείο F ), να υπολο-

γίσετε τις γωνίες BFD και AFE .

Πρόβλημα 2". Α ν p και q είναι μη μηδενικοί ακέραιοι και η εξίσωση

y4 +2(2p-l)y2 +8qy-(4p-1) =0

έχει μία ακέραια ρίζα ετερόσημη του q , να αποδείξετε ότι η εξίσωση

χ3 +px+q =0

έχει μία τουλάχιστον άρρητη ρίζα.

Πρόβλημα 3". Η τετράδα θετικών ακέραιων ( a, b,c,d) λέγεται «ενδιαφέρουσα», αν οι αριθμοί

az,bz, cz, dz+l

με τη σειρά που δίνονται, είναι διαδοχικοί όροι aύξουσας αριθμητικής προόδου.

α) Βρείτε μία τουλάχιστον «ενδιαφέρουσα» τετράδα. β) Ο αριθμός όλων των τετράδων που είναι «ενδιαφέρουσες» είναι πεπερασμένος ή άπειρος;

Να εξηγήσετε την απάντησή σας.

Πρόβλη μα 4". Έστω ότι x,y,z Ε [Ο, Ι) και χ+ y+ z =Ι. Να αποδείξετε ότι:

+ - -+ - -< -�z 8f;x 3

x+yz y+zx-

2 ·

(1)

(2)

Οι λύσεις στο επόμενο τεύχος

θέματα και t\ύσεις ΠΡΟΚΡΙΜΑΤΙΚΟΥ 2008 Π ΡΟΒΛΗ ΜΑ 1 Να προσδιορίσετε τις τιμές των a Ε JR. και η Ε Ν* για τις οποίες το πολυώνυμο

φ( χ)= χ2 -χ+ ι διαιρεί το πολυώνυμο

f{x)={x-l)n +{χ-2γπ+Ι +(1-χ2}2η+Ι +a. ΕΥΚΛΕΙΔΗΣ Β' 68 τ.4/9

---------- Μαθηματικοί Διαγωνισμοί - Μαθηματικές Ολυμπιάδες ---------Λ:ίJση ο 'ζ λ ' ( ) ? 1 ' 1 ± i.J3 π . . π ι ρι ες του πο υωνυμου φ χ = χ - - χ + ειναι οι χ 1 ? = = cοs-±ι sι η-, ·- 2 3 3

οι οποίες είναι και ρίζες του πολυωνύμου χ 3 +Ι = ( χ +Ι) φ( χ ) . Α ν θέσουμε χ 1 = ω = cos� + i siη � , τότε 3 3 η άλλη ρίζα είναι χ? = ω = cos�- i siη � . Ομως ισχύει ωω = Ι � ω3ω = ω2 � ω = -ω2 ( αφού ω3 = - 1 ) . - 3 3

Επομένως, το πολυώνυμο φ( χ) = χ2 -χ + 1 διαιρεί το πολυώνυμο f(x) =(χ-1)" +(χ-2)2"+1 +( 1-χ2)2n+ι +a , αν, και μόνον αν, f ( ω) = Ο και f ( -ω2 ) = Ο .

Επειδή έχουμε f ( ω) = Ο<::::> (ω - Ι )" + ( ω - 2)2n+Ι + ( 1 - ω2 )2n+ Ι + a = Ο<::::> ω2" + ( ω2 - 1 )2n+Ι + ( 1 - ω2 )2n+ Ι + a = Ο<::> ω2" + a = 0,

διακρίνουμε τις περιπτώσεις: ., Αν η = 3 k, k ε Ν *, τότε f (ω) = O<::::> ω6k + a = O<::::> a = - 1 . � Αν η = 3 k + 1, k ε Ν , τότε f ( ω) = Ο<::::> ω6k+2 + a = Ο<::::> a = -ω2 ε C. • Αν η = 3 k + 2, k ε Ν , τότε f ( ω) = Ο<::::> ω6k+4 + a = Ο<::::> a = ω ε C. Ομοίως έχουμε f (-ω2 ) = 0 <=> (-ω2 - 1 )" + (-ω2 - 2)2n+ Ι + ( 1 - ω4 )2n+Ι + a = O

<::::> (-Ι)" ω" - ( ω2 + 2 )2n+ ι + (Ι+ ω )2n+ι + a = Ο

<::::> (- Ι )" ω" - ( ω + 1γn+ Ι + ( 1 + ω)2'1 + 1 + a = 0 <::::> (-Ι)" ω" + a = Ο, οπότε διακρίνουμε τις περιπτώσεις:

• Αν η = 3 k, k ε Ν *, τότε: • Αν η = 3k + l, k ε Ν , τότε :

• Αν η = 3 k + 2, k ε Ν , τότε:

f ( -ω2 ) = Ο<::::> ( - 1 )3 k+ Ι ω3k+ ι + a = Ο<::::> a = ω ε C .

f -ω- = Ο<::::> - 1 ω - + a = Ο<::::> a = -ω- ε C. ( ? ) ( )3 k+2 3k+? ?

Συνοψίζοντας τα παραπάνω, λαμβάνουμε: f (ω) = Ο και f ( -ω2 ) = Ο<::::> a = - 1 και η = 3 k, k ε Ν *, δηλαδή έχουμε φ ( χ ) I f ( χ ) <::::> f (ω) = Ο και f (-ω 2 ) = Ο <::::> a = -1 και η = 3 k, k ε Ν * .

Πuραη1ρηση. Αν παρατηρήσουμε ότι το πολυώνυμο ( χ - 2 )2"+ 1 + ( 1 - χ 2 )2,1+

1 έχει παράγοντα το πολυώνυμο

χ - 2 + 1 - χ 2 = - (χ 2 - χ + Ι ) = -φ( χ ) , τότε αρκεί να εξετάσουμε πότε το πολυώνυμο φ( χ ) διαιρεί το

πολυώνυμο g ( x ) = ( x - 1 )" + a.

ΓJΙ>Oii.\H\'IA 2 Στο χωριό Χ0 βρίσκονται 80 εκδρομείς οι οποίοι πρόκειται να επισκεφθούν 5 γειτονικά χωριά

Χ1, Χ2 , Χ3 , Χ4 και Χ5 • Καθένας από αυτούς έχει μία μόνο πρώτη επιλογή. Όμως, υπάρχουν περι­

πτώσεις που για την επίσκεψη σε κάποιο χωριό υποχρεώνονται οι εκδρομείς να επισκεφθούν και ένα ή περισσότερα από τα υπόλοιπα χωριά. Κάθε εκδρομέας επισκέπτεται μόνο την πρώτη επιλογή του και όσα χωριά υποχρεώνεται να επισκεφθεί. Α ν επισκέφθηκαν τα χωριά Χ1, Χ2 , Χ3 , Χ4 και Χ5 συνολικά 40, 60, 65, 70 και 75 εκδρομείς, αντίστοιχα, τότε να βρείτε πόσες πρώτες επιλογές εκδρο­

μέων αντιστοιχούν σε κάθε χωριό και τα διατεταγμένα ζεύγη ( X;,Xj ), i,j = 1 , 2, 3, 4, 5 , που είναι τέ-

τοια ώστε για την επίσκεψη στο χωριό Χ; υποχρεώνεται ο εκδρομέας σε επίσκεψη και στο χωριό

χ .. . I

ΕΥΚΛΕ ΙΔΗΣ Β ' 69 τ.t/10

---------- Μαθη ματικοί Διαγωνισμοί - Μαθηματικές Ολυμπιάδες

Αν η επίσκεψη στο χωριό Χ2 υποχρέωνε τον εκδρομέα σε επίσκεψη και στο χωριό Χι, τότε θα επι­σκέπτονταν το χωριό Χι τουλάχιστον οι 60 εκδρομείς που επισκέφθηκαν το χωριό Χ2 , που δεν ισχύει, γιατί τελικά το επισκέφθηκαν μόνο 40. Άρα η επίσκεψη στο χωριό Χ2 δεν υποχρεώνει τον εκδρομέα σε επίσκεψη και στο χωριό Χι. Ομοίως προκύπτει ότι η επίσκεψη στα χωριά Χ3,Χ4 και Χ5 δεν υποχρεώ­νει σε επίσκεψη και στο χωριό Χι . Σημειώνουμε με χκ τον αριθμό των επισκέψεων στο χωριό )\,κ=ι2,3,4,5. Επειδή ισχύει χι+ χκ > 80, για κάθε κ = 2, 3,4, 5 , έπεται ότι υπάρχει ένας τουλάχιστον εκδρομέας που επισκέπτεται το χωριό Χι, αλλά και το χωριό Χκ. Αν αυτός ο εκδρομέας έχει ως πρώτη επιλογή το χω­ριό Χλ, λ = 1 , 2, 3, 4, 5 , τότε αυτό είναι το Χ ι , δηλαδή ισχύει Χλ =Χι και η επίσκεψη στο Χι υποχρεώνει τον εκδρομέα σε επίσκεψη και στο χωριό Χκ, κ = 2, 3,4, 5 .

Επομένως, οι Χι = 40 εκδρομείς έχουν το χωριό Χι ως πρώτη επιλογή , ενώ η επίσκεψη στο χωριό Χι υποχρεώνει τον εκδρομέα σε επίσκεψη σε όλα τα υπόλοιπα χωριά, δηλαδή 40 εκδρομείς θα επισκε­φθούν και τα χωριά Χ2,Χ3,Χ4 και Χ5 .

Έτσι απομένουν 40 εκδρομείς οι οποίοι θα κάνουν y2 = 20, y3 = 25, y4 = 30 και y5 = 35 επισκέψεις στα χωριά Χ2, Χ3, Χ4 και Χ5 , αντίστοιχα. Επειδή ισχύει ότι y 2 + y κ > 40, για κάθε κ = 3, 4, 5 , σκεπτό­μενοι όπως παραπάνω, συμπεραίνουμε ότι οι y2 = 20 εκδρομείς έχουν ως πρώτη επιλογή το χωριό Χ2,

στο οποίο η επίσκεψη υποχρεώνει σε επίσκεψη στα χωριά Χ3,Χ4 και Χ5 , και ότι η επίσκεψη σε κάποιο από τα χωριά Χ3,Χ4 και Χ5 δεν υποχρεώνει σε επίσκεψη και στο χωριό Χ2.

Έτσι οι 20 εκδρομείς με πρώτη επιλογή το χωριό Χ2 θα επισκεφθούν και τα χωριά Χ3, Χ4 και Χ5 ,

οπότε απομένουν 20 εκδρομείς οι οποίοι θα κάνουν συνολικά z3 = 5, z4 = 1 Ο και z5 = 1 5 επισκέψεις στα χωριά Χ3, Χ4 και Χ5 , αντίστοιχα.

Σκεπτόμενοι, όπως προηγουμένως, συμπεραίνουμε ότι η επίσκεψη στα χωριά Χ4 και Χ5 δεν υπο­χρεώνει σε επίσκεψη στο χωριό Χ3• Επίσης η επίσκεψη στο χωριό Χ5 δεν μπορεί να επιβάλλει επίσκεψη στο χωριό Χ4 . Όμως , επειδή είναι z4 + z5 > 20 , υπάρχει ένας τουλάχιστον εκδρομέας που επισκέπτεται και τα δύο χωριά Χ4 και Χ5 • Έτσι διακρίνουμε τις περιπτώσεις:

J< •; '-:-:1 πι:υοrτυ'(�"Ι1 Ο εκδρομέας αυτός έχει ως πρώτη επιλογή το χωριό Χ3, οπότε υποχρεώνεται σε επίσκεψη και στα

χωριά Χ4 και Χ5. Τότε 5 εκδρομείς θα επισκεφθούν τα χωριά Χ4 και Χ5 , οπότε απομένουν 1 5 εκδρο­μείς οι οποίοι θα κάνουν w 4 = 5 και w 5 = 1 Ο επισκέψεις στα χωριά Χ4 και Χ5 , αντίστοιχα. Άρα, λαμ­βάνοντας υπόψη ότι οι πρώτες επιλογές που απομένουν είναι 20, προκύπτει ότι z3 = 5, w 4 = 5 και w 5 = 1 Ο είναι οι πρώτες επιλογές που αντιστοιχούν στα χωριά Χ3, Χ4 και Χ5 , αντί­στοιχα. Σε μορφή πίνακα οι επισκέψεις δίνονται ως εξής:

Χι χ2 χ3 χ4 Xs ·1 � '

-�:,· ., �� 40 40 40 40 · - 20 20 20

5 5

.: ;.

40 60 65 70 75

Τα διατεταγμένα ζεύγη (x;,Xj), i , j = 1 , 2, 3,4, 5 , που είναι τέτοια ώστε για την επίσκεψη στο χωριό Χ; υποχρεώνεται ο εκδρομέας σε επίσκεψη και στο χωριό Χ j στη περίπτωση αυτή είναι τα εξής:

(Χι ,Xj ) , j = 2, 3,4, 5 , (ΧυΧk ), k = 3,4, 5 και (Χ3,Χ11, ) , m = 4, 5 .

Οι κινήσεις στην περίπτωση αυτή μπορούν να δοθούν από το διάγραμμα που ακολουθεί

ΕΥΚΛΕΙΔΗΣ Β' 69 τ. 1/1 1

--------- Μαθηματικοί Διαγωνισμοί - Μαθηματικές Ολυμπιάδες

20 .. Rl � ....

I

[ΞJ 5

Ο εκδρομέας αυτός έχει ως πρώτη επιλογή το χωριό Χ4 , οπότε του επιβάλλεται η επίσκεψη και στο χωριό Χ5 . Τότε 1 Ο εκδρομείς θα επισκεφθούν το χωριό Χ5 , οπότε απομένουν 1 Ο εκδρομείς οι οποίοι θα κάνουν z3 = 5 και w 5 = 5 επισκέψεις στα χωριά Χ3 και Χ5 , αντίστοιχα. Άρα στην περίπτωση αυτή z3 = 5, w 1 = Ι Ο και w 5 = 5 είναι οι πρώτες επιλογές που αντιστοιχούν στα χωριά Χ3, Χ4 και Χ5 , αντί­στοιχα. Σε μορφή πίνακα οι επισκέψεις δίνονται ως εξής:

Χι χ2 χ3 χ4 Χ5

: : 40 40 40 40 -;_-, 20 20 20 -·...,i.

;',

ι: ι 10 �·

40 60 65 70 75 Τα διατεταγμένα ζεύγη (x;,Xj ), i,j = 1,2,3,4,5 , που είναι τέτοια ώστε για την επίσκεψη στο χωριό

Χ; υποχρεώνεται ο εκδρομέας σε επίσκεψη και στο χωριό Χ j στη περίπτωση αυτή είναι τα εξής:

(X1 ,Xj), j = 2,3,4,5, (ΧυΧk ), k = 3,4,5 και (Χ4,Χ5 ).

Οι κινήσεις στην περίπτωση αυτή μπορούν να δοθούν από το διάγραμμα που ακολουθεί

20

χ2 ο--�

I I

40 i /

1 0

5

······tm , ! I

ΕΥΚΛΕΙΔΗΣ Β ' 69 τ. l/12

---------- Μαθη ματικοί Διαγωνισμοί - Μαθηματικές Ολυμπιάδες

Οι διχοτόμοι των γωνιών Α, Β και Γ τριγώνου ΑΒΓ τέμνουν το περιγεγραμμένο κύκλο C 1 (0, R)

στα σημεία Α2 ,Β2 και Γ2 , αντίστοιχα. Οι εφαπτόμενες στα σημεία Α2 ,Β2 και Γ2 του περιγεγραμ­

μένου κύκλου τέμνονται στα σημεία Α 3,83 και Γ3 (τα σημεία Α 3 και Α βρίσκονται στο ίδιο ημιε­

πίπεδο ως προς την ευθεία ΒΓ, τα σημεία Β 3 και Β βρίσκονται στο ίδιο ημιεπίπεδο ως προς την ευ­

θεία ΑΓ και τα σημεία Γ3 και Γ βρίσκονται στο ίδιο ημιεπίπεδο ως προς την ευθεία ΑΒ). Α ν ο εγ­

γεγραμμένος κύκλος C2 (1,ρ) του τριγώνου ΑΒΓ εφάπτεται στις πλευρές ΒΓ,ΑΓ,ΑΒ στα σημεία

Α 1,Β 1 και Γ 1 , αντίστοιχα, να αποδείξετε ότι οι ευθείες Α 1Α2 ,Β 1 Β2 ,Γ1 Γ2 ,ΑΑ 3,ΒΒ 3 και ΓΓ3 περνάνε

από το ίδιο σημείο.

Έστω Ι το έκκεντρο και Ο το περίκεντρο του τριγώνου ΑΒΓ . Τότε ΙΑ .1 Β1 Γ1 , ΙΒ .1 Α1Γ1 και ΙΓ .1 Α1Β1 • Για τις παρακάτω ισότητες γωνιών χρησιμοποιούμε το γεγονός ότι τα σημεία Α2 , Β2 , Γ2 , είναι μέσα

Λ Λ Λ ο ο ξ Ι ο ο ο Λ Γ Λ Α Λ Β Ά ο ΙΑ .1 Β Γ των αντιστοιχων το ων. σχυουν οι ισοτητες γωνιων: χ = l και y = 2 ω = l. ρα εχουμε : 2 2 •

Με όμοιο τρόπο αποδεικνύουμε ότι: ΙΒ .1 Α2Γ2 και ΙΓ .1 Α2Β2 , οπότε συμπεραίνουμε ότι Α 1Β 1 I/ Α2Β2 , Α 1 Γ1 // Α2Γ2 και Β 1 Γ1 // Β2Γ2 •

Άρα τα τρίγωνα Α1Β 1Γ1 και Α2Β2Γ2 είναι ομοιόθετα και κατά συνέπεια οι ευθείες Α1Α2 , Β 1 Β2 και Γ1 Γ2 που συνδέουν τις ομόλογες κορυφές τους θα συντρέχουν στο κέντρο της ομοιοθεσίας (που έστω ότι είναι το σημείο Ρ ) . Το σημείο Ο είναι το περίκεντρο του τριγώνου Α2Β2Γ2 και το έκκεντρο f του τρι­γώνου ΑΒΓ , είναι περίκεντρο του τριγώνου Α1 Β 1 Γ1 • Επειδή όμως τα τρίγωνα Α1 Β1 Γ1 , Α2 Β2Γ2 είναι ο­μοιόθετα, θα είναι ομοιόθετα και τα περίκεντρά τους, δηλαδή το σημείο Ο είναι το ομοιόθετο του ση-

μείου Ι στην ομοιοθεσία με κέντρο το σημείο Ρ και λόγο λ = R

(όπου R και ρ είναι οι ακτίνες του ρ

περιγεγραμμένου C 1 (0, R) και εγγεγραμμένου C2 (1, ρ) κύκλου του τριγώνου ΑΒΓ αντίστοιχα) .

Προφανώς λοιπόν οι κύκλοι C 1 (0,R) και C2 (1, ρ) είναι ομοιόθετοι στην ομοιοθεσία με κέντρο το

σημείο Ρ και λόγο λ = R

. Επιπλέον μπορούμε να συμπεράνουμε ότι και τα σημεία Ο, Ι , Ρ είναι συνευ­ρ

θειακά και ΡΟ = λ·ΡΙ (1)

ΕΥΚΛΕΙΔΗΣ Β ' 69 τ. l/13

--------- Μαθηματικοί Διαγωνισμοί - Μαθηματικές Ολυμπιάδες ---------

Η Α383 εφάπτεται του κύκλου Cι (O, R) στο μέσο Γ2 του τόξου Α8 , οπότε θα είναι Α383 // Α8 . Ομοίως προκύπτει ότι Α3Γ3 I/ ΑΓ και 8mrm I/ 8Γ .

Από τις τρεις προηγούμενες παραλληλίες συμπεραίνουμε ότι τα τρίγωνα ΑΒΓ και Α383Γ3 είναι ο­μοιόθετα και κατά συνέπεια οι ευθείες ΑΑ3 , 883 και ΓΓ3 που συνδέουν τις ομόλογες κορυφές τους θα συντρέχουν στο κέντρο της ομοιοθεσίας (που έστω ότι είναι το σημείο Τ ) .

Οι κύκλοι C ι (O, R) και C2 (Ι , ρ) είναι οι εγγεγραμμένοι κύκλοι των ομοιόθετων τριγώνων Α383Γ1

και Α8Γ αντίστοιχα στην ομοιοθεσία με κέντρο το Τ και λόγο λ = R . Επιπλέον μπορούμε να συμπε­ρ

ράνουμε ότι και τα σημεία Ο, Ι , Τ είναι συνευθειακά και ΤΟ = λ · τι (2) Από τις σχέσεις ( 1 ) και (2) συμπεραίνουμε ότι τα σημεία Ρ και Τ ταυτίζονται. Πράγματι, με αφαί-

ρεση κατά μέλη των ( 1 ) και (2) λαμβάνουμε ΡΟ - ΤΟ = λ ( ΡΙ - τι ) => ΡΤ = λΡΤ :::::> (λ - 1 ) ΡΤ = 6 , οπότε ' αφού είναι λ = R > 1 ' έπεται ότι ΡΤ = 6 => Ρ Ξ τ .

ρ

Μπορούμε να αποδείξουμε ότι τα σημεία Ρ και Τ ταυτίζονται και με τον εξής τρόπο : Τα σημεία Α υ 82 , Γ2 είναι τα σημεία επαφής του εγγεγραμμένου κύκλου του τριγώνου Α383Γ3

C ι (O, R) με τις πλευρές του. Τα σημεία Αι , 8ι , Γι είναι τα σημεία επαφής του εγγεγραμμένου κύκλου του τριγώνου Α8Γ C2 (Ι , ρ) με τις πλευρές του.

Επειδή όμως τα τρίγωνα είναι ομοιόθετα, οι ευθείες Αι Α υ 8ι 82 και Γι Γ2 που ορίζουν τα ομόλογα σημεία επαφής θα διέρχονται από το κέντρο της ομοιοθεσίας Τ .

Δίνεται η εξίσωση χ2 + y2 - axy + 2 = Ο , (1 ) όπου a θετική ακέραια παράμετρος. Ν α αποδείξετε ότι:

(i) Για a "* 4 , δεν υπάρχουν ζεύγη θετικών ακεραίων ( χ, y ) που είναι λύσεις της εξίσωσης (1 ) . (ii) Για a = 4 , υπάρχουν άπειρα ζεύγη θετικών ακεραίων ( x, y ) που είναι λύσεις της εξίσωσης

(1), τα οποία να προσδιορίσετε.

ΕΥΚΛΕΙΔΗΣ Β ' 69 τ. l/14

---------- Μαθηματικοί Διαγωνισμοί - Μαθηματικές Ολυμπιάδες

(i) Αν a -::F- 4 και ( χ , y) με χ , y θετικούς ακέραιους είναι μία λύση της ( 1 ) , τότε πρέπει χ -::F- y . Πράγματι, αν ήταν χ = y , τότε θα είχαμε

2χ 2 - ax 2 + 2 = Ο <=> ( a - 2) χ 2 = 2 <=> χ = 1 και a = 4, που είναι άτοπο.

Έστω τώρα μία λύση ( x 0 , y0 ) της εξίσωσης χ 2 + y2 - axy + 2 = Ο ( 1 ), με χ 0 , Υ ο θετικούς ακέραιους, χ0 > y0 . Τότε ισχύει ότι χ� + y� - ax0y0 + 2 = Ο και

( )2 ? ( ) 2 ? ? 2 ο ayo - Χ ο + Υο - a ayo - Χ ο Υ ο + = χ;; + Υο - aχσΥο + = , οπότε και το ζεύγος ( χ 1 , y 1 ) = ( ay0 - χ0 , Υ ο ) είναι επίσης λύση της ( 1 ) , αφού ισχύει και η απαιτούμενη

ανισότητα ay0 - χ0 > Ο . Η τελευταία προκύπτει από τη σχέση χ0 ( ay0 - χ 0 ) = ax0y0 - χ� = y� + 2 > Ο . Επιπλέον ισχύει ότι χ0 > Χ 1 = ay0 - Χ 0 • Πράγματι, έχουμε χ0 > χ 1 = ay0 - Χ 0 <=> χ� > Χ 0 ( ay0 - Χ0 ) <=> χ� > Υ� + 2 <=> χ� - y� > 2 ,

που ισχύει για χ0 , Υ ο θετικούς ακέραιους με χ0 > Υ ο . Έτσι με τη διαδικασία αυτή προκύπτει ακολουθία ζευγών θετικών ακεραίων

( χ ο , Υο ) , ( χ ι , Υ ι ) , . . . , ( χ η , Υn ) , . . . , οι όροι της οποίας είναι λύσεις της εξίσωσης ( 1 ) και ικανοποιούν τις ανισότητες χ0 > χ 1 > . . . > χ" > . . . .

Αυτό, όμως είναι άτοπο, αφού δεν υπάρχει ακολουθία θετικών ακεραίων που να είναι γνησίως φθίνου­σα. Ομοίως καταλήγουμε σε άτοπο, αν υποθέσουμε ότι υπάρχει λύση ( χσ,y0) της εξίσωσης ( 1 ), με χ0 , y0 θε-

τικούς ακέραιους, Χο <y0 • Πράγματι, λόγω συμμετρίας και το ζεύγος ( y0 , x0 ) είναι λύση με y0 >χσ . Για a = 4 η εξίσωση γίνεται x2 +J -4xy+ 2 =0 (2) και έχει την προφανή λύση ( x0 , y0 ) = ( 1 , 1 ) .

Πράγματι, αν αναζητήσουμε λύσεις της μορφής ( χ , y) με χ = y , τότε προκύπτει ότι χ = y = 1 . Στη συνέχεια παρατηρούμε ότι και το ζεύγος ( x 1 , y 1 ) = (4x 0 - y0 , x 0 ) = (3, 1 ) είναι επίσης λύση της

εξίσωσης (2), όπως είναι και τα ζεύγη ( yo , xo ) , ( χ σ , 4Χ ο - yo ) , ( Υο , 4Υο - χ σ ) και (4Υο - Χο , Υο ) · (3 )

Θεωρούμε τώρα την τυχούσα λύση ( x 1 , y 1 ) της εξίσωσης (2), όπου x 1 , y 1 ακέραιοι με χ 1 > y1 2': 1 . τ ' ' ' (4 )2 ? 4 (4 ) 2 ? ? 4 2 ο οτε ισχυει οτι χ 1 - y1 + Χ ϊ - χ 1 - y1 χ 1 + = Χ ϊ + Υϊ - x 1 y 1 + = , δηλαδή και το ζεύγος ( x 2 , y2 ) = (4x 1 - y1 , x 1 ) είναι λύση της (2) με x 1 , 4y 1 - χ 1 θετικούς ακέραιους, α­

φού από τη σχέση ( 4χ 1 - Υι ) y 1 = 4x 1 y1 - y� = χ � + 2 > Ο , έπεται ότι 4χ 1 - y1 > Ο . Επιπλέον ισχύει

y2 = Χ ι > Υ ι και χ 2 = 4χ ι - Υ ι = 3χ ι + ( Χ ι - Υ ι ) > 3χ ι > Χ ι . Επομένως, με τη διαδικασία που χρησιμοποιήσαμε, μπορεί να προκύψει απειρία ζευγών θετικών ακε-

ραίων ( X0 , y0 ) , ( Χ 1 , y 1 ) , . . . , ( Χ 11 , y η ) , . . . , τέτοια ώστε

και επιπλέον ισχύει

Χο < Χ ι < . . . < x n < . . . και Υο < Υ ι < . . . < yn < . . . . {Χ n+ Ι = 4xn - y" } _ , η = 0, 1 , 2 , . . . και χ0 = y0 = 1 . Υn+ ι - χ"

' ' ' ' ' {xn+2 = 4x n+ l - x n } Απο το παραπανω συστημα προκυπτει οτι _

, η = 0, 1 , 2, . . . και χ0 = y0 = l , x 1 = 3 . Yn+ I - X n

Έτσι, οι λύσεις που προκύπτουν με τη παραπάνω διαδικασία είναι οι ( Χ 11 , Χ 11 _ 1 ) , η = 1 , 2, . . . , και όσες

απορρέουν από αυτές της μορφής (3 ), όπου η ακολουθία ( χ" ) ικανοποιεί μία αναδρομική σχέση δεύτε-

ρης τάξης της μορφής Χ 11 +2 = 4x n+ l - Χ 11 , η = 0, 1 , 2, . . . Και Χ0 = 1 , Χ 1 = 3 οπότε ο γενικός της όρος είναι της μορφής χ " = c 1 α11 + c2β

11 , όπου α = 2 - .J3, β = 2 - .J3 είναι οι ρίζες της χαρακτηριστικής εξίσωσης t 2 - 4t + 1 = Ο .

Οι σταθερές c 1 , c2 προκύπτουν από τις αρχικές συνθήκες

ΕΥΚΛΕΙΔΗΣ Β' 69 τ. l/1 5

--------- Μαθηματικοί Διαγωνισμοί - Μαθηματικές Ολυμπιάδες ---------

3 + J3 3 - J3 c 1 + c2 = χ0 = 1, c 1 α + c2β = χ 1 = 3 <:::> C 1 = -

6- , C2 = -

6- .

Θεωρώντας επίσης την τυχούσα λύση (χ , y) της εξίσωσης (2), όπου χ , y ακέραιοι με χ > y � 1 , πα­

ρατηρούμε, όπως παραπάνω, ότι και το ζεύγος ( y, 4y - χ ) είναι επίσης λύση της εξίσωσης (2) με

4y - χ S:: y . Πράγματι, έχουμε 4y - χ S:: y <:::> 3y - χ S:: Ο <:::> χ - 3y � Ο , η οποία, αφού χ - y > Ο , είναι ισο­

δύναμη με την ανισότητα (χ -y) ( χ -3y) � Ο ή χ2 +31 -4xy�O ή 2 (/ - 1) � 0.

Έτσι, αν είναι y > 1 , τότε λαμβάνουμε 4y - χ < y , οπότε με την καθοδική αυτή διαδικασία θα φθά­

σουμε μετά από ορισμένα βήματα σε μία λύση ( χ , y) με y = 1 και εφαρμόζοντας τη διαδικασία που πε­ριγράψαμε παραπάνω θα βρούμε τις άπειρες λύσεις που προσδιορίσαμε προηγουμένως.

Η 25η Βαλκανική Μαθηματική Ολυμπιάδα διεξήχθη στην Οχρίδα της ΠΓ ΔΜ από 4 έως 1 Ο Μα"fου 2008 με τη συμμετοχή συνολικά 1 8 χωρών. Συμμετείχαν οι χώρες :

Αλβανία, Βοσνία και Ερζεγοβίνη, Βουλγαρία, Ελλάδα, Κύπρος, Μαυροβούνιο, Μολδαβία, ΠΓΔΜ, Ρουμανία, Σερβία, Τουρκία . Ως φιλοξενούμενες συμμετείχαν οι χώρες: Αζερμπαϊτζάν, Γαλλία, Ηνωμένο Βασίλειο, Ιταλία, Καζακστάν, Τατζικιστάν και Τουρκμενιστάν. Την Ελληνική ομάδα αποτελούσαν οι μαθητές:

Αργυρό μετάλιο Χάλκινο μετάλιο Χάλκινο μετάλιο Χάλκινο μετάλιο Χάλκινο μετάλιο Συμμετοχή.

Συνοδοί της αποστολής ήταν ο Αναπληρωτής καθηγητής του ΕΜΠ κ. Ανάργυρος Φελλούρης (αρχηγός) και ο καθηγητής Μαθηματικών Δευτεροβάθμιας Εκπαίδευσης κ. Ευάγγελος Ζώτος (υπαρχηγός).

Δίνεται οξυγώνιο σκαληνό τρίγωνο ABC με AC > BC . Έστω Ο το κέντρο του περιγεγραμμένου κύκλου, Η το ορθόκεντρο του (σημείο τομής των υψών) και F το ίχνος του ύψους του από το C. Έστω Ρ σημείο της ευθείας ΑΒ διαφορετικό του Α τέτοιο ώστε AF = PF, και Μ το μέσον του AC. Συμβολίζουμε το σημείο τομής των ευθειών ΡΗ και BC με Χ, το σημείο τομής των ευθειών ΟΜ και FX με Υ, και το σημείο τομής των ευθειών OF και AC με Ζ. Να αποδείξετε ότι τα σημεία F, Μ, Υ και Ζ είναι ομοκυκλικά.

Επειδή είναι ΟΜ ..l AC , αρκεί να αποδείξουμε ότι OF ..l FX (Σχήμα 1 ) . Αν ΟΕ ..l ΑΒ , τότε είναι φα-νερό ότι CH = 2 · ΟΕ . ( 1 ) Από την υπόθεση έχουμε PF = AF , οπότε ΡΒ = PF - BF ή ΡΒ = AF - BF . (2)

Επίσης έχουμε ΧΡΒ = ΗΑΡ και ΗΑΡ = HCX , αφού το τετράπλευρο AFGC είναι εγγράψιμο ( G εί-"' " " "

ναι το ίχνος του ύψους από το Α ) . Άρα έχουμε XPF = HCX και αφού ΒΧΡ = HXC , τα τρίγωνα XHC και ΧΒΡ είναι όμοια. Α ν XL και XD είναι ύψη των τριγώνων XHC και ΧΒΡ , αντίστοιχα, τότε θα έ­

ΧD ΡΒ χουμε την ισότητα η οποία με χρήση των ( 1 ) και (2) γίνεται

XD AF - BF FE XD FE Ε . . XFD OEF · · · λ β · - = = - :::::> - = - . πομενως τα τριγωνα και ειναι ομοια, οποτε αμ ανου-

ΧL 20Ε ΟΕ FD ΟΕ με: OFX = OFC + LFX = FOE + FXD = XFD + FXD = 90° :::::> OF ..l FX.

Αρκεί να αποδείξουμε ότιt OF ..l FX . Θεωρούμε την αρχή του μιγαδικού επιπέδου στο περίκεντρο του τριγώνου ABC . Έστω a, b, c οι μιγαδικοί αριθμοί που αντιστοιχούν στις κορυφές Α, Β και C , αντί-στοιχα, του τριγώνου ABC , με aa = bb = cc = 1 . Τότε έχουμε

ΕΥΚΛΕΙΔΗΣ Β ' 69 τ. l/16

--------- Μαθηματικοί Διαγωνισμοί - Μαθηματικές Ολυμπιάδες ---------

a + b + c - abc _ h = a + b + c, f = , p = b + c - abc 2

Σχήμα Ι Το σημείο Χ είναι η τομή των ΗΡ και BC. Τότε έχουμε

και επίσης

c - x c - b - b + c - x -- = -- = -bc => χ = ---- - bc c- χ c- b

p - x p - h p- x p- h

o - f f - x f f - x Για να αποδείξουμε ότι OF .l FX , αρκεί να αποδείξουμε ότι: -- = - -- <::::> - = - -- <::::>

σ- r r- x r r- χ

Πρώτα αποδεικνύουμε το ακόλουθο λήμμα.

( 1 )

(2)

Έστω Τ η τομή του ύψους από την κορυφή C με τον περιγεγραμμένο κύκλο του τριγώ­νου ABC. Αν D είναι η τομή των CA και ΒΤ, Ε είναι η τομή ΑΤ και CB, τότε σχηματίζουμε το πλή­ρες τετράπλευρο Τ ACBED. Αν F είναι το ίχνος του ύψους από την κορυφή C , τότε OF .l DE.

Στο πλήρες τετράπλευρο FTOACB (σχήμα 2), μπορούμε να αποδείξουμε ότι (E,T,R,A)=- 1 και (Ε,Β, R' ,C)=- 1 . Επομένως, η OF είναι η πολική του Ε ως προς τον κύκλο (0), οπότε ΟΕ .l OF. Ομοίως αποδεικνύουμε ότι 00 .l EF, οπότε το σημείο F είναι το ορθόκεντρο του τριγώνου ΟΕ, δηλαδή έχουμε F .l DE. ο

Για την απόδειξη του προβλήματος είναι αρκετό να αποδείξουμε ότι .LYFZ = 90° . Σε συνδυασμό με το προηγούμενο λήμμα, αρκεί να αποδείξουμε ότι FX 1 1 ΟΕ ή ισοδύναμα αρκεί να αποδείξουμε ότι BF ΒΧ

- - -

BQ ΒΕ Επειδή το τετράπλευρο Α ΤΡΗ είναι ρόμβος ( Τ είναι το συμμετρικό του Η ως προς την ευθεία

ΑΒ ) , , ΒΧ ΒΡ Ε , , , , επεται οτι - = - . πομενως, αρκει να ισχυει

ΒΕ ΒΑ

ΕΥΚΛΕΙΔΗΣ Β' 69 τ. l/1 7

--------- Μαθηματικοί Διαγωνισμοί - Μαθηματικές Ολυμπιάδες --------BF

= ΒΡ

ή ΒΡ

=ΒΑ � FA - FB

=ΒΑ � FA

= 1 +ΒΑ

= AQ . Από το πλήρες τετράπλευρο

BQ ΒΑ FB BQ FB BQ FB BQ BQ FA __

AQ TACBED παρατηρούμε ότι (A,F,B,Q) = - 1 , το οποίο δίνει την ισότητα FB BQ

Τi:ταρτη λ\Jση (Αναστάσιος Βαφείδης)

Ί I Ί

I I

I

Ί, I I I

' I

Ί

,t-/,1

. /

--�l· ___ :j __ _ -

I ι'

_,·

,/

/ ι' /

Σχήμα 2

/ ι rl /;

,/

I ι' ' I ; • Ί / .· I ·' ' ι ,' , I ' '

'1' ' ι' '

/ . Ι :;

Η λύση αυτή στηρίζεται σε διπλή εφαρμογή της τριγωνομετρικής μορφής του θεωρήματος Ceva. Αρκεί να αποδείξουμε ότι OF .l FX . Θέτουμε (σχήμα I )

LOFH = χ < 90' , LXFB = y < 90' ( αφού AC > BC ) . Τότε έχουμε LOCA = 90' - Β = LOAC, LOAB = 90' - Β , LOCF = LACF - LACO = Β - Α . Χρησιμοποιώντας τη τριγωνομετρική μορφή του θεωρήματος του Ceva στο τρίγωνο ACF με τις

AO,CO,BO συντρέχουσες, λαμβάνουμε

s1n χ sin ( 90' - C) sin ( 90' - Β ) sin χ sin ( 90' - C) --,-----""7 · · = I => · = I ( I ) sin ( 90' - χ ) sin ( 90' - Β ) sin (Β - Α ) sin ( 90' - χ ) sin (Β - Α )

Τα τρίγωνα ACP και ΑΗΡ είναι ισοσκελή, οπότε έχουμε LHAP = LHP Α = 90' - Β , LHPC = LFPC - LHPF = Α - ( 90' - Β ) = 90' - C ,

LFCB = 90' - Β, LBCP = LFCP - LFCB = ( 90' - Α ) - ( 90' - Β ) = 90' - C . Χρησιμοποιώντας ξανά το τριγωνομετρική μορφή του θεωρήματος του Ceva στο τρίγωνο FCP ως

προς το σημείο Χ λαμβάνουμε

sin y sin ( 90' - Β ) sin ( 90' - C) -....,.-----'--------""7" • • = 1 sin ( 90' - y ) sin (Β - Α) sin ( 90' - Β )

sin y sin ( 90' - C) -....,...--__.:._____,... . = 1 . sin (90' - y ) sin (B - A)

Από τις ( I ) και (2) προκύπτει η ισότητα sin χ sin y

sin ( 90' - χ ) sin ( 90' - y )

ΕΥΚΛΕΙΔΗΣ Β ' 69 τ. l/18

(2)

(3 )

--------- Μαθη ματικοί Διαγωνισμοί - Μαθηματικές Ολυμπιάδες

Από την (3) χρησιμοποιώντας τριγωνομετρικούς μετασχηματισμούς ή τη μονοτονία των τριγωνομε­τρικών συναρτήσεων βρίσκουμε ότι χ = y , το οποίο τελικά δίνει .LOFX = 90° .

Ομοίως, αρκεί να αποδείξουμε ότι OF .l FX . Θεωρώντας τον άξονα χ 'χ .l ΑΒ και τα σημεία F (O, O) , A ( -p , O ) , B (b , O ) , C (O, c ) λαμβάνουμε την εξίσωση της ευθείας ΑC και τις συντεταγμένες των

σημείων Η (ο bp ) x ( bc 2 - b2p bcp - cb2 ) , και 7 7 , 7 7 •

c c- - b- c- - b-Τελικά υπολογίζουμε το συντελεστή διεύθυνσης των ευθειών OF, ΟΧ και βρίσκουμε ότι

λ0F • λ0χ = - 1 . Άρα είναι OF .l FX .

Υπάρχει ακολουθία a 1 , a2 , • • • , aη , · · · θετικών πραγματικών αριθμών η οποία ικανοποιεί και τις

δύο παρακάτω συνθήκες: η η ι (ί) Σ a; ::; n 2 , για κάθε θετικό ακέραιο η, (ίί) Σ- ::; 2008 , για κάθε θετικό ακέραιο η.

i = Ι i = ι a;

Η απάντηση είναι όχι. Για την απόδειξη αυτού είναι αρκετό να αποδείξουμε ότι:

Α < 2 • θ · Σ

2 , 1 η c · ·

θ · δ · ·

λλ ν a ι + a2 + . . . + a, _ η , για κα ε η , τοτε - > - η μπορει να χρησιμοποιη ει οποια ηποτε α η i=2 a ; 4

ακριβής προσέγγιση) . Πράγματι, από την ανισότητα αριθμητικού-aρμονικού μέσου προκύπτει ότι 2''' 2' · I 1 Σ a ; Σ -� 2 2 k , για κάθε k � Ο .

i =zk + ι i=l k + ι a i , , . , ? ' ' ' 2' ' 1 1 ] 2" 1 n- ι 2' ' 1 1 η

Επειδή Σ a ; < Σ a ; ::; 22 k+2 , έπεται ότι Σ -> - , οπότε Σ- > Σ Σ - > - . i =2' + ι i = ι i=2 ' + ι a ; 4 i = 2 a ; k=O i=2 ' + ι a ; 4

Έστω η ένας θετικός ακέραιος. Το ορθογώνιο ABCD με μήκη πλευρών ΑΒ = 90n + 1 και BC = 90n + 5 διαμερίζεται σε μοναδιαία τετράγωνα που έχουν πλευρές παράλληλες προς τις πλευ­ρές του ABCD. Έστω S το σύνολο όλων των σημείων που είναι κορυφές αυτών των μοναδιαίων τε­τραγώνων. Να αποδείξετε ότι ο αριθμός των ευθειών που περνάνε από δύο τουλάχιστον σημεία που ανήκουν στο S διαιρείται με το 4.

Διπλασιάζοντας τις διαστάσεις μπορούμε να θεωρήσουμε με αρχή το κέντρο του ορθογώνιου ABCD και άξονες παράλληλους προς τις πλευρές του . Επομένως έχουμε S={(2a+l,2b+ 1) : a,b εZ,J� :s;45η, J� :s;45η+2} .

Στη συνέχεια θα χωρίσουμε τις ευθείες σε τέσσερις διακεκριμένες κατηγορίες και θα αποδείξουμε ότι ο αριθμός των ευθειών κάθε κατηγορίας διαιρείται με το 4.

Η πρώτη κατηγορία αποτελείται από ευθείες παράλληλες προς μία από τις πλευρές του ορθογώνιου ABCD και ο αριθμός τους είναι 90η + 2 + 90η + 6 = 4 ( 45η + 2) , δηλαδή είναι αριθμός που διαιρείται με το 4.

Η δεύτερη κατηγορία αποτελείται από ευθείες που δεν περνάνε από το κέντρο του ορθογώνιου και δεν είναι παράλληλες προς τις πλευρές του (Σχήμα 3) . Η κατηγορία αυτή περιέχει διαφορετικές ομάδες τεσ­σάρων ευθειών d ι , d2 , d3 , d4 πoυ είναι συμμετρικές ως προς το κέντρο του ορθογώνιου ή ως τους άξονες συμμετρίας του . Η τρίτη κατηγορία αποτελείται από ευθείες που περνάνε από το κέντρο του ορθογώνιου και από ένα σημείο του συνόλου S που βρίσκεται μέσα στο τετράγωνο Τ με διαστάσεις ( 1 80η + 2 ) χ ( 1 80η + 2) , με πλευρές παράλληλες προς τις πλευρές του ορθογώνιου, με το ίδιο κέντρο και περιεχόμενο σε αυτό, εκτός από τις διαγώνιες του τετραγώνου . Η κατηγορία αυτή περιέχει διαφορετικές ομάδες τεσσάρων ευθειών ε ι , ε2 , ε3 , ε4 έτσι ώστε οι ευθείες ε ι , ε2 είναι συμμετρικές ως προς την πρώτη

διαγώνιο, ενώ οι ευθείες ε3 , ε4 είναι συμμετρικές ως προς τη δεύτερη διαγώνιο .

ΕΥΚΛΕΙΔΗΣ Β ' 69 τ. t /19

---------- Μαθηματικοί Διαγωνισμοί - Μαθηματικές Ολυμπιάδες

D c

Α Σχήμα 3 Β Σχήμα 4

Η τέταρτη κατηγορία αποτελείται από τις διαγώνιες του τετραγώνου Τ , καθώς επίσης και εκείνες τις ευθείες που περνάνε από το κέντρο του ορθογώνιου ABCD και από κάποιο σημείο του συνόλου S - Τ , αλλά όχι από κάποιο σημείο του συνόλου S n Τ . Οι ευθείες αυτές χαρακτηρίζονται από το ότι έχουν

κλίση της μορφής Ε. , όπου το κλάσμα Ε. είναι ανάγωγο με q ε { 90η + 3, 90η + 5 } , p περιττό με q q

I P I :::; 90η + I . Για την καταμέτρηση αυτών των ευθειών, σημειώνουμε ότι το σύνολο των περιττών θετι­

κών ακέραιων που είναι σχετικώς πρώτοι με τον 90η + 3 και βρίσκονται στο διάστημα [ -90η - ι, - ι ] εί­

ναι σε ένα προς ένα αντιστοιχία με το σύνολο των άρτιων θετικών ακέραιων που είναι σχετικώς πρώτοι με τον 90η + 3 και βρίσκονται στο διάστημα [ 2, 90η + 2] , σύμφωνα με την αντιστοιχία με τύπο

χ � χ + 90η + 3 . Έτσι το σύνολο των ανάγωγων κλασμάτων της μορφής __ Ρ_ , p περιττός, I PI ::s; 90n + I , 90n + 3

έχει τον ίδιο πληθικό αριθμό με το σύνολο των θετικών ακέραιων που είναι σχετικώς πρώτοι με τον 90η + 3 και βρίσκονται στο διάστημα [ I , 90n + 2 ] , δηλαδή είναι ίσος με φ ( 90η + 3 ) , όπου φ είναι η

γνωστή συνάρτηση Euler. Ομοίως, το σύνολο των ανάγωγων κλασμάτων της μορφής

Ρ , p περιττός, I P I :::; 90η + 3 , 90η + 5

έχει φ ( 90n + 5 ) - 2 στοιχεία, αφού πρέπει να αφαιρέσουμε τα στοιχεία που αντιστοιχούν στις τιμές

± ( 90n + 3 ) . Λαμβάνοντας υπόψη και τις δύο διαγώνιες του τετραγώνου ο συνολικός αριθμός των ευ­

θειών της τέταρτης κατηγορίας είναι φ ( 90n + 3 ) + φ ( 90η + 5 ) . Απομένει να αποδείξουμε ότι οι αριθμοί φ (90η + 3 ) και φ (90η + 5 ) διαιρούνται με το 4. Πράγματι,

επειδή είναι (3, 30η + l ) = ι , έχουμε ότι φ (90η + 3 ) = φ (3 )φ (30n + Ι ) = 2φ (30η + ι ) και ο αριθμός φ ( 30n + ι ) είναι άρτιος, ενώ ομοίως για τον αριθμό φ ( 90n + 5 ) έχουμε

φ (90n + 5 ) = φ (5 )φ ( Ι 8n + ι ) = 4φ ( Ι 8η + Ι ) .

Έστω c ένας θετικός ακέραιος. Η ακολουθία a 1 , a2 , • • • , a. , . . . ορίζεται από a1 = c , και

a.+ ι = a� + a. + c3 , για κάθε θετικό ακέραιο η. Βρείτε όλες τις τιμές του c για τις οποίες υπάρχουν

κάποιοι ακέραιοι k � 1 και m � 2 , τέτοιοι ώστε ο a� + c3 είναι δύναμη με βάση κάποιο θετικό ακέ­

ραιο και εκθέτη m.

Έχουμε a2 + c3 = (a2 + a + c3 )2 + c3 = (a2 + c3 ) (a2 + 2a + ι + c3 ) n + l η η 11 π π •

Θα αποδείξουμε ότι οι αριθμοί a� + c3 και a� + 2a. + ι + c3 είναι πρώτοι μεταξύ τους. Πρώτα θα αποδεί­

ξουμε με επαγωγή ότι ο 4c3 + ι είναι πρώτος με τον αριθμό 2a. + ι , για κάθε η ::=:: ι . Για η = I , έστω p ένας κοινός πρώτος διαιρέτης των αριθμών 4c3 + I και 2a. + I = 2c + ι . Τότε προ-

ΕΥΚΛΕΙΔΗΣ Β' 69 τ. l/20

--------- Μαθηματικοί Διαγωνισμοί - Μαθηματικές Ολυμπιάδες --------­

κύπτει ότι ο p διαιρεί και τον αριθμό 2 ( 4c3 + l ) = ( 2c + 1 ) ( 4c2 - 2c + l ) + 1 , οπότε ο p διαιρεί το 1 , που είναι άτοπο. Υποθέτουμε τώρα ότι ( 4c3 + Ι , 2an + l ) = l , για κάποιο η 2 1 . Θα

αποδείξουμε ότι ( 4c3 + 1 , 2an + ι + l ) = 1 . Πράγματι, αν p είναι ένας κοινός πρώτος διαιρέτης των αριθμών

4c3 + l και 2an+ ι + l , τότε ο p διαιρεί και τον αριθμό 2 ( an + ι + l ) = (2an + 1 )2 + 4c3 + l , που είναι άτοπο. Ας υποθέσουμε τώρα ότι για κάποιο η 2 Ι ο αριθμός

a2 + c3 = (a2 + a + c3 )2 + c3 = (a2 + c3 ) (a 2 + 2a + l + c3 ) 11 + Ι π η n η η

είναι μία δύναμη . Επειδή οι a � + c3 και a ;, + 2an + l + c3 είναι πρώτοι μεταξύ τους, έπεται ότι και ο

a � + c3 είναι επίσης μία δύναμη . Ομοίως συνεχίζουμε αν υποθέσουμε ότι ο αριθμός

a ; + C3 = C2 + C3 = C2 ( C + I ) είναι μία δύναμη . Αν a2 ( a + 1 ) = t m , με m 2 3 , τότε a = t � και a + l = t; , το οποίο είναι αδύνατο. Επομέ­

νως έχουμε a 2 ( a + 1 ) = t2 , οπότε λαμβάνουμε τις λύσεις a = s2 - 1 , s 2 2, s ε Ν.

.... -- . 0ΙΕΘΝΗΣ ΜΑΘΗΜΑΤΙΚΗ OnYMΠIAOA Μαδρίτη, Ισπανία, Ιούλιος 10-22, 2008

Η 49η Διεθνής Μαθηματική Ολυμπιάδα έγινε φέτος στη Μαδρίτη της Ισπανίας από l Ο έως 22 Ιουλίου 2008 . Συμμετείχαν συνολικά 99 χώρες. Η Ελληνική ομάδα συμμετείχε με τους μαθητές:

Χάλκινο μετάλιο · ' Τ' ' · • · · , 1 Χάλκινο μετάλιο

. . , Εύφημη μνεία • 1 , • • Εύφημη μνεία

, Εύφημη μνεία :. , Εύφημη μνεία.

Συνοδοί της Ελληνικής ομάδας ήταν ο Αναπληρωτής Καθηγητής του Ε.Μ.Π. Ανάργυρος Φελλούρης (αρχηγός) και ο Καθηγητής του Πανεπιστημίου Ιωαννίνων Θεόδωρος Μπόλης (υπαρχηγός) .

Έστω οξυγώνιο τρίγωνο ABC με ορθόκεντρο το σημείο Η. Ο κύκλος που περνάει από το Η και έχει κέντρο το μέσον της πλευράς BC τέμνει την ευθεία BC στα σημεία Α ι και Α2• Ομοίως, ο κύκλος που περνάει από το Η και έχει κέντρο το μέσον της πλευράς CA τέμνει την ευ­θεία CA στα σημεία Βι και Β2 , και ο κύκλος που περνάει από το Η και έχει κέντρο το μέσον της πλευράς ΑΒ τέμνει την ευθεία ΑΒ στα σημεία Cι και C2• Να αποδείξετε ότι τα σημεία Α ι , Α2, Β ι , Β2, Cι, C2 είναι ομοκυκλικά.

Οι μεσοκάθετοι των ευθυγράμμων τμημάτων Α 1 Α υ Ββz . C 1 C2 είναι επίσης μεσοκάθετοι των πλευρών BC, CA, ΑΒ , οπότε τέμνονται στο περίκεντρο Ο του τριγώνου . Επομένως το σημείο Ο είναι το μοναδικό σημείο που μπορεί να είναι το κέντρο του ζητουμένου κύκλου, αν βέβαια αυτός υπάρχει.

Έστω Α0 ,Β0 και C0 τα μέσα των πλευρών BC,CA και ΑΒ , αντίστοιχα. Τότε από το ορθογώ-νιο τρίγωνο ΟΑ0Α1 έχουμε ΟΑ ; = ΟΑ� + Α0Α; = ΟΑ� + Α0Η2

• ( 1 ) Έστω Κ το μέσον του ΑΗ και έστω L το μέσον του CH . Επειδή τα σημεία Α0 και 80 είναι τα

μέσα των πλευρών BC και CA , έπεται ότι A0L 1 1 ΒΗ και B0L 1 1 ΑΗ . Έτσι τα ευθύγραμμα τμήματα A0L και B0L είναι κάθετα προς τις AC και BC , και άρα παράλληλα προς τα 080 και ΟΑ0 , αντίστοι­

χα. Επομένως το τετράπλευρο OA0LB0 είναι παραλληλόγραμμο, οπότε τα ευθύγραμμα τμήματα ΟΑ0 και B0L είναι παράλληλα και ίσα. Επίσης , το ευθύγραμμο τμήμα B0L που έχει άκρα τα μέσα 80 και

ΕΥΚΛΕΙΔΗΣ Β' 69 τ. Ι /2 1

--------- Μαθηματικοί Διαγωνισμοί - Μαθη ματικές Ολυμπιάδες

L δύο πλευρών του τριγώνου AHC είναι ίση και παράλληλη προς τα ΑΚ και ΚΗ .

Β c Σχήμα 1

Άρα τα τετράπλευρα ΑΚΑ00 και ΗΑ00Κ είναι παραλληλόγραμμα. Από το πρώτο από αυτά έπεται ότι Α0Κ = ΟΑ = R, όπου R η ακτίνα του περιγεγραμμένου κύκλου του τριγώνου ABC . Από το δεύτερο και τον κανόνα του παραλληλογράμμου έπεται ότι

2 (0Α� + Α0Η2 ) = 0Η

2 + Α0Κ

2 = 0Η

2 + R

2• (2)

ο ? ?

Από τις ( Ι ) και (2) λαμβάνουμε ΟΑ� = Η- + R-

. Λόγω συμμετρίας, το ίδιο προκύπτει και για τα 2

τμήματα ΟΑ2 ' ΟΒι , ΟΒ2 , οcι και oc2 . Έτσι τα σημεία Α ι , Αυ 8ι , Β2 , Cι , C2 βρίσκονται όλα σε κύκλο

κέντρου Ο και ακτίνας ( ΟΗ2 + R

2 )j2 .

Θα αποδείξουμε πάλιν ότι το σημείο Ο ισαπέχει από τα σημεία Α ι , Α2 , 8 ι , Β υ Cι και C2 . Όπως και

πριν ονομάζουμε Α0 ,80 και C0 τα μέσα των πλευρών 8C,CA και Α8 , αντίστοιχα. Επιπλέον, ο­νομάζουμε ωa , ωb και ωc τους κύκλους που περνάνε από το σημείο Η και έχουν κέντρα τα σημεία

Α0 ,Β0 και C0 , αντίστοιχα. Έστω Α' το δεύτερο σημείο τομής των κύκλων ωb και ωc (σχήμα 2) . Η

διακεντρική ευθεία 80C0 των δύο αυτών κύκλων, η οποία περνάει από τα μέσα δύο πλευρών του τριγώ­νου A8C , είναι παράλληλη προς την 8C και κάθετη προς το ύψος ΑΗ. Τα σημεία Α0 και Η είναι συμμετρικά ως προς την διακεντρική ευθεία 80C0 , οπότε το σημείο Α' βρίσκεται πάνω στην ευθεία ΑΗ . Από τους δύο κύκλους ωb και ωc λαμβάνουμε ΑCι · AC2 = ΑΑ' · ΑΗ = Α8ι · ΑΒ2 .

Άρα το τετράπλευρο Β ι Β2CιC2 είναι εγγράψιμο . Οι μεσοκάθετες των πλευρών 8ιΒ2 και C ιC2 τέμνο­νται στο Ο , οπότε το Ο είναι το περίκεντρο του εγγράψιμου τετραπλεύρου 8ι 82CιC2 και ΟΒι = 082 = οcι = oc2 . Ομοίως λαμβάνουμε ΟΑι = ΟΑ2 = ΟΒ ι = 082 και Ο Αι = ΟΑ2 = οcι = oc2 , οπότε τα σημεία Α ι , Α2 , 8 ι , 8υ Cι , C2 ανήκουν στον ίδιο κύκλο κέντρου Ο .

� .: ; "- • , , I . Το πρόβλημα μπορεί να λυθεί χωρίς μεγάλη δυσκολία υπολογιστικά, με χρήση τριγωνομετρίας, Ανα­

λυτικής Γεωμετρίας ή μιγαδικών αριθμών. Στη συνέχεια θα δώσουμε μία λύση που στηρίζεται στο δια­νυσματικό λογισμό.

Έστω Ο το περίκεντρο και R η ακτίνα του περιγεγραμμένου κύκλου του τριγώνου ABC. Θεωρού-με τα διανύσματα ΟΑ = a, ΟΒ = b, OC = c, όπου a

2 = b

2 = c

2 = R

2.

Είναι γνωστό ότι ΟΗ = a + b + c , οπότε θα έχουμε

Α Η = ΟΗ - ΟΑ = (a + b + c ) -b + c

= 2a + b + c

ο ο 2 2

και ΟΑ� = ΟΑ� + Α0Α� = ΟΑ� + Α0Η2

= ( b; c )2 + ( 2a +ξ+ c )2 ΕΥΚΛΕΙΔΗΣ Β ' 69 τ. t /22

--------- Μαθη ματικοί Διαγωνισμοί - Μαθηματικές Ολυμπιάδες

Σχήμα 2 = � (b2 + 2b o c + c2 ) + � ( 4a2 + 4a ο b + 4a o c + b2 + 2b o c + c2 )= 2R2 + (a ο b + a o c + b o c ) o

Ανάλογοι τύποι προκύπτουν και για τα τμήματα ΟΑ2 , ΟΒ1 , ΟΒ2 , OC1 και OC2 ο

2 0 ( υ ) Να αποδείξετε ότι ισχύει χ2 y2 z 2

---=- + + > 1 (χ - 1)2 (y - 1)2 (z - 1)2 - ' (*)

για όλους τους πραγματικούς αριθμούς χ, y, z, που είναι διαφορετικοί από το 1 και ικανοποιούν την ισότητα xyz = 1 .

(β) Ν α αποδείξετε ότι στη σχέση (*) η ισότητα ισχύει για άπειρο πλήθος τριάδων ρητών αριθ­μών χ, y, z, που είναι διαφορετικοί από το 1 και ικανοποιούν την ισότητα xyz = 1 .

( ) -- • • χ Υ b z

α .=.εκιναμε με την αντικατασταση -- = a, -- = , -- = c , χ - ι y - ι z - ι

από την οποία προκύπτει ότι a b c χ = --, y = -- , z = -- 0

a - ι b - ι c - ι Τότε η ανισότητα (*) είναι ισοδύναμη με την a 2 + b2 + c2 � ι , ( ι )

ενώ οι νέες μεταβλητές υπόκεινται στους περιορισμούς a , b, c 1= ι και ( a - ι ) ( b - ι ) ( c - ι ) = abc ο

Η τελευταία είναι ισοδύναμη με την a + b + c - ι = ab + bc + ca , 2 (a + b + c - ι ) = (a + b + c )2 - (a2 + b2 + c2 ) , a 2 + b2 + c2 - 2 = (a + b + c )2 - 2 (a + b + c ) ,

? ? ? ( )? a - + b- + c- - 1 = a + b + c - ι - , από την οποία προκύπτει η ζητούμενη ανισότητα a 2 + b2 + c2 � ι ο

Από την εξίσωση a 2 + b2 + c2 - 1 = ( a + b + c - 1 )2 , προκύπτει ότι η σχέση ( 1 ) αληθεύει ως ισότη-

τα, αν, και μόνον αν, τα δύο αθροίσματα a 2 + b2 + c2 και a + b + c έχουν τιμή 1 , δηλαδή όταν ισχύουν { a + b + c = 1 } { a + b + c = I } ( a + b + c γ - 2 ( ab + bc + ca ) = 1 <=> ab + bc + ca = Ο ο

a , b, c 1= Ι a , b, c 1= Ι

Με απαλοιφή του c λαμβάνουμε την εξίσωση a 2 + ab + b2 = a + b , η οποία είναι δευτεροβάθμια ως προς b και γράφεται στη μορφή

b2 + (a - l ) b + a (a - 1 ) = 0 ο (2) Η εξίσωση (2) έχει διακρίνουσα Δ = ( a - 1 )2 - 4a ( a - 1 ) = ( 1 - a ) ( Ι + 3a ) ο

Για την εύρεση ρητών τριάδων ( a, b, c ) , αρκεί να θεωρήσουμε το a ρητό και καθέναν από τους παράγο-

ΕΥΚΛΕΙΔΗΣ Β ' 69 τ. Ι /23

--------- Μαθηματικοί Διαγωνισμοί - Μαθηματικές Ολυμπιάδες

ντες 1 - a και Ι + 3a τέλια τετράγωνα ρητών. Για παράδειγμα, αν θεωρήσουμε a = �, όπου m

k, m Ε Ζ, m * Ο , τότε (m - k ) (m + 3k ) Δ = ( 1 - a ) ( 1 + 3a ) = 2 . ,

m οπότε αρκεί καθένας από τους παράγοντες m - k και m + 3k να είναι τετράγωνο ακεραίου . Αυτό μπορεί να επιτευχθεί, για παράδειγμα, αν πάρουμε m = k2 - k + 1 , k Ε Ζ , αφού τότε

?

( k2 - 1 ) -m - k = (k - 1 )2 και m + 3k = (k + 1( 'Ετσι, για k E Z,m = k2 - k + 1, a = � , τότε είναι Δ = 2 m m

m - k ± (k2 + 1 ) και η δευτεροβάθμια εξίσωση (2) έχει τις ρίζες b = . Θεωρώντας

2m m - k + k

2- 1 m + (m - 2 ) m - 1 b = =

προκύπτει ότι

2m 2m 1 - k c = 1 - a - b = -- . m

m

Η συνθήκη a, b, c "f:. 1 οδηγεί στην εξαίρεση των τιμών k = Ο και k = 1 . Έτσι, αν k Ε Ζ, k > 1 , λαμ-

β ' ' ' ' δ ( b ) ( k k2 - k 1 - k ) ανουμε απειρια ρητων τρια ων a, , c =

k2 _ k + 1, k 2 _ k + 1

, k2 _ k + 1 ·

Επιστρέφοντας στις αρχικές μεταβλητές λαμβάνουμε απειρία ρητών τριάδων

( x , y, z) = ( k ο , k - k2 , k � 1 J

(k - 1 ) - k

που ικανοποιούν τη ζητούμενη ισότητα.

Το σύστημα των εξισώσεων a 2 + b2 + c2 = 1 , a + b + c = 1 , με a, b, c * 1 , παριστά στον τρισδιάστατο

χώρο ως προς καρτεσιανό σύστημα συντεταγμένων Oabc, κύκλο με κέντρο την αρχή Ο (Ο, Ο, Ο) με ε­

ξαίρεση τριών σημείων. Έτσι το δεύτερο ερώτημα είναι ισοδύναμο με την εύρεση άπειρου πλήθους ρη­τών σημείων πάνω στον προηγούμενο κύκλο.

Ε , , , a b c , θ , , ' λλ πισης, η αρχικη αντικατασταση χ = --, y = --, z = -- , μπορει να αντικαταστα ει απο α ες

ανάλογες αντικαταστάσεις, όπως 1 1 1 " χ = 1 - � , y = 1 - β , z = 1 - γ ,

c<> χ = χ ' - 1 , y = y' - 1, z = z' - 1 ,

a - 1 b - 1 c - 1

"' 1 - yz = u, 1 - zx = ν, 1 - xy = w ,

οι οποίες ανάγουν την προς απόδειξη ανισότητα σε μία ανισότητα της μορφής Α 2 :?: Ο . Ανάλογα με τη μέθοδο που θα ακολουθήσει κάποιος, μπορεί να πάρει διάφορες ακολουθίες τριάδων

ρητών ( x , y, z ) . Μία τέτοια ακολουθία είναι η ( χ , y, z ) = ο , 2 ,-- , r Ε QJ - { - 1 , 1 } . ( 2r - 2 2r + 2 r2 - 1 : ( r + 1γ ( r - 1 ) 4

-�\ι-�·-Jτεμη λ·(J t τ�-� � ( υ ) Θεωρώντας z = -1- και απαλείφοντας παρανομαστές η ζητούμενη ανισότητα γίνεται

xy ( xy - 1 )2 ( x z ( y - 1 )2 + y2 ( χ - 1 )2 ) + ( χ - 1 )2 :?: ( χ - 1 )2 ( y - 1 )2 ( xy - 1 )2 .

Αν θέσουμε p = χ + y, q = xy , τότε μετά από αρκετές πράξεις λαμβάνουμε

q4 - 6q3 + 2pq2 + 9q2 - 6pq + p2 :?: 0� (q2 - 3q + p)2 :?: 0, που ισχύει.

ΕΥΚΛΕΙΔΗΣ Β' 69 τ. l/24

--------- Μαθηματικοί Διαγωνισμοί - Μαθηματικές Ολυμπιάδες --------­

Για να ισχύει η ισότητα, αρκεί q2 - 3q + p = Ο . Από τον ορισμό τους οι p και q είναι ρίζες της

δευτεροβάθμιας εξίσωσης t 2 - pt + q = Ο , ως προς t , της οποίας η διακρίνουσα γίνεται Δ = p2 - 4q = ( 3q - q 2 ) 2 - 4q = q ( q - 1 / ( q - 4) .

Έτσι, αρκεί να είναι και οι δύο παράγοντες q και q - 4 τετράγωνα ρητών αριθμών, αφού τότε

p = 3q - q2 Ε Q, JΔ Ε Q και ρητές επίσης προκύπτουν οι ρίζες της εξίσωσης t 2 - pt + q = Ο . Θέτοντας

q = ( :)2 = 4 + ( �)2 , η , m, f Ε Ζ , τότε λαμβάνουμε την εξίσωση ( 2m )2 + e2 = η 2 , η οποία είναι η γνω­

στή Πυθαγόρεια εξίσωση που, όπως είναι γνωστό έχει άπειρες ακέραιες λύσεις.

Jl � · Να αποδείξετε ότι υπάρχουν άπειροι θετικοί ακέραιοι αριθμοί n τέτοιοι ώστε ο

αριθμός n2 + 1 να έχει ένα πρώτο διαιρέτη, που είναι μεγαλύτερος από το 2n + ili . Έστω Ν άρτιος θετικός ακέραιος και έστω p ένας πρώτος διαιρέτης του Ν2 + 1 που είναι περιττός.

Το διάστημα [- Ρ� 1 , Ρ� 1 ] περιέχει δύο ακέραιους, η0 και -η0 , ο ι οποίοι είναι ισοϋπόλοιποι με το Ν

και το -Ν mod ulo p , αντίστοιχα. Θέτοντας η = Ι η 0 I , βρίσκουμε ένα μη αρνητικό ακέραιο η για τον ο-

ποίο ισχύει ότι η2 + 1 = (±Ν)2 + 1 = Ο ( mod p) .

Είναι η ;t: Ο , αφού η 2 + 1 2: p > 1 , οπότε θα είναι: 1 .::; η .::; Ρ - Ι . 2

Θέτουμε χ = p - 2η. Από τη σχέση χ 2 = (p - 2η )2 = 4η 2 = -4 ( mod p) , συμπεραίνουμε ότι ο p διαιρεί το χ 2 + 4 , οπότε χ 2 + 4 2: p. Τότε

( ( p - 2η )2 + 4 ) - p = ( x 2 + 4) - p 2: 0 , p2 - (4η + 1 ) p + 4η2 + 4 2: 0

Επειδή ισχύει ότι η .::; Ρ - Ι , έπεται ότι p - 2η -..!. > Ο . Αν υποθέσουμε ότι η > 8 , τότε θα είναι και 2 2

2η - _!2 > Ο . Τότε από την ( 1 ) προκύπτει ότι: p - 2η -..!. 2: J2η - 1 5 . 4 2 4

Επιπλέον, από η > 8 έπεται ότι ili > 4 , οπότε θα έχουμε

p 2: 2η + ..!. + J2η - Ι 5 > 2η + ..!. + /'2_η ___ J2;;_2_η_+ ___ 1 = 2η + ili .

2 4 2 ν 4

( 1 )

Για τη συμπλήρωση της λύσης, αρκεί να αποδείξουμε ότι η παραπάνω διαδικασία, με κατάλληλη επιλογή του Ν , δίνει άπειρο πλήθος διαφορετικών τιμών του η που ικανοποιούν το ζητούμενο του προβλήματος.

Θέτουμε Ν = k ! , όπου k 2: 6 1 . Επειδή ο αριθμός Ν2 + 1 = ( k !γ + Ι είναι σχετικά πρώτος με τους α­

ριθμούς 1 , 2, . . . , k, έχουμε ότι p > k 2: 6 1 , οπότε είναι p 2: 67 και η > 8 . Επειδή είναι p > k , ο p μπορεί

να πάρει άπειρο πλήθος διαφορετικών τιμών και επειδή ο η 2 + Ι δεν μπορεί να έχει περισσότερους από έναν πρώτους διαιρέτες μεγαλύτερους του η, από διαφορετικές τιμές του p προκύπτουν διαφορετικές τι-μές του η .

Γ1J 1 . Το ζητούμενο του προβλήματος μπορεί ελαφρώς να βελτιωθεί. Συγκεκριμένα, θα αποδείξουμε ότι ο όρος ili μπορεί να αντικατασταθεί από τον όρο Mn .

Έστω p πρώτος τέτοιος ώστε p = Ι ( mod 8) . Η ισοτιμία χ 2 = - Ι ( mod p) έχει δύο λύσεις στο διάστη­

μα ( 1 , p - 1 ] των οποίων το άθροισμα είναι p . Αν η είναι η μικρότερη από αυτές τις δύο λύσεις, τότε ο

ΕΥΚΛΕΙΔΗΣ Β ' 69 τ. l/25

--------- Μαθη ματικοί Διαγωνισμοί - Μαθηματικές Ολυ μπιάδες

ρ διαιρεί το η 2 + Ι και ισχύει η :::; Ρ - Ι . Θα αποδείξουμε ότι ρ > 2η + Mn . 2

Πράγματι, αν η = Ρ - Ι - €, όπου € � Ο , τότε από την ισοτιμία η 2 = - Ι ( mod ρ) έπεται ότι 2

( Ρ� Ι - €) 2 = - I ( mod ρ) ή ( 2€ + Ι ) 2 + 4 = Ο ( mod ρ) .

Έτσι έχουμε ( 2 e + I )2 + 4 = rp, για κάποιο r � Ο . Αφού ( 2€ + Ι )2 = Ι = ρ ( mod 8 ) , έχουμε ότι

r = 5 ( mod 8 ) , οπότε r � 5. Άρα είναι ( 2 e + Ι )2 + 4 � 5p, οπότε € � � . Αν θέσουμε �5p - 4 = u , 2

' � u - I ' το τε r_ � --, οποτε 2

ρ - 1 Ι η = - - e :s; -(p - u ) . 2 2

Σ δ ' ' u 2 + 4 ' ' ' 5 1 0 4 Ο Λ ' ' ε συν υασμο με τη σχεση ρ = -- , προκυπτει οτι u- - u - η + � _ υνοντας την ανισοτητα 5

, 0 λ β , , 5 + ν' 40η + 9 , , δ , , αυτη ως προς u � αμ ανουμε οτι u � , απο την οποια σε συν υασμο με τη σχεση 2

ρ � 2η + u � 2η + k(5 + ν'40η + 9 ) > 2η + Mn-p - u η :::; -- , οδηγεί στην 2

Επειδή υπάρχουν άπειροι πρώτοι της μορφής 8k + I , έπεται ότι επίσης υπάρχουν άπειροι η με τη ζη­τούμενη ιδιότητα.

Ν

Π αραη) ρηση 2 . Θεωρώντας την παραγοντοποίηση του γινομένου Π ( η2 + Ι ) σε γινόμενο πρώτων n= l

παραγόντων, μπορεί κάποιος να παρατηρήσει ότι ο μεγαλύτερος πρώτος διαιρέτης του είναι τουλάχιστον Ν log Ν . Αυτό οδηγεί σε καλύτερο φράγμα της μορφής ρ > η log η. Η απόδειξη χρησιμοποιεί στοιχεία από την κατανομή των πρώτων αριθμών της μορφής 4k + I .

Είναι επίσης γνωστό ότι το φράγμα ρ > η log η μπορεί ακόμα να βελτιωθεί στη μορφή ρ > Cηα , για κάποιο α ε { I , 2} και κάποια θετική σταθερά C .

Π ρόβλη μα 4. Βρείτε όλες τις συναρτήσεις f : (Ο,+οο) � (Ο,+οο) (δηλαδή, η f είναι συνάρτηση από το σύνολο των θετικών πραγματικών αριθμών στο σύνολο των θετικών πραγματικών αριθμών) για τις οποίες ισχύει:

( f(w) ) 2 + (f(x) ) 2 f(y2 ) + f(z2 )

για όλους τους θετικούς πραγματικούς αριθμούς w, χ, y, z, που ικανοποιούν την ισότητα wx = yz . \ \ι ση Έστω f μία συνάρτηση που ικανοποιεί τη δεδομένη συνθήκη . Θέτοντας w = χ = y = z = I λαμβά-

νουμε ( f ( I ) )2 = 1 , οπότε f ( I ) = I . Στη συνέχεια θέτουμε w = υ , χ = Ι , y = z = Fu, όπου υ > Ο , οπότε

προκύπτει η ισότητα

η οποία δίνει

Έτσι,

( f (υ ) )2 + 1 υ2 + Ι -'----'---'-'--:- = 2f (υ) 2υ '

υ (f ( υ) )2 + υ = υ2f (υ) + f (υ) , (υf (υ) - Ι ) (f (υ) - υ) = Ο .

για κάθε υ > Ο ισχύει f (υ) = υ ή f (υ) = _!_ . υ

Κατ' αρχήν εύκολα επαληθεύουμε ότι οι συναρτήσεις Ι f (υ) = υ, υ > Ο ή f (υ) = - , υ > Ο , υ

( Ι )

(2)

ικανοποιούν τη δεδομένη συνθήκη , οπότε αποτελούν λύσεις του προβλήματος. Θα αποδείξουμε ότι οι

ΕΥΚΛΕΙΔΗΣ Β' 69 τ. l/26

--------- Μαθη ματικοί Διαγωνισμοί - Μαθηματικές Ολυμπιάδες --------­

συναρτήσεις αυτές είναι και οι μοναδικές λύσεις. Α ν υποθέσουμε ότι υπάρχει συνάρτηση f που ικανοποιεί το πρόβλημα, διαφορετική από τις δύο συ-

ναρτήσεις της ( Ι ), τότε για κάποια a, b > Ο θα ισχύουν f ( a ) =t- a και f ( b) =t- _!_ . Λόγω της ( Ι ) θα πρέπει b

f ( a ) = _!_ και f ( b) = b . Θέτοντας τώρα στη δεδομένη συνθήκη w = a , χ = b, y = z = Jab λαμβάνουμε a

Ι ο b ( Ι b2 ) --o + b" 2 2 a ---z + a- - a + b , f ( ab ) - a

(3) 2f ( ab ) -

2ab η -

a 2 + b2

Όμως, λόγω της ( 1 ), πρέπει f ( ab ) = ab ή f ( ab ) = -1 , οπότε έχουμε: ab

Αν f ( ab ) = ab , τότε από την (3) προκύπτει a-2 + b2 = a 2 + b2 , οπότε είναι a = 1 , το οποίο αντιφάσκει προς τη σχέση f ( a ) =t- a .

Αν f ( ab ) = __!__ , τότε από την (3) προκύπτει ότι a2 b2 ( a-2 + b2 ) = a2 + b2 , ab

οπότε είναι b = 1 , το οποίο αντιφάσκει προς τη σχέση f (b) =t- _!_ . b

Άρα οι μοναδικές λύσεις του προβλήματος είναι οι συναρτήσεις (2). Η δεδομένη συνθήκη έχει τέσσερις μεταβλητές που υπόκεινται μόνο στη συνθήκη

wx = yz και έτσι αφήνει τρεις βαθμούς ελευθερίας και δίνει πλήθος πληροφοριών. Υπάρχουν πολλές α-ντικαταστάσεις με τις οποίες μπορεί να μελετήσει κάποιος τη συνάρτηση που ζητάμε. Παρακάτω δίνουμε μία, που οδηγεί πάλι στη σχέση ( I ) .

Βρίσκουμε πάλιν την ισότητα f ( 1 ) = Ι και θέτουμε w = χ = 1, y = Fu, z =},; , υ > Ο,

οπότε προκύπτει η σχέση f (υ) + f ( �) = υ + � . ( 4)

Θέτοντας επίσης w = υ, χ = _!_ , y = z = 1 , υ > Ο , λαμβάνουμε υ

(5)

Λύνουμε το σύστημα των (4) και (5) υψώνοντας στο τετράγωνο τα δύο μέλη της (4) και αφαιρώντας τη προκύπτουσα εξίσωση από την (5) , οπότε προκύπτει η εξίσωση

2f (υ) f (�) = 2 . (6)

Με αφαίρεση της (6) από την (5) λαμβάνουμε (f (υ ) - f (�))2 = (υ -�)2 ή f (υ ) - f (�) = ± (υ -�) ,

οι οποίες μαζί με την (4) οδηγούν στις συναρτήσεις (2) . Έστω η και k θετικοί ακέραιοι με k � η και k - η άρτιος. Δίνονται 2η λαμπτήρες

αριθμημένοι με τους αριθμούς 1 , 2, . . . , 2η, ο καθένας από τους οποίους μπορεί να είναι στην κα­τάσταση αναμμένος ή στην κατάσταση σβηστός. Αρχικά όλοι οι λαμπτήρες είναι σβηστοί. Θεωρού­με ακολουθίες βημάτων, στις οποίες σε κάθε βήμα ένας μόνο από τους λαμπτήρες αλλάζει κατάστα­ση (από αναμμένος σε σβηστός ή από σβηστός σε αναμμένος).

Έστω Ν ο αριθμός εκείνων των ακολουθιών που αποτελούνται από k βήματα και έχουν ως α­ποτέλεσμα την κατάσταση κατά την οποία οι λαμπτήρες με αριθμό από 1 μέχρι η είναι όλοι αναμ­μένοι και οι λαμπτήρες από η + 1 μέχρι 2η είναι όλοι σβηστοί.

Έστω Μ ο αριθμός εκείνων των ακολουθιών που αποτελούνται από k βήματα και έχουν ως α­ποτέλεσμα την κατάσταση κατά την οποία οι λαμπτήρες με αριθμό από 1 μέχρι η είναι όλοι αναμ­μένοι και οι λαμπτήρες με αριθμό από η + 1 μέχρι 2η είναι όλοι σβηστοί, αλλά κανένας από τους

ΕΥΚΛΕΙΔΗΣ Β' 69 τ. l/27

--------- Μαθηματικοί Διαγωνισμοί - Μαθηματικές Ολυμπιάδες

λαμπτήρες με αριθμό από η + 1 μέχρι 2n ποτέ δεν βρέθηκε στη κατάσταση αναμμένος. Να προσδιορίσετε το λόγο Ν/Μ.

Ονομάζουμε επιτρεπτή διαδικασία μία ακολουθία k βημάτων (άναμμα-σβήσιμο) η οποία οδηγεί στη κατάσταση να είναι αναμμένοι οι λαμπτήρες με αριθμό 1 , 2, . . . , η και σβηστοί οι λαμπτήρες με αριθμό η + 1, . . . , 2η . Αν η ακολουθία των k βημάτων δεν μεταβάλλει σε καμία φάση την κατάσταση κανενός από τους λαμπτήρες η + 1, . . . , 2η , τότε αυτή ονομάζεται περιορισμένη. Άρα, σύμφωνα με το πρόβλημα, υπάρχουν Ν επιτρεπτές διαδικασίες και μεταξύ αυτών υπάρχουν Μ περιορισμένες.

Σε κάθε επιτρεπτή διαδικασία, περιορισμένη ή όχι, καθένας από τους λαμπτήρες 1 , 2, . . . , η αλλάζει κα­τάσταση (από σβηστός σε αναμμένος), οπότε επιδρούμε σε αυτόν περιττό αριθμό φορών, ενώ καθένας από τους λαμπτήρες η + 1 , . . . , 2η δεν αλλάζει τελικά κατάσταση (από σβηστός παραμένει σβηστός), οπό-τε επιδρούμε σε αυτόν άρτιο αριθμό φορών. Σημειώνουμε ότι Μ > Ο , δηλαδή υπάρχει μία τουλάχιστον περιορισμένη επιτρεπτή διαδικασία. Πράγματι, αρκεί να ανάψουμε μία φορά καθένα από τους λαμπτήρες 1 , 2 , . . . , η και στη συνέχεια να πάρουμε τυχαία έναν από αυτούς και να του αλλάξουμε την κατάσταση k - η φορές ( από την υπόθεση ο αριθμός k - η είναι άρτιος.

Θεωρούμε μία τυχαία περιορισμένη διαδικασία p . Παίρνουμε τον λαμπτήρα .e, 1 � .e � η και υποθέ-τουμε ότι έχουμε επιδράσει σε αυτόν k 1 φορές, όπου ο αριθμός k 1 πρέπει να είναι περιττός. Επιλέγουμε τυχαία ένα άρτιο αριθμό από αυτά τα k 1 βήματα και aντικαθιστούμε αυτά με αντίστοιχα βήματα στον λαμπτήρα η + .e . Αυτό μπορεί να γίνει με 2k , - ι τρόπους (γιατί ένα σύνολο με k 1 στοιχεία έχει συνολικά 2k , - ι υποσύνολα με άρτιο αριθμό στοιχείων). Σημειώνουμε ότι k1 + k2 + . . . + k" = k .

Οι παραπάνω δράσεις, για τα διάφορα e ε { 1, 2, . . . , η } είναι ανεξάρτητες, υπό την έννοια ότι η δράση στον λαμπτήρα .e δεν επηρεάζει τη δράση σε κάποιον από τους υπόλοιπους λαμπτήρες. Έτσι συνολικά υπάρχουν 2k , - ι · 2k, - ι · · · 2k, - ι = 2k-n τρόποι για να συνδυάσουμε τις παραπάνω δράσεις. Καθένας από αυ­τούς τους συνδυασμούς οδηγεί σε τελική κατάσταση ίδια με αυτήν της αρχικής διαδικασίας p . Αυτό α-ποδεικνύει ότι καθεμία από τις περιορισμένες επιτρεπτές διαδικασίες μπορεί να τροποποιηθεί κατά 2k-n τρόπους, δίνοντας έτσι 2k-n επιτρεπτές διαδικασίες. Στη συνέχεια θα αποδείξουμε ότι κάθε επιτρεπτή διαδικασία q μπορεί να προέλθει με τον τρόπο που περιγράψαμε προηγουμένως. Πράγματι, αρκεί να α-ντικαταστήσουμε κάθε αλλαγή κατάστασης ενός λαμπτήρα με αριθμό fl > η η οποία συμβαίνει στη δια­δικασία q με μία αλλαγή κατάστασης του αντίστοιχου λαμπτήρα με αριθμό fl - η . Στην προκύπτουσα διαδικασία p οι λαμπτήρες η + 1 , . . . , 2η δεν αλλάζουν ποτέ κατάσταση . Οι αλλαγές κατάστασης σε κάθε λαμπτήρα με αριθμό fl > η συνέβαινε στη διαδικασία q άρτιο αριθμό φορών. Έτσι οι αντικαταστάσεις που περιγράψαμε προηγουμένως αλλάζουν την κατάσταση κάθε λαμπτήρα με αριθμό fl � η επίσης ένα άρτιο αριθμό φορών, οπότε η τελική κατάσταση του συνόλου των λαμπτήρων δεν αλλάζει. Αυτό σημαί­νει ότι η προκύπτουσα διαδικασία είναι επιτρεπτή και περιορισμένη . Αν τώρα θεωρήσουμε τη διαδικασία p και αντιστρέψουμε όλες αυτές τις αντικαταστάσεις, τότε προκύπτει η διαδικασία q . Έτσι, υπάρχει μία αντιστοιχία 1 - 2k-n μεταξύ του συνόλου των Μ περιορισμένων επιτρεπτών διαδικασιών και του συνό-

λου των Ν επιτρεπτών διαδικασιών, οπότε � = 2k-n . Μ

Θεωρούμε μία τυχαία διαδικασία k βημάτων τέτοια ώστε στο τέλος της οι πρώτοι η λαμπτήρες εί­ναι αναμμένοι και οι τελευταίοι η λαμπτήρες είναι σβηστοί. Προφανώς, ο λαμπτήρας με αριθμό i , (i = 1 , 2, . . . , η) έχει αλλάξει κατάσταση περιττό αριθμό φορών, έστω 2a; + 1 , ενώ κάθε λαμπτήρας με α-ριθμό η + i, έχει αλλάξει κατάσταση άρτιο αριθμό φορών, έστω 2b; , όπου a ; , b; είναι μη αρνητικοί ακέ-ραιοι. Σημειώνουμε Α = 2a1 + 2a2 + . . . + 2a" + η και Β = 2b1 + 2b2 + 2b" , οπότε θα είναι Α + Β = k . Για σταθερές τιμές των a ; , b; , ο αριθμός των επιτρεπτών διαδικασιών ισούται

k ! με

ΕΥΚΛΕΙΔΗΣ Β ' 69 τ. l/28

--------- Μαθη ματικοί Διαγωνισμοί - Μαθηματικές Ολυμπιάδες ---------

0 συνολικός αριθμός των επιτρεπτών διαδικασιών προκύπτει με άθροιση πάνω σε όλους τους δυνα­τούς συνδυασμούς των a ; , b; που δίνουν άθροισμα Α + Β = k :

Ν = Σ k ! ( Ι )

A+B=k Π �)2a ; + 1 ) ! ( 2b ; ) !

Σημειώνουμε ότι ο αριθμός των διαδικασιών κατά τις οποίες κανένας από τους τελευταίους η λα­μπτήρες δεν έχει αλλάξει κατάσταση σε κάποια φάση , λαμβάνεται με τον περιορισμό των τιμών b ; = 0, i = 1 , 2, . . . , η : Μ = Σ k !

(2) A=k Π�)2a; + Ι ) !

Έστω c ; = a ; + b; , οπότε 2c; = 2a ; + 2b; + I είναι ο αριθμός των φορών που αλλάζουν κατάσταση οι λαμπτήρες με αριθμό i και η + i . Θέτουμε C = 2c1 + 2c2 + . . . + 2cn + η , οπότε C = Α + Β = k . Σημειώ­νουμε ότι, για σταθερή τιμή του c; , ο b; μπορεί να πάρει όλες τις τιμές από Ο μέχρι και C; . Επιπλέον, ι­σχύει a; = C ; - b; . Έτσι η σχέση ( 1 ) μπορεί να γραφεί ξανά ως άθροισμα πάνω σε όλους τους συνδυα­σμούς των C; με C = k :

c , c, c, π I (2c + l) k ' π c, (2c + 1) Ν = Σ Σ Σ · · ·Σ k !Π

( ) 1 = Σ π

. Π Σ 1

C"=k b, =o b, =o b, =o i = ι 2c; + I ! 2b; ι·=k Π i=1( 2c; + Ι ) ! i = ι b, =o 2b;

Χρησιμοποιώντας τη γνωστή σχέση Σ = Σ = -Σ = 2 π , π (2η + l) π (2η + Ι) 1

π (2η + 1) 2

p=O 2p p=O 2p + ] 2 q=O q

λαμβάνουμε k l 11 k ' Ν = Σ π

. π 22c, = 2k

-11 Σ π .

= 2k-π Μ .

ι·=k Πi= 1 ( 2c; + Ι ) ! i= ι c:= k Π ;)2c; + Ι ) !

Σημειώνουμε με Ν (η , k ) και Μ (η , k ) τις αντίστοιχες τιμές των Ν και Μ για δεδο­μένες τιμές των Ν και Μ . Η ισότητα Ν (η , k ) = 2k

-π Μ (η , k ) μπορεί να αποδειχθεί με επαγωγή ως προς

το η . Η αρχική ισότητα για η = 1 είναι προφανής, αφού Ν ( 1 , k ) = 2k-ι και M ( l , k ) = 1 . Στη συνέχεια για

το βήμα της επαγωγής πρέπει να χρησιμοποιήσουμε τις αναδρομικές σχέσεις: k-π k-π

N (η + l , k ) = Σ 22c , .., ( k

\.1 (η , k - 2cπ+ ι - 1 ) , Μ (η + 1 , k ) = Σ 2 2c'" ' ( k \."' (η , k - 2cπ+ ι - 1 ) .

c , . , =Ο 2cπ+ Ι + 1}' c,,. , =Ο 2cπ+ Ι + 1 Jn Έστω ABCD κυρτό τετράπλευρο με IBAI :�; ΙΒCΙ . Συμβολίζουμε τους εγγεγραμμέ­

νους κύκλους των τριγώνων ABC και ADC με ω1 και ω2 , αντίστοιχα. Υποθέτουμε ότι υπάρχει κύ-

κλος ω ο οποίος εφάπτεται της προέκτασης της πλευράς ΒΑ προς το μέρος του Α και της προέ­κτασης της πλευράς BC προς το μέρος του C, ο οποίος επίσης είναι εφαπτόμενος των ευθειών AD και CD.

Να αποδείξετε ότι οι κοινές εξωτερικές εφαπτόμενες των κύκλων ω1 και ω2 τέμνονται πάνω στον κύκλο ω.

Η απόδειξη στηρίζεται σε δύο λήμματα που ακολουθούν: ; Ε. . Δίνεται κυρτό τετράπλευρο ABCD. Υποθέτουμε ότι υπάρχει κύκλος ω εγγεγραμμένος

στη γωνία ABC και εφαπτόμενος των επεκτάσεων των πλευρών AD και CD προς το μέρος του D. Τότε ισχύει: ΑΒ + AD = CB + CD .

Ο κύκλος ω εφάπτεται των ευθειών AB, BC,CD, DA , έστω στα σημεία K, L,M,N , όπως φαίνεται στο σχήμα . Τότε έχουμε AB + AD = (BK - AΚ) + (AN - DN) , CB+CD = (BL-CL) + (CM- DM) .

Επίσης, ισχύουν ΒΚ = BL, DN = DM, AK = AN, CL = CM , οπότε από τις προηγούμενες ισότητες εύκολα προκύπτει ότι: ΑΒ + AD = CB + CD .

Έστω ω1 ο εγγεγραμμένος κύκλος του τριγώνου ABC εφάπτεται της πλευράς AC

στο σημείο Ρ. Έστω ΡΡ' η διάμετρος του εγγεγραμμένου κύκλου που περνάει από το Ρ και έστω ότι η ευθεία ΒΡ' τέμνει την πλευρά AC στο Q. Τότε το σημείο Q είναι το σημείο επαφής της πλευ-

ΕΥΚΛΕΙΔΗΣ Β' 69 τ. l/29

--------- Μαθηματικοί Διαγωνισμοί - Μαθηματικές Ολυμπιάδες

ράς AC με τον αντίστοιχο παρεγγεγραμμένο κύκλο του τριγώνου ABC. Έστω ότι η εφαπτόμενη του εγγεγραμμένου κύκλου ω1 στο σημείο Ρ' τέμνει τις πλευρές

ΒΑ και BC στα σημεία Α' και C' (σχήμα 3) . Ο κύκλος ω1 είναι ο παρεγγεγραμμένος κύκλος του τρι­γώνου A'BC' ως προς την πλευρά A'C' και τέμνει την πλευρά A'C' στο σημείο Ρ' . Επει-δή είναι A'C' I I AC , η ομοιοθεσία κέντρου Β και λόγου Β%Ρ ' μεταφέρει τον κύκλο ω1

στον παρεγγεγραμμένο κύκλο του τριγώνου ABC ως προς την πλευρά AC . Έτσι το λήμμα προκύπτει από το ότι η παραπάνω ομοιοθεσία aντιστοιχίζει το σημείο Ρ' στο σημείο Q .

Θυμίζουμε ακόμη ότι, αν ο εγγεγραμμένος κύκλος ενός τριγώνου ABC τέμνει την πλευ­ρά AC στο σημείο Ρ , τότε το σημείο επαφής της ιδίας πλευράς AC με τον αντίστοιχο πα­ρεγγεγραμμένο κύκλο της είναι το μοναδικό σημείο του ευθύγραμμου τμήματος AC με την ιδιότητα ΑΡ = CQ .

Συνεχίζουμε με την κύρια απόδειξη του προβλήματος. Έστω ότι οι κύκλοι ω1 και ω2 τέμνουν την πλευρά AC στα σημεία Ρ και Q , αντίστοιχα. Τότε ισχύει

ΑΡ = AC + AB - BC

CQ = CA + CD - AD

. 2 ' 2

Σχήμα 3

Επειδή ΑΒ - BC = CD - AD , από το Λήμμα 1 έπεται ότι ΑΡ = CQ . Άρα στο τρίγωνο ABC η πλευ­ρά AC και ο αντίστοιχος παρεγγεγραμμένος κύκλος της εφάπτονται στο σημείο Q . Ομοίως, στο τρίγω­νο ADC η πλευρά AC και ο αντίστοιχος παρεγγεγραμμένος κύκλος της εφάπτονται στο σημείο Ρ . Ση­μειώνουμε ότι Ρ 7= Q , αφού ΑΒ 7= BC .

Έστω ΡΡ ' και QQ' οι διάμετροι των κύκλων ω1 και ω2 , αντίστοιχα, που είναι κάθετες προς την πλευρά AC. Τότε, από το Λήμμα 2, προκύπτει ότι τα σημεία Β, Ρ' και Q είναι συνευθειακά και το ίδιο επίσης ισχύει για τα σημεία D,Q' και Ρ .

Θεωρούμε τη διάμετρο του κύκλου ω που είναι κάθετη στην πλευρά AC και σημειώνουμε με Τ το άκρο της που απέχει μικρότερη απόσταση από την πλευρά AC . Η ομοιοθεσία με κέντρο το σημείο Β και λόγο Β'% Ρ' aντιστοιχίζει τον κύκλο ω 1 στον κύκλο ω , οπότε τα σημεία Β, Ρ' και Τ είναι συνευ­

θειακά. Ομοίως, τα σημεία D,Q' και Τ είναι συνευθειακά , αφού η ομοιοθεσία με κέντρο το σημείο D

και λόγο - DJbQ' aντιστοιχίζει τον κύκλο ω2 στον κύκλο ω .

Σημειώνουμε ακόμα ότι τα σημεία Τ, Ρ' και Q είναι συνευθειακά , όπως και τα σημεία T,Q' και Ρ . Επειδή ΡΡ' 1 1 QQ' , τα ευθύγραμμα τμήματα ΡΡ ' και QQ' είναι ομοιόθετα σε ομοιοθεσία κέντρου Τ . Το ίδιο ισχύει για τους κύκλους ω1 και ω2 , αφού αυτοί έχουν τις ΡΡ' και QQ' ως διαμέτρους. Επιπλέον, το σημείο Τ βρίσκεται στο ίδιο ημιεπίπεδο με τα σημεία Q και Q' , ως προς την ευθεία ΡΡ' , οπότε ο λόγος της ομοιοθεσίας αυτής είναι θετικός. Ειδικότερα, οι κύκλοι ω1 και ω2 δεν είναι ίσοι.

Συνοψίζοντας τα προηγούμενα έχουμε ότι το Τ είναι το κέντρο ομοιοθεσίας που aντιστοιχίζει τον κύκλο ω1 στον κύκλο ω2 • Αυτό ολοκληρώνει την απόδειξη , αφού το μοναδικό σημείο με την ιδιότητα αυτή είναι η τομή των κοινών εξωτερικών εφαπτόμενων των κύκλων ω1 και ω2 •

ΕΥΚΛΕΙΔΗΣ Β ' 69 τ. J /30

HIJMIJ MArHHEMAriCVS Η Homo Mathematicus είναι μια στήλη στο περιοδικό μας, με σκοπό την ανταλλαγή απόψεων και την ανάπτυξη προβληματισμού πάνω στα εξής θέματα: I ) τι είναι τα Μαθηματικά, 2) Πρέπει ή όχι να διδάσκονται, 3) Ποιοι είναι οι κλάδοι των Μαθηματικών και ποιο το αντικείμενο του καθενός, 4) Ποιες είναι οι εφαρμογές τους, 5) Ποιες επιστήμες ή κλάδοι επιστημών απαιτούν καλή γνώση των Μαθηματικών για να μπορέσει κάποιος να τους σπουδάσει.

Για τους σΙΙ\'ψγ6πς της σηjλη;: : παράκληση ! τα κείμενα της στήλης αυτής, ως προς το περιεχόμενό τους και ως προς το επίπεδό τους, θα πρέπει να είναι συμβιβαστά με τα ενδιαφέροντα και το επίπεδο κατανόησης από μέρους των παιδιών.

r.πιμi:λεια : Καρκάνης Βασίλης, Κφασαρ ίδης Γιάννης I. "τι ::ίναι τα ΜαΟηματιιαi;

« . . . Παρόλ ' αυτά το αίμα της επιστήμης μας για να σπρώξουν την ανάπτυξη μονόπλευρα προς ανεβαίνει από τις ρίζες της. Οι ρίζες αυτές τον ένα πόλο της ζωοδότρας aντινομίας. απλώνονται σε ατέλειωτα ριζάκια βαθιά μέσα σ ' Δεν πρέπει ν ' αποδεχθούμε την παλιά βλάσφημη αυτό που θα μπορούσαμε να ονομάσουμε ανοησία, ότι η έσχατη δικαιολόγηση της πραγματικότητα, είτε η «πραγματικότητα» αυτή μαθηματικής επιστήμης είναι η «δόξα του είναι η μηχανική, η φυσική, η βιολογική μορφή, η ανθρώπινου μυαλού». Δεν θα πρέπει να επιτρέψουμε οικονομική συμπεριφορά, η γεωδαισία, ή πάλι στα Μαθηματικά να διχαστούν και ν ' αποκλίνουν κάποια άλλη μαθηματική ουσία που βρίσκεται στη προς μια εκδοχή τους «καθαρή» και προς μιαν σφαίρα του γνωστού. Η αφαίρεση και η γενίκευση «εφαρμοσμένη». Θα πρέπει να παραμείνουν και να δεν είναι για τα Μαθηματικά πιο ζωτικές από τη ενισχυθούν σαν ένα ενιαίο ζωτικό ρεύμα μες στο μοναδικότητα των φαινομένων και, πριν απ ' όλα, πλατύ ποτάμι της επιστήμης και θα πρέπει να τα από την επαγωγική διαίσθηση. Μόνο η εμποδίσουμε να γίνουν ένα μικρό παραποταμάκι που αλληλεπίδραση των δυνάμεων αυτών και η σύνθεσή ενδέχεται να το ρουφήξει η άμμος . . . » τους μπορούν να κρατήσουν τα Μαθηματικά [πηγή :Από την απάντηση του Richard Kurand στις απόψεις

ζωντανά και να τα εμποδίσουν ν' αποστεγνωθούν του Μ. Η. Stone, στο: George F. Caπier «Appl ied Mathematics:What i s Needed in Research and Education»,

και να καταντήσουν νεκρός σκελετός. Θα πρέπει να SIAM Reνiew, 4, 1 962, 297-320] πολεμήσουμε εναντίον των aποπειρών που γίνονται

Γιάννης Κφασαρίδη ς 1 1 . "Α υ η) το ξ::ραη:; , [η απαντήσεις στο τέλος της στήλης]

α. τι είναι η κυβική παραβολή και που χρησιμοποιείται; β. Τι είναι τα "σώματα του Αρχιμήδη ";

1 1 1 . "οι σΙJ ιη:ριάη:r:: τικ στιίJ.ιιr:: γράφο υ ι •-ιρωτο ι) ν ,

Πμ (ύτο Οψια: ΜαΟημο. τιιιά ιcαι Πχι'η (δ::ι5η:ρ η συνι:Χι:ια)

1/ρο/.::γiψι:ι,α. Συνεπείς στην υπόσχεσή μας, σας παρουσιάζουμε, σ' αυτό το τεύχος, το δεύτερο μέρος της ενδιαφέρουσας εργασίας του Αποστόλη Παπανικολάου.

«Τέχνη και Μ αθη ματικ{ι στο Μ ουσ}:ίο Η ρακλειδών» , Π απανικολάου Απόστολος Μ έρος Β '

Συνεχίζουμε την παράλληλη πορεία στην ιστορία των Μαθηματικών και της Τέχνης με τους Πυθαγόρειους. Η μουσική , τα ωραία ακούσματα βασίζονται στη διαδοχή ήχων με συγκεκριμένο λόγο συχνοτήτων (μουσικά διαστήματα) . Οι Πυθαγόρειοι ανακάλυψαν ότι συνηχούν αρμονικά δύο χορδές όταν έχουν λόγους μηκών αντίστοιχα 2 : I , 3 :2 , 4 : 3 και 9 : 8 . (Σήμερα αυτό έχει επιβεβαιωθεί από τη Φυσική με την ανάλυση Fourier και τις έρευνες του Helmholtz. Όταν δυο νότες συνηχούν, η αρμονία που παράγεται οφείλεται στην σύμπτωση των αρμονικών τους συνιστωσών και αυτό γίνεται μόνο, όταν ο λόγος συχνοτήτων τους είναι λόγος μικρών φυσικών αριθμών) .

Η οκτάβα π.χ. (δια πασών όπως την έλεγαν οι Πυθαγόρειοι) είναι το μουσικό διάστημα που προκύπτει από το λόγο 2 : 1 . Αν κρουσθεί μια χορδή και ξανακρουσθεί αφού δεσμευθεί η μισή χορδή οι δύο ήχοι που ακούγονται είναι πανομοιότυποι αλλά σε διαφορετικό ύψος. Το επόμενο διάστημα είναι το 3 :2 κατόπιν το 4 :3 έπειτα το 9 : 8 κοκ.

Πεπεισμένοι, ότι όλα είναι αναλογίες φυσικών αριθμών προσπάθησαν να βρουν το λόγο της διαγωνίου δ προς την πλευρά α ενός τετραγώνου αλλά και το λόγο της πλευράς-διαγωνίου ενός κανονικού πενταγώνου που μάλιστα το είχαν και έμβλημά τους. Δηλαδή προσπάθησαν να βρουν ένα τμήμα μ (κοινό μέτρο) ώστε δ = κ·μ και α = λ·μ. (οπότε ο λόγος δ/α θα ήταν ίσος με κ/λ) .

ΕΥΚΛΕΙΔΗΣ Β ' 69 τ. 1 /3 1

-------------- ΗΟΜΟ MArHtMAriCVS Οι ιστορικοί μιλάνε για μια από τις μεγάλες

στιγμές των Μαθηματικών και ταυτόχρονα για την πρώτη μεγάλη κρίση στα θεμέλιά τους. Οι Πυθαγόρειοι έκαναν πρώτοι τη διαπίστωση ότι τα τμήματα δ και α είναι ασύμμετρα. Δεν είναι δυνατόν να βρεθεί ένα κοινό μέτρο όσο και αν μικραίνει το μ και έτσι οι λόγοι τους δεν μπορούν να γραφούν ως κλάσματα φυσικών , είναι άρρητοι. Οι Πυθαγόρειοι έτσι ήρθαν αντιμέτωποι με το άπειρο, την επ' άπειρο χωρίς αποτέλεσμα ελάττωση του μ. (Με σημερινή ορολογία τα άπειρα δεκαδικά ψηφία χωρίς καμία περιοδικότητα που προσεγγίζουν έναν άρρητο αριθμό ) .Κατασκεύασαν όμως ένα σύστημα «ανθυφαιρετικών γνωμόνων» με το οποίο διαπίστωσαν την περιοδικότητα του (με σημερινή ορολογία) συνεχούς κλάσματος που αναπαριστά το ρίζα 2, και επινόησαν τους πλευρικούς-διαμετρικούς αριθμούς. (Με σημερινούς όρους δύο ισοσυγκλίνουσες ακολουθίες στο ρίζα 2 η μια με έλλειψη και η άλλη με υπερβολή) . Κατόρθωσαν έτσι να καθυποτάξουν αυτήν την απειρία και να φθάσουν στο θαυμαστό επίτευγμα να μπορούν να προσεγγίσουν οσονδήποτε το λόγο διαγώνιος/πλευρά ενός τετραγώνου (το σημερινό ρίζα 2) .

Ένας από τους επηρεασμούς της Τέχνης από αυτές τις αναζητήσεις των Πυθαγορείων είναι ότι σε αυτούς πιστώνεται ιστορικά η πρώτη μελέτη του λόγου που αναφέρθηκε προηγουμένως, αυτού της διαγωνίου προς την πλευρά ενός κανονικού πενταγώνου, ο περίφημος αριθμός Φ, η χρυσή τομή . Η πιο αρμονική διαίρεση ενός τμήματος σε δύο άνισα τμήματα. Οι Αρχαίοι Έλληνες δεν τον ονόμαζαν ούτε Φ ούτε χρυσή τομή , τον ονόμαζαν διαίρεση σε μέσο και άκρο λόγο όπως τουλάχιστον εμφανίζεται αργότερα στα «Στοιχεία του

Ποια είναι η σχέση όλων αυτών με την τέχνη; Μα η αναντίρρητη πεποίθηση ότι κάθε μέρος ενός οικοδομήματος, μέσα και έξω, ενός γλυπτού, ενός ζωγραφικού πίνακα ή μιας μουσικής σύνθεσης θα πρέπει να διέπεται από ένα ενιαίο σύστημα μαθηματικών αναλογιών και ότι το επίπεδο τελειότητας έχει σχέση με τον πλούτο του θεωρητικού υποβάθρου δηλαδή της Γεωμετρίας και κυρίως της μελέτης των αναλογιών. Με το άγαλμα «Δορυφόρος» ο Πολύκλειτος, γλύπτης σύγχρονος του Φειδία, δημιούργησε ένα σύστημα ιδανικών αναλογιών, τον «κανόνα», για τη δημιουργία ενός ανθρώπινου γλυπτού, κανόνα που εξακολούθησε να χρησιμοποιείται στη Δύση μέχρι την ύστερη Αναγέννηση . Τα θαυμαστά έργα του χρυσού αιώνα στην Αθήνα αλλά και την υπόλοιπη Ελλάδα προκαλούν ακόμα και σήμερα το θαυμασμό αλλά

Ευκλείδη» . Οι Ευρωπαίοι τον ονόμασαν έτσι όταν μετά την Αναγέννηση , έκπληκτοι διαπίστωσαν τη γνώση και χρήση του από τους Έλληνες στην τέχνη . Τον έλεγαν θεία αναλογία (diνina proportione) και έχουμε πάρα πολλές μαρτυρίες για τη χρήση του στην Αναγέννηση (Luca Paccioli , Daνinci κ .α .) . Χρυσή τομή τον ονόμασε ο Martin Ohm Γερμανός μαθηματικός, αδερφός του γνωστού φυσικού από τη μονάδα μέτρησης της ηλεκτρικής αντίστασης περίπου το 1 835 . και τον ονόμασε Φ ο Mark Barr Αμερικανός μαθηματικός στις αρχές του 20ου αιώνα προς τιμή του Φειδία από το αρχικό του γράμμα και έκτοτε είναι γνωστός έτσι ως «number phi» ή «golden ratio».

Ο μεγάλος Kepler έλεγε ότι δύο είναι τα διαμάντια της Γεωμετρίας το ένα είναι το Πυθαγόρειο θεώρημα και το άλλο αυτή η διαίρεση σε μέσο και άκρο λόγο.

Υπάρχει μεγάλη συζήτηση σε επίπεδο Πανεπιστημιακών ερευνητών σήμερα για τον υπερτονισμό της αισθητικής αξίας της χρυσής τομής , αυτό όμως δεν μειώνει καθόλου την αξία της Πυθαγόρειας αναζήτησης για τις αναλογίες. Γιατί κατόπιν τη σκυτάλη πήραν οι Μαθηματικοί της Ακαδημίας του Πλάτωνος , ο Θεαίτητος και ο μεγάλος Εύδοξος. Ο Εύδοξος και αργότερα ο Αρχιμήδης έθεσαν τις βάσεις για το σύστημα των πραγματικών αριθμών που σήμερα βασίζεται πάνω του όλος ο aπειροστικός λογισμός. Πιο συγκεκριμένα ο Εύδοξος στον οποίο αποδίδεται ολόκληρο το 5° βιβλίο των στοιχείων του Ευκλείδη έδωσε μια επέκταση της έννοιας της αναλογίας, έναν ορισμό που ουσιαστικά ισοδυναμεί με τις τομές Dedekind.

και την εξονυχιστική μελέτη των αναλογιών τους. Ο Πλάτων εξ άλλου περιγράφει στο διάλογο

«τίμαιος» τον κόσμο σαν μια σύνθεση γεωμετρικών αρμονικών σωμάτων, των περίφημων 5 Πλατωνικών Στερεών που θα περάσουν και στην Αναγέννηση και θα αποτελέσουν τον προβληματισμό πολλών από τα φωτισμένα πνεύματά της. (π.χ. Kepler) .

Εδώ λοιπόν είναι η μεγάλη συνεισφορά των αρχαίων Ελλήνων μαθηματικών στην ανθρωπότητα αλλά και στην τέχνη . Η διερεύνηση της έννοιας της αναλογίας, του Μαθηματικού λόγου, η σύνδεση της Αισθητικής με τα Μαθηματικά. Στο επόμενο τεύχος θα συνεχίσουμε με την αλληλεπίδραση Τέχνης και Μαθηματικών στην Αναγέννηση

ΕΥΚΛΕΙΔΗΣ Β ' 69 τ. l/32

-------------- HIJMIJ MA7ΉEMAτtCUJ' /Jι ι:ι ! τι.ρ ο · Ίr'Jr<.ρJψιίι:i:rς ι. ω ιVι!ηαiiιαω[ (i(Ι iOμoi

Στις αρχές Απρίλη, στο ταχυδρομικό κουτί του ενός των υπευθύνων της στήλης (Γ.Κ.) έφτασε ένα δέμα που μέσα είχε ένα βιβλίο 206 σελίδων και τίτλο «ΣΥΜΒΟΛΉ ΣΤΗ ΜΕΛΕΤΗ ΤΩΝ ΥΠΕΡΔΟΜΩΝ ΜΕ ΕΦΑΡΜΟΓΕΣ ΣΤΗΝ ΥΠΟΧΡΕΩτΙΚΉ ΕΚΠΑΙΔΕΥΣΗ)) . Το βιβλίο αυτό ήταν η διδακτορική διατριβή του Νίκου Ανταμπούφη . Ο συγγραφέας του είναι ένας σημαντικός φίλος της στήλης μας και καταξιωμένος μαθηματικός στην περιοχή της Θράκης (Ξάνθη) . Αντιλαμβάνεται κανείς πως ένα τέτοιο θέμα δεν μπορούσε να μας αφήσει αδιάφορους, για το λόγο αυτό τον παρακαλέσαμε να μας φτιάξει ένα μικρό κείμενο στο οποίο να δίνει στα παιδιά της Γ Λυκείου να καταλάβουν την έννοια των «Υπερπράξεων)) . Εγχείρημα αρκετά δύσκολο να δώσεις, μέσα σε περιορισμό 500 λέξεων, να γίνει εύκολα κατανοητό (η στενότητα του χώρου της στήλης δεν επιτρέπει πολλά) . Εμείς πιστεύουμε πως ο συγγραφέας τα κατάφερε. Το κείμενο αυτό θα το δημοσιεύσουμε σε δύο συνέχειες.

Στα παρακάτω θα δώσουμε τον ορισμό της υπερπράξης και στη συνέχεια ένα παράδειγμα από το σύνολο των μιγαδικών αριθμών που είναι το ευρύτερο και πληρέστερο σύνολο με το οποίο έρχονται σε επαφή οι μαθητές του Λυκείου . Στη δωδεκαετή μαθηματική του εκπαίδευση κάθε μαθητής και μαθήτρια έρχεται σε επαφή με μια ποικιλία πράξεων μεταξύ αριθμών, γεωμετρικών σχημάτων και αλγεβρικών εννοιών. Κάθε πράξη μεταξύ δύο στοιχείων χ, y είναι μια απεικόνιση του ζεύγους (x,y) σε ένα στοιχείο ω που ονομάζεται αποτέλεσμα της πράξης. Για παράδειγμα, το ζεύγος των αριθμών (2, 3) απεικονίζεται με την πρόσθεση στον αριθμό 5 δηλαδή 2+3 = 5 . Τα χ, y, ω ανήκουν στο ίδιο

σύνολο (γνωστές πράξεις στα σύνολα !R!. και <C) ή σε διαφορετικά σύνολα (πολλαπλασιασμός αριθμού με γωνία ή διάνυσμα, εσωτερικό γινόμενο διανυσμάτων).

Το κοινό γνώρισμα των παραπάνω πράξεων είναι το μονότιμο αποτέλεσμα. Δηλαδή όλες οι πράξεις ανάμεσα σε δύο στοιχεία παράγουν ένα (ως προς το πλήθος των τιμών) και μόνο ένα αποτέλεσμα, ανεξάρτητα από το είδος τους. Ο Γάλλος μαθηματικός Frederic Marty το 1 934 εισάγει[4] την έννοια της Υπερπράξης διευρύνοντας την προηγούμενη θεώρηση .

Υπερπράξη μεταξύ δύο στοιχείων χ, y ενός

συνόλου Η είναι μια απεικόνιση κάθε ζεύγους (χ, y) σε ένα υποσύνολο του Η (εκτός από το κενό σύνολο) .

Δηλαδή το αποτέλεσμα μπορεί να είναι πλειονότιμο. Για παράδειγμα, το ζεύγος των αριθμών (2,3) μπορεί να απεικονίζεται με κάποιου είδους ' π � · 1·J σ0 ι:ωJ ' στο υπεράθροισμα { 5 , 6 } δηλαδή

2Ef>3 = {5 , 6 }

(το υπεράθροισμα περιέχει το γνωστό άθροισμα και τον επόμενο φυσικό) .

Ο ορισμός του Marty αποτελεί μια ανακάλυψη η οποία φανερώνει τους βαθμούς πολυπλοκότητας της υπερπράξης. Επειδή το αποτέλεσμα μιας πράξης είναι ένα μόνο στοιχείο τότε η συνεχής επανάληψή της οδηγεί πάντα σε πράξη μεταξύ δύο στοιχείων. Για παράδειγμα,

((2+3)+4)+5 = (5+4)+5 = 9+5 = 1 4. Αντίθετα, η υπερπράξη οδηγεί σε ' άθροισμα'

στοιχείων ή ' άθροισμα' στοιχείου με σύνολο ή ' άθροισμα' συνόλων. Για παράδειγμα,

2Ef>3 = {5 , 6 } , (2Ef>3)Ef>4 = {5 , 6 }ΕΘ 4 = {9 , 1 0, 1 1 } , ((2Ef>3)EB 4 )Ef>(2Ef>3) = {9 , 1 0, 1 1 }ΕΘ { 5 , 6 } =

= { 1 4, 1 5 , 1 6, 1 7 , 1 8 } .

[· / �:l. 'ι .'·. ��· ' :'. l : J :ς :;,:ι: ·;.' :Ι/_ {ρ :[) � }i � :ς • Ανταμπούφης, Ν . : Συμβολή στη μελέτη των Υπερδομών με εφαρμογές στηv Υποχρεωτική Εκπαίδευση, Διδακτορική

Διατριβή, Δημοκρίτειο Πανεπιστήμιο Θράκης, (2008) . • Corsini, Ρ. : Prolegomena of Hypergroup Theory, Aνiani Editore. ,( Ι 993) . • Corsini, Ρ., Leoreanu, V. : Applίcations of Hyperstructure Theory, Kluwer Academic Publishers, (2002). • Marty, F., Sur une generalίsatίon de !α notίon de groupe, 811' Congres Math . Scandinaνes, Stockholm, ( 1 93 4) ,

45-49 . • Vougiouklis, Th. : Hyperstructures and theίr Representatίons, Hadronic Press . , ( Ι 994) .

'/ '•!! (>το fu) . «Το τραίνο μας νοιώθουμε, από σημεία όπου οι σιδηροτροχιές έχουν τρέzει πάνω σε ευθύγραμμο τμήμα της σιδηροδρομικής σχετική καμπυλότητα. Η ομαλή κίνηση του τραίνου στις ;ραμμή;. ενώ που και που περνάει, χωρίς να το καμπυλότητες της σιδηροδρομικής γραμμής οφείλεται

ΕΥΚΛΕΙΔΗΣ Β' 69 τ. l/33

-------------- HIJMIJ MArHEMAr/CUS στο ότι το πέρασμα από την ευθεία στην καμπύλη γίνεται στις σιδηροδρομικές γραμμές όχι τυχαία, αλλά με βάση μια καμπύλη, που σχηματίζεται από πολύπλοκους συνδυασμούς καμπυλών.

Μετά το ευθύγραμμο τμήμα οι σιδηροτροχιές συνεχίζουν με τη λεγόμενη μεταβατική καμπύλη (όπου η καμπυλότητα αυξάνει βαθμιαία) και μόνο αργότερα περνάει η καμπύλη αυτή σε τόξο κυκλικής περιφέρειας. Το αντίστροφο γίνεται στο τέλος της στροφής. Σαν

μεταβατικές χρησιμοποιούνται διάφορες καμπύλες γραμμές (ανάλογα με την καμπυλότητα της στροφής, την ταχύτητα που προβλέπεται στο σημείο αυτό για το τραίνο κ.λ.π.) . Συνήθως χρησιμοποιείται το τόξο της κυβικής παραβολής» (καμπύλη με εξίσωση y=χ3)(άλλες καμπύλες είναι: το τόξο της καμπύλης Μπερvουγί, το τόξο της σπείρας του Κορvύ)

[πηγή : 'ΆΚΑΔΗΜΑΪΚΗ", τόμ.2, εκδ . Γιαννίκος, σελ. 299]

η απl!.1'τηση στιι ι.;ι} (:ιτΙifια (β.! : Τα ήξερε και ο τετράγωνα - εξάγωνα), 4. κόλουρος κύβος (έδρες: Αρχιμήδης, γι' αυτό λέγονται και "σώματα του τρίγωνα - οκτάγωνα), 5. ρομβο-κυβο-οκτάεδρο (έδρες: Αρχιμήδη". Οι έδρες τους είναι κανονικά πολύγωνα με τρίγωνα - τετράγωνα), 6. κόλουρο κυβο-οκτάεδρο διαφορετικό αριθμό πλευρών, ενώ όλες οι στερεέ γωνίες (έδρες: τετράγωνα - εξάγωνα - οκτάγωνα), 7. κόλουρος είναι μεταξύ τους ίσες. Γνωστά σαν στα Μαθηματικά κύβος (έδρες: τρίγωνα - τετράγωνα) , 8. κόλουρο "η μ ικυνι;ν ικ6 1ωλ1Ί : ;(i ι ια" . εικοσάεδρο (έδρες : πεντάγωνα - εξάγωνα), 9. κολοβό

Υπάρχουν συνολικά δεκατρία τελείως ορισμένα δωδεκάεδρο (έδρες: τρίγωνα - πεντάγωνα), I Ο. είκοσι­ημικανονικά πολύεδρα κι ακόμη δύο άπειρες σειρές από δωδεκάεδρο (έδρες: τρίγωνα - πεντάγωνα), 1 1 . κόλουρο τα λεγόμενα πρίσματα και αντιπρίσματα του Αρχιμήδη . δωδεκάεδρο (έδρες : τρίγωνα - δεκάγωνα) , 12 . ρομβο­Τα δεκατρία αυτά στερεά είναι: 1. κόλουρο τετράεδρο είκοσι-δωδεκάεδρο (έδρες: τρίγωνα - πεντάγωνα -(έδρες : τρίγωνα - εξάγωνα), 2. κυβο-οκτάεδρο (έδρες : τετράγωνα), 13. κόλουρο είκοσι-δωδεκάεδρο (έδρες: τρίγωνα - τετράγωνα), 3. κόλουρο οκτάεδρο (έδρες: τετράγωνα - εξάγωνα - δεκάγωνα )

Παρακάτω σας δίνουμε τις εικόνες αυτών των στερεών:

/

4

�-- --;7' ..-r------;;:" -.,

t r � [ I b _ / .c.__::_ _,

- / � -1'

(� ι\ I. \ -� ι

Α

(r---,_ - _,__

1 1 1�

[πηγή : 'ΆΚΑΔΗΜΑΪΚΗ",τόμ.2, εκδ. Γιαννίκος, σ. 302. Τα σχήματα είναι από την ίδια έκδοση]

ΕΥΚΛΕΙΔΗΣ Β' 69 τ. Ι/34

�·•ιιιι•"�••• [Υοω tJ[[J[!) !iJ' tJcfJ{J[[J tJ(j)fJJ iJfJJωe5rJ(f)fJJ

Οι πραγματικοί αριθμοί Αντώνης Κυριακόπουλος - Θανάσης Μαλαφέκας

Οι παρακάτω ασκήσεις αναφέρονται στο πρώτο κεφάλαιο: «Οι πραγματικοί αριθμοί», της Άλγεβρας Α 'Ενιαίου Λυκείου .Από τη θεωρία, θα υπενθυμίσουμε τις ταυτότητες, τις ιδιότητες των ανισοτήτων και τις απόλυτες τιμές. Προσπαθήσαμε οι ασκήσεις που παραθέτουμε να μην είναι πολύ δύσκολες αλλά . . . έξυπνες!

ΤΛ ΥΊΌΊΉ ΊΈl:

Με α, β, γ ε IR και ν Ε Ν* , ισχύουν: 1 . (α + β)2 = α2 + 2αβ +β2 2 . (α - β)2 = α2 - 2αβ +β2 3 . (α - β)(α + β) = α2 - β2 4. (α + β + γ)2 = α2 + β2 + γ2 + 2αβ +2βγ + 2γα 5 . (α + β)3 = α3 + 3α2β + 3αβ2 +β3

6 . (α - β)3 = α3 - 3α2β + 3αβ2 - β3 7 . α3 - β3 = (α - β) (α2 + αβ +β2) 8 . α3 + β3 = (α + β) (α2 - αβ +β2) 9. αν _ βν = (α _ β) (αν- Ι + αν-2 β + . . . + αβν-2 + βν- 1 )

Ι Δ Ι ΟΤ Η Π\ 1: Α Ν I Σ(Η Η ΤΩ Ν

Με α, β, γ , δ , λ Ε IR ισχύουν: l . (α > β και β > γ) => α > γ 2 . α > β <=> α + γ > β + γ

3 . {α > β => α + γ > β + δ γ > δ {α>β<=>λα>λβ {α> β <=>λα< λβ

4. λ>Ο : α β και λ < Ο : α β α>β<=>- >- α> β <=>- <-λ λ λ λ {α > β > Ο 5 . γ > δ > ο => αγ > βδ

Π ΡΟΣΟΧ Η :

Από τις ανισότητες: {α > β γ > δ , δεν έπεται ανα-

γκαία ότι: α - γ > β - δ, ούτε (υποθέτοντας ακόμα

γδ :;t Ο) ότι: α > � . γ δ

Με α, β Ε IR ορίζουμε : α � β<::::> (α > β ή α=β)

ορ.

α � β<::::> (α < β ή α=β) ορ.

Για παράδειγμα, η σχέση 5 Ξ:: 3 είναι αληθής, η σχέση 5 Ξ:: 5 είναι αληθής ενώ η σχέση 5 Ξ:: 7 είναι ψευδής. Α Π ΟΛ Υ Τ Η τΙ Μ Η Π ΡΑ ΓΜ Α τ Ι Κ Ο Υ Λ Ρ Ι Θ !\ ΙΟΥ

Ορ ισμός. Με α Ε IR ορίζουμε: {α, αν α>Ο Ιαl = Ο, αν α=Ο

-α, αν α<Ο • Ά μεσες συνέπr.ιες :

Ιαl = α <=> α � Ο, Ιαl = -α <=> α � Ο, Ιαl = Ο <=> α = Ο. Ι δ ιότητr.ς ι . ια ι � ο

2 . 1-αΙ = Ια Ι 3 . ΙαΙ � α και ΙαΙ � -α 4. Ιαβl = lα l l β l

lα ι αz . . . αν l = !α ι ! lαz Ι . . . Ιαv l 5 . Ιαν i = Ιαlν (ν Ε Ν* )

6 . Ι; Ι = Ί;ι1 (β * ο) 7 . l x l � α <=> -α � χ � α

l x l < α <=> -α < χ < α

ΕΥΚΛΕΙΔΗΣ Β ' 69 τ.l /35

Μαθηματικά για την Α ' Λυκείου

8 . l χΙ �α�(χ�α ή χ�---{1) , l� >α�(χ >α ή χ<-α) 9 . Ια+� � ΙαJ + I� (με το " = " αν, και μόνο αν αβ �Ο)

Ι α-� � ΙαJ + I� (με το " = " αν, και μόνο αν αβ � Ο)

lα ι + α2 + . . . + αν I � lα ι l + Ι α2 Ι + . . . + Ι αν l Ι Ο . l lαl - l βl l � Ια + � (με το " = "

αν, και μόνο αν αβ � Ο)

l lαl - lβl l � Ια -βl (με το " = " αν, και μόνο αν αβ � Ο)

Συνοπτικά: l α l - lβ l � l l α l - lβ l l � Ι α ± βl � l αl + I β I

Ι . Έστω ότι: β = α - 2. Να βρείτε την αριθμη­

τική τιμή της παράστασης:

Κ = -2 [-3(α - 2β) + 2 (2β - α) - 2β] + 3(α + β).

Έχουμε: Κ = 6(α - 2β) - 4(2β - α) + 4β + 3(α + β) =

= 6α - Ι 2β - 8β + 4α + 4β + 3α + 3 β = =Ι 3α- 1 3 β= 1 3α- Ι 3(α-2)= 1 3α-Ι 3α+26 = 26.

β-r γ-� 2 'Ε , � Ι αβ 2 rl:f 2 ( . στω οτι α τ- και - --- =- --- Ι).

2 3 2 3

Ν α δείξετε ότι: β = γ.

Από την ( Ι ) έχουμε: αβ 2β-γ = ωι -2y-β �

2 6 2 6 � 3αβ - (2β - γ) = 3αγ - (2γ - β) � 3αβ - 2β + γ

= 3αγ - 2γ + β � 3αβ - 3β = 3αγ - 3γ �

(α - Ι )β = (α - Ι )γ � β = γ (γιατί α-Ι ;/= 0) . 3, ί) Να βρείτε τις τιμές του χ για τις οποίες

'ζ , Κ Ι 1 Ι ορι εται η παρασταση : = - - - + ---χ χ •

__ χ_

χ - 1

ii) Να απλοποιηθεί η παράσταση αυτή.

1 ) Η παράσταση Κ ορίζεται αν και μόνο αν:

x :;t: O x - I :;t: O

χ I - - :;t: O χ - Ι χ - 1 - χ --- :;t: O χ - 1

Άρα, η παράσταση Κ ορίζεται για όλες τις τιμές του χ ε IR , εκτός από τις τιμές Ο και 1 . 2) Με χ 1= Ο και χ 1= 1 , έχουμε:

Ι χ - 1 κ == ο + = -- = -χ + 1 . χ - Ι - χ - 1

χ - Ι

4 'Ε ,

β � Ο Ι 1 1

Ο , . στω οτι α γ τ- και - + - + - = . Βρειτε

α β γ

, β+γ γ+α α+β το αθροισμα: Κ = - +- +-- .

α β γ

'Εχουμε : K=f +l +I +� +� +Q = α α β β γ γ

= α (i + �) + β (� +�) + γ(� +iJ ·

Α ' ' θ ' 1 1 1 λ πο την υπο εση εχουμε : - + - = -- κτ .

β γ α 'Ετσι, έχουμε : Κ=α( -±)+β(-� J+γ( -� J=-3 .

5. Για δύο αριθμούς α και β ισχύουν: α + β =2

(1) και α2 + β2 = 2 (2). Να δείξετε ότι:

αιο + βιο = 2 (3).

Από την ( l ) έχουμε: β == 2 - α (4) Έτσι, από τη (2), έχουμε : α2 + (2 - α)2 == 2 � α2 + 4 - 4α +α2 = 2 � 2α2 - 4α + 2 == Ο � α2 - 2α + 1 = Ο �

(α - 1 )2 == Ο � α = 1 . Αντικαθιστώντας στην (4) βρίσκουμε β = 1 . Έτσι, η (3) προφανώς ισχύει. 6. Να αναλύσετε σε γινόμενο παραγόντων τις

παραστάσεις: Α = α4 + 4β4

Β = α3 + 3α2 + 3α + 2

Γ = α2(β - γ) + β2(γ - α) + γ2(α - β) Δ = (l + χ + χ2 + χ3)2 _ χ3.

Έχουμε : Α = α4 + 4β4 + 4α2β2 - 4α2β2 == (α2 + 2β2)2 -(2αβ)2 == ( α2 + 2β2 - 2αβ)( α2 + 2β2 + 2αβ). Β == ( α3 + 3α2 + 3α + Ι ) + Ι = (α + 1 )3 + 1 3 = [ (α + Ι ) + 1 ] [ (α + 1 )2 - (α + 1 ) + 1 ] ==

= (α + 2) (α2 + α + Ι ) . Γ == α\β - γ) + β2γ - β2α + /α - γ2β = α2(β - γ) + βγ (β - γ) - α (β - γ)(β + γ) = = (β - γ) (α2 + βγ - αβ - αγ) == (β - γ) [α( α - β) - γ( α - β)] == (β - γ) (α -β) (α - γ) . Δ == [( 1 + χ + χ2) + χ3]2 _ χ3 == ( l + χ + χ2)2 + 2( I + χ + χ2) χ3 + χ6 _ χ3 == = ( 1 + χ + χ2)2 + 2( 1 + χ + χ2) χ3 + χ3 (χ3 - 1 ) = == ( 1 + χ + χ2)2 + 2( 1 + χ + χ2) χ3 + χ3 (χ - 1 ) . (χ2 + χ + 1 ) = = ( l + χ + χ2) ( 1 + χ + χ2 + 2χ3 + χ4 - χ3) == ==( 1 + χ + χ2) ( l + χ + χ2 + χ3 + χ4) .

7. Με α , β Ε IR να δείξετε ότι:

(αχ + β = Ο, για κάθε χ Ε IR ) <=> α = β = Ο.

ΕΥΚΛΕΙΔΗΣ Β ' 69 τ. Ι/36

Μαθη ματικά για την Α ' Λυκείου

i) Έστω ότι αχ + β = Ο, για κάθε χ Ε IR . Τότε, με χ = Ο και χ = 1 θα έχουμε αντίστοιχα: { β = 0 {α = 0

α + β = Ο=>

β = Ο i i ) Αντιστρόφως. Έστω ότι α = β = Ο. Τότε, έ­χουμε για κάθε χ Ε IR : αχ + β = Οχ + Ο = Ο.

8. Με α Ε IR να δείξετε ότι, αν η εξίσωση : (2α - 1)χ + α + 3 = α(χ + 2)

_ 1 (I) 3 6 2

είναι αδύνατη, τότε και η εξίσωση :

α( αχ - 1) χ - α = 5( α 2 χ - 1) + 1 2 3 12

(2)

είναι αδύνατη. Το αντίστροφο αληθεύει;

Έχουμε: ( 1 ) <:::::> 2(2α - Ι )χ + α + 3 = 3 α( χ + 2) - 6

<:::::> ( 4α - 2)χ - 3αχ = -α - 3 + 6α - 6 <:=> (α - 2)χ = 5α - 9 (3)

(2) <:::::> 6α(αχ - 1 ) - 4(χ - α) = 5 (α2χ - 1 ) + 1 2 <:::::> 6α2χ - 6α - 4χ + 4α = 5α2χ - 5 + 1 2 <:::::> (α2--4)χ=2α+7<:=>(α-2)(α+2)χ=2α+7 (4) .

Έστω ότι η ( I ), δηλαδή η (3) είναι αδύνατη. Τότε α=2. Με α=2 η (4), δηλαδή η (2) είναι επίσης αδύνατη. Το αντίστροφο δεν αληθεύει, γιατί με α=---2 η (4) είναι αδύνατη, αλλά η (3) όχι.

9. Να λυθεί η εξίσωση : χ χ - 1 χ - 2 χ - 3 χ - 4 - +-- +-- +-- +-- = 5 (1 )

1 00 99 98 97 96 Έχουμε:

(� <=> (��-ι){ χ� Ι -Ι){ χ;2 -ι){ χ;3 -Ι){ χ;; -�=Ο χ - 100 χ - 100 χ - 100 χ - 100 χ - 100

<:::::> -- +-- +--+-- +-- = 100 99 98 97 % ( 1 1 1 1 1 )

<:::::> ( χ - 1 00) - + - + - + - + - = 0 1 00 99 98 97 96

<:::::> χ - 1 00 = ο <:::::> χ = 1 00 10 . Έστω ότι: α > β , γ > δ και α + β > γ + δ. Να

δείξετε ότι: α > δ.

·εχουμΕ {�:�1��:�ι:δ)=ό 2α > α + β > γ + δ > 2δ => 2α > 2δ => α > δ.

1 1 . Θεωρούμε δύο αριθμούς α,βΕ JR* . Να δείξετε

ότι ένας τουλάχιστον από τους αριθμούς: � 4β

2β ' ' ' 1 και - ειναι μικροτερος απο το . α

'Ε ' ' δ λ θ ' ' α Ι στω οτι αυτο εν α η ευει, οποτε : - 2': 4β

και 2β 2': 1 . Πολλαπλασιάζοντας αυτές τις ανισό­α

, 'λ β , α 2β I

I 1 , τητες κατα με η , ρισκουμε: - ·- 2': =>- 2': , α-4β α 2 τοπο. Άρα, ένας τουλάχιστον από τους αριθμούς αυτούς είναι μικρότερος του Ι . 12 . Για έναν αριθμό αΕ IR ισχύει: α5+α+3=0. Να

δείξετε ότι: -3 < α <0.

Έχουμε: α5 + α + 3 = Ο => α(α4 + I ) = = -3<0 => α(α4 + 1 )<0 => α<Ο (αφού α4 + 1 >0). Εξάλλου, επειδή α < Ο, έχουμε : α5 + α + 3 = Ο => α + 3 = -α5 > Ο (γιατί α5 < Ο)

=> α + 3 > Ο => α > -3 . Άρα: -3 < α <0. 13 . Καθένας από τους αριθμούς α, β και γ είναι

θετικός κα μικρότερος του 3. Να δείξετε ότι:

ί) αβ + βγ + γα > αβγ ίί) α2 + β2 + γ2 > αβγ

i) Έχουμε : I 1 - >-

0<α<3 α 3 Ι I

ι 1 Ι 0<β<3 => - >- =>-+- +- > l =>αβ+βγ+γα>αβγ. β 3 α β γ 0<γ<3 I 1 - >­γ 3

ii) Έχουμε : (α - β)2 + (β - γ)2 + (γ - α)2 2': Ο => α2 - 2αβ + β2 + β2 - 2βγ + γ2 + γ2 - 2αγ + α2 2': Ο => α2 + β2 + γ2 2': αβ + βγ + γα. Και επειδή, όπως δεί­ξαμε προηγουμένως: αβ + βγ + γα > αβγ, έπεται ότι: α2 + β2 + γ2 > αβγ. 14. Να βρείτε τους αριθμούς α Ε IR , για τους

οποίους η λύση της εξίσωσης: 3χ + 2 χ 1 (1) ' λ ' -- - χ = - + ει ναι και υση της 7 2

, 3 2χ + α ( ) ανισωσης: -(χ - α) + α � -- 2

2 4 Έχουμε : ( 1 ) <:::::> 2(3χ + 2) - 1 4χ = 7χ +

2 1 4 <:::::> - 1 5χ = 1 0 <:::::> χ = -- . 3

Για να είναι η λύση χ = -� της εξίσωσης ( I ) 3 και λύση της ανίσωσης (2) πρέπει και αρκεί να

ΕΥΚΛΕΙΔΗΣ Β ' 69 τ. l/37

Μαθη ματικά για την Α ' Λυκείου

την επαληθεύει, δηλαδή : 2(-η+α

�(---.3-α)+α< 3 <::::>-1 -�α+α::::; -4+3α 2 3 4 2 12

8 <=>-12- 1 8α+ 12α:::;;-4+3α<::>9α 2 -8 <::>α2 -- . 9

Ά ζ ' ' ' > 8 ρα, οι ητουμενες τιμες του α ει ναι: α _ - - . 9

1 5. Θεωρούμε έναν αριθμό α του διαστήματος

12 ' 3) και θέτουμε: Υ = I α I + 1 2-α I +3 1 3-α ι . Να δείξετε ότι: 4 < y :::;; 5. . ,, . Έχουμε : 2 :::;; α < 3 . Επειδή α > Ο, έχου-με: I α I = α. Επειδή 2 - α :::;; Ο, έχουμε : 1 2 - α I = α - 2 και επειδή 3 - α > Ο, έχου­με : 1 3 - α I = 3 - α. Έτσι, έχουμε : y = α + α-2+3(3 - α) = 2α - 2 + 9-3α = 7 - α . Εξάλλου, έχουμε: 2 :::;; α < 3 => - 2 2 - α > - 3 => 7 - 2 2 7 - α > 7 - 3 => 4 < y :::;; 5 .

16. Για έναν αριθμό α Ε JR ισχύει: : I α I < ε, για

κάθε αριθμό ε > Ο. Να δείξετε ότι: α = Ο. ' · Έστω ότι α;t:Ο. Τότε: I α I > Ο. Έτσι, η δο-σμένη ανισότητα οφείλει να ισχύει και για ε= I α I , οπότε θα έχουμε I α I > I α I , άτοπο. Άρα, α=Ο.

1 7. Να βρείτε τους πραγματικούς αριθμούς χ και y, για τους οποίους ισχύει:

1 0χ2 + 2y2 - 4xy + I χ - 1 1 = (χ + y)2 (1) : Έχουμε :

Ο ) <=> ι οχ2 + 2/ - 4xy + Ι χ - 1 1 = x2+2xy+/ <::::> 9χ2 + / - 6xy + I χ - Ι I = Ο

<=>(3x-y)2+ l x- Ι I = O <=> <=> {3x - y = 0 {χ = I

χ - 1 = 0 y = 3 1 8. Να δείξετε ότι για κάθε α Ε 9i ισχύει:

ι:�:: ι < 2. • iΗ ; ι 1 : Για να δείξουμε τη ζητούμενη ανισότη­

τα, αρκεί να δείξουμε ότι: 2α - I , ? 2 -2 < -,- < 2, αρκει: - 2α- - 2 < 2α - 1 < 2α α- + Ι

2 , {2α2 + 2α + I > Ο και , + , αρκει , , αρκει 2α- - 2α + 3 > Ο

{α2 + (α + 1)2 > 0 και α2 + (α - 1)2 + 2 > 0 Οι δύο αυτές ανισότητες ισχύουν.

19. Έστω ότι l 3α - β I < 2 και I α - 3β I < 2. Να

δείξετε ότι: I α - β I <1 : Προσθέτοντας κατά μέλη τις δοσμένες

ανισότητες βρίσκουμε : 4> l 3α - β I + I α - 3β l 2 l (3α - β)+( α - 3β) I =

l 4α - 4β Ι => 4 > 4 1 α - β Ι => Ι α - β Ι < 1 . � � - 1 χ-1 1-xl - x 20. Ν α λυθεί η εξίσωση : -- +- = -- (1)

(. ' " " 1 :: Έχουμε: 2 3 6

( 1 ) <=> 3( I χ 1 - 1 ) + 2(χ - I ) = I χ 1 - χ <=> 3 Ι χ 1 - 3 + 2χ - 2 - I χ Ι + χ = ο <=> 2 1 χ Ι + 3χ = 5 (2)

ί) Έστω ότι χ 2 Ο . Τότε: (2) <=> 2χ + 3χ = 5 <=> χ = I , δεκτή (γιατί χ 2 Ο) i i) Έστω ότι χ < Ο. Τότε: (2) <=> - 2χ + 3χ = 5 <=> χ = 5 , μη δεκτή (γιατί χ < 0) . Άρα, η εξί­σωση ( I ) έχει μία μόνο ρίζα, την χ= 1 .

2 1 . Να λυθεί η ανίσωση : �(21 � -1) <2)��2 (I) : Έχουμε:

( 1 ) <=> 2(2 l x i - Ι ) < 1 2 + ( 1 x l - 2) <=> 4 l x l - 2 < 1 2 + l x l - 2 <=> 3 1 χ Ι < 1 2 <=> Ι χ Ι < 4 <=> - 4 < χ < 4 .

22. Να λυθεί η ανίσωση : l l x - t l - 3 1 < 5 (1 )

ι' Ι ·· ; � � ' Έχουμε : 0 ) <=> - 5 < Ι χ - 1 Ι - 3 < 5 <=> - 2 < Ι χ - 1 Ι < 8 <=> I χ - I I < 8 <=> - 8 < χ - 1 < 8<::::> - 7 <χ < 9 .

23 . Να δείξετε ότι: .fi)s + .Jl5 = .Jl5 + 1 . ·,. . Επειδή και τα δύο μέλη είναι θετικά, αρκεί να δείξουμε ότι:

( J2)8 + M ) 2 = (Μ + 1 )2 , αρκεί:

2 (8 + Μ ) = Ι 5 + 2Μ + Ι,

αρκεί: 1 6 + 2Μ = 1 6 + 2Μ, ισχύει. 24. 'Εστω ο αριθμός: Α = { .fi - .J6}�.-2---J3=3 .

Να βρείτε τον αριθμό Α 2 και μετά τον Α.

: ί) Έχουμε : Α2 = ( J2-J6γ ( 2 -.J3) = (2 - 2m + 6) ( 2 -.J3 ) =

= ( 8 - 2ν'4.3 ) ( 2 + F3) = ( 8 - 4F3)( 2 + F3) =

= 4 ( 2 - F3 ) ( 2 + F3 ) = 4 ( 4 - 3) = 4.

i i ) 'Ε χουμε : Α2 = 4 <=> JAi = J4 <=> IΑ Ι = 2 .

ΕΥΚΛΕΙΔΗΣ Β ' 6 9 τ. Ι /38

Μαθη ματικά για την Α ' Λυκείου

Εξάλλου, επειδή J2 - J6 < Ο , έχουμε Α < Ο και άρα I Α I = - Α. Έτσι, έχουμε : I Α I = 2 <::::> - Α = 2 <::::> Α = - 2 .

25. Οι αριθμοί α και β είναι θετικοί και ισχύει:

�+�=.J5. Να δείξετε ότι: � -)! = 1. \i>ση :Εχουμ" ( � +�)

' =

5 =i+�+2 =

5 =>

� +� - 2 = 1 => [ �)' + ( J!)' -2�� = I =>

[ � -J!)' = Ι => �-J! = I

26. Ν α δείξετε ότι για κάθε χ Ε IR ισχύει:

lx - 1 1 + l2x - 1 1 � _!_ , Πότε ισχύει το ίσον; 2 \ \: π ι . Με χε ΙR έχουμε: l x-1 1 + 1 2x-1 1 =

I χ - 1 1 + 2 1 χ - ΞI � I χ - 1 1 + I χ -ΞI �

� I( χ - 1 ) - (χ -Ξ )I = 1 - 1 + ΞI = Ξ ·

27. Για τους αριθμούς α, β και γ ισχύουν:

(1 + αβ)(1 + βγ)(1 + γα) -:;:. Ο και α - β + β - γ + γ - α = Ο (1 ). 1 - αβ 1 + βγ 1 + γα

Να δείξετε ότι δύο τουλάχιστον από τους

αριθμούς αυτούς είναι ίσοι.

: Από την ( 1 ) μετά την απαλοιφή των παρονομαστών, τις πράξεις και την αναγωγή ο­μοίων όρων βρίσκουμε :

αβ2 + βγ2 + γα2 _ αγ2 _ βα2 _ γβ2 = 0 � (αβ2 _ αγ2) _ (γβ2 _ βγ2) _ (βα2 _ γα2) = 0 � α(β + γ)(β - γ) - βγ(β - γ) - α2(β - γ) = Ο �

(β - γ)( αβ + αγ - βγ - α2) = Ο � (β-γ)[β(α-γ)-α(α-γ)]=Ο�(β-γ)(α-γ)(β-α) = Ο � ( β = γ ή α = γ ή β = α) .

28. Υποθέτουμε ότι: χ + y + ω = α + β + γ (1 ) και xz + yz + ωz = αz + βz + γz (2). Να δείξετε ότι: (α-ω)(α-χ)=(y-β)(y-γ) (3). Λ ίιση : Για να ισχύει η (3) , αρκεί να δείξουμε ότι: α2 - (ω + χ)α + ωχ = / - (β + γ)y + βγ και λόγω της ( 1 ), αρκεί να δείξουμε ότι : α2-(α+β+γ-y)α+ωχ=/-(χ + y + ω - α)y + βγ, αρκεί: α2 - α2 - βα - γα + yα + ωχ = / - xy - / - ωy + αy + βγ, αρκεί: xy + yω + ωχ = αβ + βγ + γα ( 4) Από την ( 1 ) έχουμε : (χ + y + ω)2 = (α+β+γ)2 � � χ2 + y2 + ω2 + 2(xy + yω + ωχ) = =α2 + β2 + γ2 +2(αβ + βγ + γα) . Από αυτή , λόγω της (2), έπεται η (4) .

29. Με χ, y, ω Ε IR και ν Ε Ν* , να δείξετε ότι αν

η ισότητα: (χ+y+ω)2ν+ Ι=χ2ν+ Ι+y2ν+ Ι+ω2ν+ Ι (1 ) ισχύει για ν = 1 , τότε ισχύει για κάθε ν Ε Ν* . c\ ίJση : Έστω ότι η ισότητα ( 1 ) ισχύει για ν = 1 . Τότε έχουμε : (χ + y + ω)3 = χ3 + / + ω3 � [(χ + y) + ω]3 = χ3 + / + ω3 � � (χ + y)3 + 3(χ + y)2ω + 3(χ + y)ω2 + ω3 = χ3 + / + ω3 � χ3 + 3x2y + 3χ/ + / + 3(χ + y)2ω + 3(χ + y)ω2 = χ3 + / � �(x+y)(xy+xω+yω+ω2)=0�(x+y)[x(y +ω) + ω(y+ω)]=Ο�(χ + y) (y + ω) (ω + χ) = Ο � (χ = - Υ ή Υ = - ω ή ω = - χ). Έστω ότι χ = - y. Για να δείξουμε την ( I ) , αρ­κεί να δείξουμε ότι: (- y + y + ω)2ν+ ι = (- y)2v+ Ι + Υ2ν+ Ι + ω2ν+ Ι , αρκεί ω2ν+ Ι = ω2ν+ Ι , ισχύει. Όμοια εργαζόμαστε αν y = - ω ή ω = - χ .

30. Για τους θετικούς αριθμούς α, β και γ ισχύει

α2 = β2 + γ2. Να δείξετε ότι:

1) α < β + γ . 2) α3 > β3 + γ3 . : I ) Έχουμε: α2 = β2 + γ2 < β2 + γ2 + 2βγ = =(β + γ)2 � α2 < (β + γ)2 � α < β + γ. 2) Επειδή α2 = β2 + /, έχουμε α2 > β2 και

, ? , β 'Ε , α� > γ- , οποτε α > και α > γ. τσι, εχουμε : α3 = α2 . α = (β2 + γ2)α = β2 α+ γ2α > β2 β+ γ2γ = β3 + γ3 � α3 > β3 + /.

ΕΥΚΛΕΙΔΗΣ Β ' 69 τ. l /39

Μαθηματικά για την Α ' Λυκείου

_Ι · Ο _ ,

Παναγιώτης Χριστόπουλος

Για να αποδείξουμε ισότητα γωνιών ή ευθυγράμμων τμημάτων χρησιμοποιούμε τα κριτήρια ισότητας τρί­γωνων, αλλά και για να συγκρίνουμε γωνίες ή ευθύγραμμα τμήματα χρησιμοποιούμε τις ανισοτικές σχέσεις μεταξύ των στοιχείων των τριγώνων. Πολλά θεωρήματα και ασκήσεις στην Ευκλείδεια Γεωμετρία αποδει­κνύονται με αυτά τα βασικά εργαλεία. Στο συγκεκριμένο άρθρο παραθέτουμε απλές ενδεικτικές ασκήσεις εφαρμογών για τη δική σας εξοικείωση.

� ,, παίρνουμε την γωνία ζΟρ ώστε να δημιουργή-

Α ν το μέσο Μ μιας πλευράς ενός τριγώνου σου με με τις πλευρές τους δυο ίσες οξείες γωνί­

ΑΒΓ ισαπέχει από τις άλλες δυο πλευρές του. Το ες ω και φ. Παίρνουμε τα σημεία Α,Β,Μ,Ν α­

τρίγωνο είναι ισοσκελές και αντίστροφα. ντίστοιχα στις πλευρές Οχ, Οψ, Οζ, Ορ με 0-Α=ΟΒ και ΟΜ=ΟΝ.

/ ['· I ' I \ ! •

Ευ01• Έστω Μ το μέσο της πλευράς ΒΓ, ΜΕ και ΜΖ οι αποστάσεις από τις πλευρές ΑΒ και ΑΓ αντίστοιχα, με ΜΕ=ΜΖ. Συγκρίνουμε τα τρίγωνα ΒΜΕ και ΖΜΓ. Αυτά είναι ορθογώνια γωνΕ=γωνΖ, έχουν μια κάθετη πλευρά τους ίση (ΜΕ=ΜΖ υπόθεση) και τις υποτείνουσες ίσες (ΜΒ=ΜΓ υπόθεση) . Από τα κριτήρια ισότητας ορθογωνίων τριγώνων έχουμε ότι τα τρίγωνα είναι ίσα. Επομένως και οι αντίστοιχες γωνίες τους Β,Γ είναι ίσες. Άρα το τρίγωνο ΑΒΓ είναι ισοσκελές.

Με την υπόθεση ότι το ΑΒΓ εί­ναι ισοσκελές θα δείξουμε ότι ΕΜ=ΜΖ. Συγκρί­νουμε τα ορθογώνια τρίγωνα ΕΜΒ και ΖΜΓ. Αυτά έχουν ΜΒ=ΜΓ (υπόθεση) και γωνΒ=γωνΓ (υπό­θεση) . Από τα κριτήρια ισότητας ορθογωνίων τρι­γώνων έχουμε ότι τα τρίγωνα είναι ίσα. Επομένως και οι αντίστοιχες πλευρές ΕΜ,ΜΖ είναι ίσες.

Να αποδείξετε ότι: α. Τα ΑΜ, ΒΝ είναι ίσα.

β. Τα τρίγωνα ΡΑΜ,ΡΒΝ είναι ίσα.

γ. Οι ΑΝ και ΒΜ τέμνονται στο Ρ το οποίο είναι

σημείο της διχοτόμου της γωνίας χΟψ.

· ι .1

α. Τα τρίγωνα ΟΑΜ και ΟΒΝ έχουν: ΟΑ=ΟΒ (υπόθεση) ΟΜ=ΟΝ (υπόθεση) γωνΑΟΜ=ω=φ=γωνΒΟΝ (υπόθεση) σύμφωνα με το κριτήριο ισότητας τριγώνων ΠΓΠ τα τρίγωνα είναι ίσα . Άρα ΑΜ=ΒΝ. β. Τα τρίγωνα ΟΑΝ και ΟΒΜ έχουν: ΟΑ=ΟΒ (υπόθεση) ΟΜ=ΟΝ (υπόθεση) γωνΑΟΝ = γωνΑΟΜ+γωνΜΟΝ = γωνΜΟΝ+γωνΝΟΒ = γωνΜΟΒ σύμφωνα με το κριτήριο ισότητας τριγώνων ΠΓΠ

Στο εσωτερικό μιας οξείας γωνίας χΟψ τα τρίγωνα είναι ίσα. Άρα ΑΝ=ΒΜ.

ΕΥΚΛΕΙΔΗΣ Β ' 69 τ. l/40

Μ αθη ματικά για την Α · Λυκείου

Τα τρίγωνα τώρα ΑΜΝ και ΒΜΝ έχουν: >�_ σ ;κ:: nlJ rΠu -! ' ' ΑΜ=ΒΝ προηγούμενη σχέση Σε ορθογώνιο τρίγωνο ΑΒΓ (Α=ορθή) φέρ-

ΑΝ=ΒΜ προηγούμενη σχέση νουμε κάθετη στη ΒΓ στο σημείο Β. Πάνω στην

ΜΝ κοινή πλευρά κάθετη αυτή παίρνουμε σημείο Δ ώστε ΒΔ=ΑΒ.

σύμφωνα με το κριτήριο ισότητας τριγώνων ΠΓΠ Τα τρίγωνα ΑΒΓ και ΓΒΔ είναι ίσα;

τα τρίγωνα είναι ίσα. . , Τα τρίγωνα όμως ΑΜΝ και ΒΜΝ έχουν κοινό μέ-ρος το τρίγωνο ΡΜΝ άρα διαγράφοντάς το προκύ-πτει ότι τα τρίγωνα ΑΡΜ και ΒΡΝ είναι ίσα. γ. Το Ρ θα είναι στη διχοτόμο της γωνίας χΟψ αν και μόνο αν η γωνΑΟΡ=γωνΡΟΒ Συγκρίνουμε τα τρίγωνα ΑΟΡ , ΒΟΡ αυτά έχουν: ΟΑ=ΟΒ (υπόθεση) ΟΡ κοινή πλευρά ΑΡ=ΒΡ από το συμπέρασμα της προηγούμενης περίπτωσης β. Επομένως σύμφωνα με το κριτήριο ισότητας τρι­γώνων ΠΠΠ τα τρίγωνα είναι ίσα. Άρα γωνΑΟΡ=γωνΡΟΒ δηλαδή το Ρ ανήκει στη διχοτόμο Οδ της γωνίας χΟψ.

i I i ι�.-�---�-�--��--�----�·--, \ \-::::

Τα τρίγωνα είναι ορθογώνια έχουν τις κάθετες πλευρές τους ΒΔ,ΑΒ ίσες (υπόθεση), επί πλέον έχουν την ΒΓ κοινή, προσοχή όμως τα τρίγωνα δεν είναι ίσα διότι η ΒΓ στο πρώτο τρίγωνο είναι υποτείνουσα ενώ στο άλλο κάθετη πλευρά του .

_:; :.-� Σε τρίγωνο ΑΒΓ είναι Α=30° και η Βες =50° . Να δείξετε ότι τα τρίγωνα ΑΒΓ και Α'Β'Γ Να μπουν σε σειρά μεγέθους (να διαταχθούν) οι

που έχουν β=β ', γ=γ ' και μp=μp· είναι ίσα. πλευρές του και το ύψος του από την κορυφή Α.

, , \,

Α ν Μ είναι το μέσο της ΑΓ και Μ ' το μέσο της Α 'Γ ' , προεκτείνουμε τις ΑΜ και ΑΜ ' κατά ίσο μήκος και παίρνουμε τις ΜΕ=ΜΑ και Μ 'Ζ=ΜΆ' τότε τα τρίγωνα ΑΒΕ και ΑΈ 'Ζ είναι ίσα, κριτήριο ΠΠΠ, διότι γ=γ ' , ΑΕ=2μβ=2μβ ·=Α 'Ζ και ΒΕ=β=β '=Β 'Ζ ( τα τρίγωνα ΒΜΕ,ΓΜΑ είναι ίσα, έχουν ΠΓΠ, το ίδιο και τα Β 'Μ 'Ζ, Γ 'ΜΆ') . Αφού τα τρίγωνα ΑΒΕ και ΑΈ 'Ζ είναι ίσα θα εί­ναι ίσα και όλα τα λοιπά στοιχεία τους δηλαδή οι

διάμεσοι ΒΜ=Β 'Μ ' (� = �·} Επομένως τα τρί-

γωνα ΑΒΓ και Α Ή 'Γ έχουν ΠΠΠ, δηλαδή είναι ίσα.

Αφού η Β εξ =50° άρα η Β= 1 30° αμβλεία. Η γωνΓ= 1 80° -( 1 30"+30")=20" . Έτσι Γ<Α<Β επομέ­νως ισοδύναμα ΑΒ<ΒΓ<ΑΓ και το ύψος ΑΔ<ΑΒ (κάθετη με πλάγια) .

Άρα ΑΔ< ΑΒ<ΒΓ<ΑΓ.

Σε τετράπλευρο ΑΒΓ Δ φέρνουμε τη διαγώ­

νιο ΒΔ και σχηματίζονται οι εξής γωνίες:

ΒΔΑ=80° , ΔΒΑ=l0° , ΒΔΓ=95° , ΔΒΓ=45° Να

διαταχθούν οι πλευρές του και η διαγώνιος.

Οι γωνίες Α και Γ είναι: Α= 1 80°-

ΕΥΚΛΕΙΔΗΣ Β' 69 τ. l/4 1

Μαθηματικά για την Α ' Λυκείου

(80°+ 1 0°)=90°, Γ= Ι 80°-(95°+45°)=40° Στο τρίγω­νο ΑΔΒ είναι Β<Δ<Α ισοδύναμα ΑΔ<ΑΒ<ΒΔ, ενώ στο τρίγωνο Γ ΔΒ είναι Γ <Β<Δ ισοδύναμα ΒΔ<ΓΔ<ΓΒ.

Γ Τελικά ΑΔ<ΑΒ<ΒΔ<Γ Δ<ΓΒ .

Άσκηση 7'1 Σε ένα τρίγωνο με περίμετρο 45 μονάδες μήκους

οι πλευρές του α, β, γ είναι ανάλογες των αριθ­

μών 6, 4, 5. Να διαταχθούν οι γωνίες του.

ΛίJση

, α β γ α + β + γ 45 , Εχουμε - = - = - = = - = 3 δηλαδη

6 4 5 6 + 4 + 5 1 5

Διορθώσεις Τ68

� = 3 ή α= 1 8 , ή β= 12 , � = 3 ή γ= 1 5 . Επομέ­

νως β<γ<α ισοδύναμα οι γωνίες είναι Β<Γ<Α.

Π ροτεινόμενες Ασκήσεις

Ι . Ένα τρίγωνο είναι ισοσκελές αν και μόνο αν οι διάμεσοι των ίσων πλευρών του είναι ίσες.

2. Σε τρίγωνο ΑΒΓ με β>γ στην πλευρά ΑΓ παίρνουμε σημείο Δ και στην προέκταση της ΑΒ προς το Α σημείο Ε τέτοια ώστε ΑΔ=ΑΕ

= β - γ

. Ν α αποδείξετε ότι η ευθεία που διέρ-2

χεται από τα σημεία Δ και Ε διέρχεται και από το μέσο Μ της ΒΓ.

3. Έστω κυρτό τετράπλευρο ΑΒΓΔ με ΑΓ?:ΒΔ. Να δείξετε ότι 2ΑΓ>ΑΒ+ΓΔ.

4 . Δυο ευθύγραμμα τμήματα ΑΒ, Γ Δ τέμνονται στο Μ χωρίς να διχοτομούνται. Α ν τα τρίγωνα ΜΑΔ, ΜΓΒ είναι ίσα τότε τα ευθύγραμμα τμήματα ΑΒ, Γ Δ είναι ίσα.

5 . Για ποια τιμή του χ έχουμε ισοσκελές τρίγωνο

με πλευρές 20, 2, χ μονάδες μήκους;

• Στην Αι Άλγεβρα Β ' Λυκείου έπρεπε να ζητηθεί απλώς η αριθμητική τιμή της συνάρτησης Ρ(χ) για χ = ; , αντί του υπολοίπου της διαίρεσης Ρ( χ) : (χ - ;) . • Στην Α2 Γεωμετρία Β ' Λυκείου εκ παραδρομής η γωνία ΑΔΚ βρέθηκε ίση με 30°. Όμως για τη λύ-

-- 2 ση αρκεί το συν ΑΔΚ = J5 .

Αλληλογραφία : Λάβαμε το αξιόλογο και σημαντικό άρθρο του Γ. Σδρίμα, φυσικού­

ραδιοηλεκτρολόγου, με θέμα: τη φθίνουσα ταλάντωση που μελετά το βιβλίο κατεύθυνσης της Γ Λυκεί­ου . Αξιόλογες αναπτύξεις στα μέγιστα και ελάχιστα της γ(t) και ακραίες θέσεις της ταλάντωσης, όπως επίσης στα σημεία καμπής της γ(t) για τις θέσεις της μέγιστης ταχύτητας του ταλαντωτή . Υπόσχεση για μελλοντική αξιοποίηση του άρθρου.

Βιβλία που λάβαμε : Το Βιβλίο των μαθηματικών Χρήστου Γκουβιέρου και Θύμιου Διαμαντόπουλου «Μαθηματικά:

Γ Λυκείου>> Θετικής και Τεχνολογικής Κατεύθυνσης» με πολλές ασκήσεις και παραδείγματα

Τεύχος Α ' [Μιγαδικοί αριθμοί, Συναρτήσεις όριο - Συνέχεια συναρτήσεων Διαφορικός Λογισμός] .

ΕΥΚΛΕΙΔΗΣ Β' 69 τ. Ι /42

�-··[Ό811111. ,., τι,. Jr τι�Ιφ τι•ιι Αιι••Ι•ιι

Άλγεβρα Β ' Λυκείου Z)J�rωvoμ���\.�

Μενέλαος Τζιόρτζιος

Καλή σχολική χρονιά. Οι ασκήσεις που ακολουθούν είναι μια ευκαιρία επανάληψης στην διδαχθείσα ύλη του πρώτου κεφαλαίου της Άλγεβρας. Βοηθούν τους μαθητές στην καλύτερη εμπέδωση των τριγωνομετρικών τύπων καθώς και στην εφαρμογή τους με την χρήση αλγεβρικών πράξεων. Επίσης γίνεται προσπάθεια να κατανοήσουν και την χρησιμότητα των τριγωνομετρικών συναρτήσεων. Ι ) Δίνεται η συνάρτηση /(χ) = 4συν(αχ) - 2α με α>Ο. Αν η f έχει ελάχιστη τιμή το - 8, τότε:

α) Να δείξετε ότι α = 2. β) Να βρεθεί η περίοδος και η μέγιστη τιμή της συνάρτησης f. γ) Να βρεθούν τα σημεία τομής της γραφικής παράστασης της f με τον χ 'χ καθώς και οι θέσεις μεγίστου και ελαχίστου στο διάστημα [0, π) .

Λ1Jση α) Η συνάρτηση f 1 (x) = 4συν(αχ) έχει ελάχιστη τιμή ίση με -4, οπότε η ελάχιστη τιμή της f είναι ίση με -4 - 2α. Άρα - 4 - 2α = -8 => α = 2 .

β) Για α = 2 έχουμε j{x) = 4συν2χ - 4 . Ισχύει ότι η περίοδος είναι Τ = 2π = π και η μέγιστη τιμή

2 είναι fma' = 4 - 4 = ο . γ) Τα σημεία τομής με τον χ 'χ στο [0, π] είναι οι λύσεις της εξίσωσης f(x) = Ο στο [0, π] . Άρα j{x) = Ο <=> 4συν2χ - 4 = Ο <=> συν2χ = 1 <=> 2χ = 2κπ <=> χ = κπ , κ Ε Ζ. Όμως: χ ε [0, π] <=>

Ο � χ � π <=> Ο � κπ � π <=> Ο � κ � 1 . <=> κ = Ο ή κ = 1 αφού κΕΖ . Επομένως χ = Ο ή χ = π. Άρα τα σημεία τομής με τον χ 'χ είναι τα 0(0, Ο) και Α(π, 0). Η μέγιστη τιμή της είναι : fn1a, = 4· 1 -4 = Ο, που σύμφωνα με τα παραπάνω βρίσκεται για χ = Ο ή χ = π. Η ελάχιστη τιμή της είναι -8 .

Αλλά j( χ)= -8 <=> 4συν2χ -4 = -8 <=> συν2χ = -1 <=> συν2χ = συνπ <=> 2χ = 2κπ ± π <=> χ = κπ ± π , 2

π κ Ε Ζ <=> χ = - , αφού χ Ε [Ο, π] .

2

2 ) Δίνεται η συνάρτηση f(x) � Ξ[ συν ' ( χ - =) - συν ' ( χ + =)] με χ Ε (Ο, π( .

α) Να αποδειχθεί ότι: f(x) = � ημ2χ. β) Να γίνει η γραφική παράσταση της συνάρτησης f. 2

3 γ) Να βρεθούν τα σημεία τομής του γραφήματος της f με την ευθεία y = - .

4

δ) Να συγκρίνετε τους αριθμούς t(�) και t(�) 1000 2000

.

ΕΥΚΛΕΙΔΗΣ Β ' 69 τ. l/43

Μαθηματικά για την Β ' Λυκείου

α) Έχουμε

}{χ) = Ξ[ συν' ( χ -�) - συν' ( χ +�)] = Ξ[ συν ( χ -�) + συν ( χ +�)] [συ{ χ -�) - συ{ χ +�)] =

= � (συνχσυν π + ημχημ π + συνχσυν π - ημχημ π) · 2 4 4 4 4

(συνχσυν π + ημχημ π - συνχσυν π + ημχημ π ) = � .fiσυνχ .fiημχ = 3ημχσυνχ = 4 4 4 4 2

3 = 2 ημ2χ ,

άρα j{x) = � ημ2χ, χ Ε [Ο, π] . 2

3 3 ' 2π β) Η f έχει μέγιστο /,nax = 2 , ελάχιστο /,nin = -2 και περιοδο Τ = l = π .

.χ '

312

ο π/4j π/2 • 3π/4 π

-3/2 """""""""" '" '""" ' ' "

y'

χ

χ π/4 �/2 �π/4

f(x) 3/2 ο 3/2

π

ο

γ) Οι τετμημένες των σημείων τομής είναι οι λύσεις της εξίσωσης ι( χ) = � ,( I ) χ Ε [Ο,π] 4

3 3 1 π π π Αλλά ( l ) <=> - ημ2χ = - <=> ημ2χ = - <=> ημ2χ = ημ - <=> 2χ = 2κπ + - <::::> χ = κπ + - ή 2 4 2 6 6 1 2

2 χ = 2κπ + Sπ <=> χ = κπ + Sπ , κ Ε Ζ . Ομως χ Ε [0, π] <=> κ = Ο. Οι λύσεις λοιπόν είναι 6 1 2

χ = Sπ . Άρα τα σημεία τομής είναι: Α(� , �) και Β( Sπ , �) .

1 2 1 2 4 1 2 4

π Χ = - ή 1 2

δ) Οι αριθμοί _π_ και _

π_ ανήκουν στο διάστημα [ο, π4] στο οποίο η f είναι γνησίως αύξουσα,

1 000 2000

οπότε : 1 0�0 > 20�0 � ι( Ι 0�0

) > ι( 20�0) .

'Εστω η συνάρτηση /(χ) = 2 - 2συν2χ - 64ημ4 �συν 4 �

. 2 2

α) Να δειχθεί ότι f(x) = η μ 2

2χ . β) Να βρεθεί η περίοδος της f και τα ακρότατα της.

γ) Να λυθεί η εξίσωση f(x) + 2� f(x) - 3 = Ο με χ ε [Ο, : ] · α) Πεδίο ορισμού της f είναι Α = 9\ .

4 χ 4 χ 4 χ 4 χ Έχουμε fiχ) = 2 - 2συν2χ - 64ημ - συν - = 2 - 2συν2χ - 4 · 1 6ημ - συν - =

2 2 2 2

ΕΥΚΛΕΙΔΗΣ Β ' 69 τ. l/44

Μαθηματικά για την Β ' Λυκείου

( χ χ ) 4 ( 1 - συν2χ ) 2 = 2 - 2συν2χ - 4 2ημ 2 συν 2 = 2 - 2συν2χ - 4ημ4 χ = 2 - 2συν2χ - 4

2 =

= 2 - 2συν2χ - 1 + 2συν2χ - συν 2 2χ = 1 - συν 2 2χ = ημ2 2χ . , 2 1 - συν4χ 1 1 , β) Εχουμε j{x) = ημ 2χ = = - - συν4χ + - . Οπότε η περιοδος της συνάρτησης είναι:

2 2 2 2π π 1 1 1 Τ = 4 = 2 Η συνάρτηση fι (χ) = - 2 συν4χ έχει μέγιστο 2 και ελάχιστο

2, οπότε η

' ' ( ' !. 1 1 1 λ !. 1 1 ο μεγιστη τιμ η της ει ναι max = - + - = και η ε άχιστη ηίn = - - + - = . 2 2 I 2 2

γ) Αφού f(x) :Ξ:: Ο, για κάθε χ ε 9\, η εξίσωση ορίζεται σε όλο το 9\, όπου :

j{x) + 2� j{x) - 3 = Ο q ημ 2 2χ + 2�ημ 2 2χ - 3 = Ο q ημ 2 2χ + 2lημ2χ l - 3 = Ο G ημ22χ + 2ημ2χ

- 3 = ο, αφού χ ε [ο, π ] � Ο � χ � π � Ο � 2χ � π � ημ2χ � ο .

4 4 2 Στην εξίσωση η μ 2 2χ + 2ημ2χ - 3 = Ο , θέτουμε ημ2χ = ω ( 1 ) οπότε η επιλύουσά της είναι η

ω2 + 2ω - 3 = 0 , με ρίζες ω 1 = 1 , ω2 = -3 . Για ω = 1 έχουμε: ( 1 ) G ημ2χ = 1 q ημ2χ = ημ π

q 2χ = 2κπ + π q χ = κπ + π , με κεΖ

2 2 4

Όμως χ ε [ο, π ] G Ο :Ξ:;; κπ + π :::;; π

G _ _!_ :Ξ:;; κ :Ξ:;; Ο q κ = Ο. Η ( l ) G χ = π . 4 4 4 4 4

Αν ω = -3 έχουμε : (l ) G ημ2χ = -3 αδύνατη, αφού - 1 � ημ2χ � 1 .

π α) Να δείξετε ότι ο αριθμός χ = - είναι ρίζα της εξίσωσης συν3χ = ημ2χ .

10

β) Να δείξετε ότι συν3χ = 4συν 3 χ - 3συνχ

γ) Να δείξετε ότι συν3χ - ημ2χ = συνχ(-4ημ2 χ - 2ημχ + 1)

π δ) Να δείξετε ότι ο αριθμός ημ

10 είναι μια ρίζα της εξίσωσης 4χ 2 + 2χ - 1 = ο

ε) Να υπολογίσετε τα ημ9°, ημ36° .

α) Αρκεί να αποδείξουμε ότι 3π π συν - = ημ- . Ισχύει

3π π π - + - = - δηλαδή οι γωνίες είναι 1 0 5 1 0 5 2

,

3π π συμπληρωματικές άρα συν - = η μ- .

1 0 5 β) Έχουμε συν3χ = συν(2χ + χ) = συν2χσυνχ - ημ2χημχ = = (2συν 2 χ - l )συνχ - 2ημχσυνχημχ = 2συν 3 χ - συνχ - 2ημ 2 χσυνχ =

3 2 '

3 3 = 2συν χ - συνχ - 2( 1 - συν χ)συνχ = 2συν ο χ - συνχ - 2συνχ + 2συν χ = = 4συν χ - 3συνχ . (β)

γ) Α = συν3χ - ημ2χ = 4συν 3 χ - 3συνχ - 2ημχσυνχ = συνχ(4συν 2 χ - 3 - 2ημχ) = = συνχ (4( 1 - ημ 2 χ) - 3 - 2ημχ) = συνχ( -4ημ 2 χ - 2ημχ + 1 ) .

δ) Αρκεί να δείξουμε ότι 4ημ 2 � + 2ημ � - 1 = Ο . Πολλαπλασιάζουμε και τα δύο μέλη της με 1 0 1 0

π _J. ο , . . π (4 2 π 2 π ι) ο . - συν - τ , οποτε αρκει να ισχυει: - συν - ημ - + ημ - - = η

1 0 1 0 1 0 1 0

ΕΥΚΛΕΙΔΗΣ Β ' 69 τ. l/45

Μαθηματικά για την Β ' Λυκείου

συν � (- 4ημ 2 � - 2ημ� + ι) = 0 . ( 1 ) Από το ερώτημα (γ) για χ = � διαπιστώνουμε ότι για να 1 0 1 0 1 0 1 0

. 3π π ο . 3π π . . . ( ισχύει η ( I ) αρκει: συν - - η μ - = η συν - = η μ - , το οποιο ισχυει απο το α) . 1 0 5 1 0 5

ε) Από το (δ) το ημ� = ημl 8° είναι μια λύση της εξίσωσης 4χ 2 + 2χ - 1 = 0 1 0

- l + J5 - I - JS Η διακρίνουσα είναι Δ=20. Άρα οι ρίζες της είναι χ 1 = , Χ2

= ---4 4

- I + J5 - 1 - JS Δηλαδή η μl 8° = >0, αφού < Ο . Επίσης ημ2 1 8° + συν2 1 8° = I <=>

4 4

συν 2 1 8 --1 - (- Ι + J5

J2

_,._..,_ συν 2 Ι 8 --1 0 + 2J5 1 8 ° Ο --........ και αφού συν > θα είναι 4 1 6

80 �1 0 + 2J5 2 1 - συν2α 90 ? 9ο 1 - συν1 8° συν Ι = . Από τον τύπο ημ α = για α= έχουμε ημ- = ----4 2 2

1 �r-1 0_+_2_J5=5 -4 4 - �1 0 + 2-JS . J4 - �1 0 + 2J5 = = και αφού ημ9° > Ο θα ειναι: ημ9° = . Στον

2 8 8 τύπο ημ2α=2ημασυνα για α = 1 8° ισχύει ημ36° = 2ημ l 8°συν l 8°

= 2 - I + JS �1 0 + 2J5 = ( - l + J5)�1 0 + 2J5 . 4 4 8

3 1 3 5 ) α) Ν α αποδείξετε ότι η μ 6 χ + συν 6 χ = 1 - - η μ 2 2χ = - + - συν 2 2χ 4 4 4

5 β) Να λύσετε την εξίσωση ημ 6 χ + συν 6 χ = -

8

• • • 6 π 6 π γ) Να υπολογισετε την τιμη της παραστασης ημ - + συν -

1 2 12

δ) Να βρεθούν η περίοδος, η μέγιστη και η ελάχιστη τιμή της συνάρτησης f(x) = ημ 6χ + συν 6χ

. \ "iJση α) ημ 6 χ + συν 6 χ = (ημ 2 χ) 3 + (συν 2 χ) 3 = (ημ 2 χ + συν 2 χΧημ 4 χ - ημ 2 χσυν 2 χ + συν 4χ ) = = (ημ 2 χΥ + (συν 2 χΥ - ημ 2χσυν 2 χ = (ημ 2 χ + συν 2χΥ - 2ημ 2 χσυν 2 χ - ημ 2 χσυν 2 χ =

= 1 - 3ημ 2 χσυν 2 χ = 1 - 3(ημχσυνχ )' = 1 - {Ξ� ημ2χ ) ' = 1 - �η μ 2 2χ =

3( 2 ) 1 3 2 = Ι - - 1 - συν 2χ = - + - συν 2χ.

4 4 4

6 6 5 (α) 3 2 5 3 2 3 2 Ι J2 β) ημ χ + συν χ = - <=> 1 - -ημ 2χ = - <=> - ημ 2χ = - <=> ημ 2χ = - <=> ημ2χ = ± - .

8 4 8 4 8 2 2 J2 π 2 2 π π . ημ2χ = - <=> ημ2χ = ημ- <=> χ = κπ + - <=> χ = κπ + - η 2 4 4 8

ΕΥΚΛΕΙΔΗΣ Β ' 69 τ. Ι /46

Μ αθη ματικά για την Β ' Λυκείου

π 3π 2χ = 2κπ + π - - � χ = κπ + - , κ Ε Ζ . 4 8

12 π π π ημ2χ = - - � ημ2χ = ημ(-- ) � 2χ = 2κπ - - � χ = κπ - - ή 2 4 4 8

π 5π 2χ = 2κπ + π + - � χ = κπ + - , κ Ε Ζ . 4 8

6 Π 6 Π (α) 3 2 Π 3 2 Π 3 1 1 3 γ) ημ - + συν - = 1 - - ημ 2 - = 1 - - ημ - = 1 - - · - = -

1 2 1 2 4 1 2 4 6 4 4 1 6 < α) 3 � 3 1 - συν4χ 3 3 3 5

δ) ./(χ) = 1 - - ημ- 2χ = 1 - - · = 1 - - + - συν4χ = - συν4χ + - . 4 4 2 8 8 8 8

. . δ . τ 2π π . . f . !. 3 5 λ . Οποτε η περιο ος ειναι: = 4 = 2 . Η μεγιστη τιμ η της ειναι ιnax = 8 + 8 και η ε αχιστη

3 5 1 J.nin = -8 + 8 = 4 .

6 ) α) Να δείξετε ότι 2ημ 2 ( α ; β ) = 1 - συνασυνβ + ημαημβ

β) Να δείξετε ότι ημαημβ ::::; ημ 2 ( α; β ) γ) Να δείξετε ότι ημαημ199α ::::; ημ 2 100α

π .fi 1 δ) Αν Ο<α,β< - , ημαημβ = - και εφαεφβ = - , να υπολογίσετε τους αριθμούς:

2 10 6

η μ( α ; β} συν( α + β), εφ( α + β), εφ α και εφ β.

Λί1ση , ( α + β ) 1 - συν( α + β)

α) Ισχύει 2ημ - -2- = 2

2 = 1 - συν( α + β) = 1 - συνασυνβ + ημαημβ

ο ( α + β ) . " ( α + β ) β) Αρκεί : ημαημβ ::::; ημ--2- η 2ημαημβ ::::; 2ημ -

-2-ή

2ημαημβ � 1 - συνασυνβ + ημαημβ ή συνασυνβ + ημαημβ � 1 ή συν( α - β) :::; Ι , που ισχύει.

γ) Από το (β) ερώτημα αντικαθιστώντας όπου β το 1 99α έχουμε : Ί ( α + 1 99α ) Ί ημαημ1 99α � η μ -

2 � ημαημ1 99α � η μ - ι ΟΟα .

12 . 1 ημα ημβ 1 10 1 312

δ) Ε χουμε εφαεφβ = - � -- -- = - � = - � συνασυνβ = -- . 6 συνα συνβ 6 συνασυνβ 6 5

. . . 2 z ( α + β ) 1 β β Απο το (α) ερωτημα ισχυει: ημ -2-

= - συνασυν + ημαημ =

1 _ 312 + 12 = 5 - 712 . Άρα ημ( α + β ) � �5 - 7,/2. 2 1 0 5 2 5

Επειδή η μ( α; β ) > Ο , αφού Ο < α < ; και Ο < β < ; , δηλαδή Ο < α; β

< ; . ΕΥΚΛΕΙΔΗΣ Β ' 69 τ. l/47

Μ αθηματικά για την Β ' Λυκείου

3-fi .fi .fi Θα έχουμε συν( α + β) = συνασυνβ - ημαημβ = -- - - = - .

5 1 0 2

Ισχύει ημ 2 (α + β) = 1 - συν 2 (α + β) = 1 - ..!_ = ..!_ , άρα ημ(α + β) = .fi , αφού ημ(α+β) >Ο επειδή 2 2 2

Ο < α + β < π . Άρα εφ( α + β) = η μ( α + β) = 1 . Οπότε: εφ( α + β) = Ι � συν( α + β)

εφα + εφβ εφα + εφβ 5 5 , ____:. __ .:._:__ = 1 => = 1 => εφα + εφ β = - � εφα + εφ β = - , εχουμε και 1 - εφαεφβ 1 _ ..!_ 6 6

6

1 εφαεφβ = - , 6

7 5 1 οπότε οι αριθμοί εφα και εφβ είναι οι ρίζες της εξίσωσης: χ - - - χ + - = Ο . Λύνοντας:

6 6 1 1 1 1 εφα = - και εφβ = - ή εφα = - και εφβ = - . 2 3 3 2

7) α) Να αποδείξετε ότι σε κάθε τρίγωνο ΑΒΓ ισχύει η σχέση : σφΑσφΒ + σφΒσφΓ + σφΓσφΑ = 1

β) Αν σε οξυγώνιο τρίγωνο ΑΒΓ ισχύει σφ 2 Α + σφ 2Β + σφ 2 Γ = 1 , να αποδείξετε:

i) σφΑ + σφΒ + σφΓ = .J3 ii) Το τρίγωνο ΑΒΓ είναι ισόπλευρο.

α) Αφού οι Α, Β, Γ είναι γωνίες τριγώνου ισχύει Α + Β + Γ = π � Α + Β = π - Γ � σφΑσφΒ - 1 σφ(Α + Β) = σφ(π - Γ) => = -σφΓ => σφΑσφΒ - 1 = -σφΒσγΓ - σφΑσφΓ => σφΒ + σφΑ

σφΑσφΒ + σφΒσφΓ + σφΓσφΑ = 1 .

β) i ) (σφΑ + σφΒ + σφrγ = σφ 2 Α + σφ 2Β + σφ 2Γ + 2σφΑσφΒ + 2σφΒσφΓ + 2σφΓσφΑ = (α)

= 1 + 2(σφΑσφΒ + σφΒσφΓ + σφΓσφΑ) = 1 + 2 = 3 Όμως σφΑ, σφΒ, σφΓ > Ο αφού το τρίγωνο

είναι οξυγώνιο, άρα σφΑ + σφΒ + σφΓ = J3 . i i ) Ισχύουν σφΑσφΒ + σφΒσφΓ + σφΓσφΑ = 1 και σφ 2 Α + σφ2Β + σφ2Γ = 1 άρα:

σφΑσφΒ + σφΒσφΓ + σφΓσφΑ = σφ 2 Α + σφ 2Β + σφ 2Γ => σφ 2 Α + σφ 2Β + σφ2 Γ - σφΑσφΒ - σφΒσφΓ - σφΓσφΑ = Ο => ..!. [(σφΑ - σφΒ γ + (σφΒ - σφrγ + (σφΓ - σφΑγ ] = Ο � σφΑ-σφΒ = Ο και σφΒ - σφΓ = Ο και 2 σφΓ - σφΑ = Ο � σφΑ = σφΒ = σφΓ � Α = Β = Γ, αφού είναι γωνίες τριγώνου . Άρα το τρίγωνο είναι ισόπλευρο.

8) Να βρεθούν οι τιμές των πραγματικών αριθμών α και β ώστε να ισχύει για κάθε γωνία χ φ. κπ,

κ Ε Ζ η ισότητα: -1

- = ασφ χ + βσφχ . Ακολούθως να υπολογισθεί το άθροισμα: S =

ημχ 2

1 1 1 1 , ν -- + -- + -- + • • . + , οταν 2 α ;:f:. κπ για ν, κ Ε Ζ. η μα ημ2α ημ4α ημ2 ν α

ΕΥΚΛΕΙΔΗΣ Β ' 69 τ. t /48

Μαθη ματικά για την Β ' Λυκείου

ι\{Jση χ συν -1 χ

Η ισότητα που θέλουμε να ισχύει γίνεται: -- = ασφ - + βσφχ <=> ημχ 2

1 = α --2-ημχ + χ ημ -2

χ �ν - χ συνχ 2 χ χ - 1 + �νχ + β -- ημχ <=> 1 = α -- 2ημ - �ν - + βσυνχ <=> 1 = ασυν2 2 + βσυνχ <=> 1 = 2α ---

ημχ χ 2 2 2 ημ -2

+ βσυνχ <=> 1 = α +ασυνχ + βσυνχ <=> (α + β)συνχ + (α - Ι ) = 0. ( 1 ) Έστω η ( 1 ) ισχύει για κάθε χ =j:. κπ, π π 1

οπότε για χ = - είναι: α - 1 = Ο και για χ = - είναι: (α + β) - + α - 1 = Ο. Άρα έχουμε: 2 3 2 {α - 1 = 0 } <=> {α = 1 } . Για α = ι και β = - 1 η ( 1 ) γίνεται: ( 1 - 1 )συνχ + ( 1 - 1 ) = Ο, που

3α + β = 2 β = - 1 ισχύει για κάθε χ =j:. κπ. Γενικά λοιπόν πρέπει και αρκεί α = 1 και β = - 1 .

1 1 1 1 α Επομένως: S = -- + -- + -- + . . . + = σφ - - σφα + σφα - σφ2α + σφ2α - σφ4α

ημα ημ2α ημ4α ημ2 ν α 2

9) Αν για την γωνία χ ισχύουν νχ =j:. κ π

και � =j:. κ π

για κάθε ν, κ ε Ζ να αποδειχθεί ότι: 2 ν 2

2 σφ 2 χ - ι α) σφ χ = β) εφχ = σφχ - 2σφ2χ γ) εφχ + 2εφ2χ + 4εφ4χ + 8σφ8χ = σφχ .

2σφχ

ι χ ι χ ι χ ι χ ι χ δ) - εφ - + - εφ - + - εφ - + . • . + - εφ - = - σφ - - σφχ .

2 2 4 4 8 8 2 ν 2 ν 2 ν 2 ν Λ i'ωΊ�

σφχσφχ - 1 σφ 2 χ - 1 α) Είναι: σφ2χ = σφ(χ + χ) = = .

σφχ + σφχ 2σφχ σφ 2 χ - Ι σ φ 2 χ - σφ 2 χ + 1 1

β) σφχ - 2σφ2χ = σφχ - 2 = = -- = εφχ . 2σφχ σφχ σφχ

γ) Από (β) ερώτημα ισχύει: εφχ = σφχ - 2σφ2χ , για οποιαδήποτε γωνία χ, άρα και για 2χ: εφ2χ = σφ2χ - 2σφ4χ . Επίσης για 4χ έχουμε : εφ4χ = σφ4χ - 2σφ8χ . Άρα: εφχ + 2εφ2χ + 4εφ4χ + 8σφ8χ = σφχ - 2σφ2χ + 2σφ2χ - 4σφ4χ + 4σφ4χ - 8σφ8χ + 8σφ8χ = σφχ.

, , , Χ Χ Χ Π , Χ Χ . δ) Απο το (β) ερωτημα αφου - , - , . . . . , - -:�; κ - εχουμε: εφ - = σφ - - 2σφχ . Επισης: 2 4 2ν 2 2 2

χ χ χ χ χ χ χ χ χ εφ - = σφ - - 2σφ - εφ - = σφ - - 2σφ - . . . εφ - = σφ - - 2σφ -4 4 2 ' 8 8 4 ' ' 2 ν 2 ν 2 ν- Ι

1 χ 1 χ 1 χ 1 χ Άρα: - εφ - + - εφ - + -εφ - + . . . + - εφ - =

2 2 4 4 8 8 2 ν 2 ν 1 χ 1 χ 1 χ 1 χ 1 χ 1 χ 1 χ = - σφ - - σφχ + - σφ - - - σφ - + - σφ - - - σφ - + . . . + - σφ - - -- σφ -- = 2 2 4 4 2 2 8 8 4 4 2 ν 2 ν 2 ν- Ι 2 ν-Ι 1 χ = - σφ - - σφχ .

2 ν 2 ν

ΕΥΚΛΕΙΔΗΣ Β ' 69 τ. l/49

Μ αθηματικά για την Β ' Λυκείου

Γεω μετρία Β ' Λυκείου Θεω ρή ματα διχοτό μων - Αρ μονικά συζυγή

Ομοιότητα τριγώνων. Καρδαμίτσης Σπύρος

«Τά δμοια πολύyωνα ε%; τε δμοια τρίγωνα διαιρείι:αι καί είς ίiJα τό πλήθος καί Φόλογα τοk; δλοις, καί τό πολύyωνον πρό:; τό πολύyωνον διπλασίονα λόγον έχει /fπερ ή Φόλογος πλευρά πρό:; τήv Φόλογον πλευρόν. >>

ΑΣ Κ Η Σ Η 1 '1 Σε τρίγωνο ΑΒΓ με β > γ η εσωτερική και

εξωτερική διχοτόμος της γωνίας Α τέμνουν την ευθεία της πλευράς ΒΓ στα σημεία Δ και Ε αντίστοιχα.

α) Να δείξετε ότι τα σημεία Δ και Ε είναι αρμονικά συζυγή των Β και Γ.

β) Αν α = 13, β = 14 και γ = 12 να υπολογιστεί το μήκος του ευθυγράμμου τμήματος ΔΕ.

Λ Υ Σ Η

Ε . - - - - - -

- - - - - - - ­

_ _ . . - - - - - -

· --..(to=_:"- -

α) Είναι η ΑΔ διχοτόμος της γωνίας Α, επομένως ΑΒ ΔΒ ξ . δ . - = - , και ε ωτερικη ιχοτόμος της γωνιας Α, ΑΓ ΔΓ

ΑΒ ΕΒ ΔΒ ΕΒ οπότε - =- άρα - = - , δηλαδή τα σημεία ΑΓ ΕΓ ΔΓ ΕΓ Δ και Ε είναι αρμονικά συζυγή των Β και Γ.

β) Ε . αγ 1 3 · 1 2 6 . β ιναι ΔΒ = -- = = αφου > γ β + γ 1 4 + 1 2

. . αγ 1 3 · 1 2 8 εχουμε οτι ΕΒ = -- = 7

β - γ 1 4 - 1 2

ΑΣ Κ Η Σ Η 2 '� Δίνεται παραλληλόγραμμο ΑΒΓ Δ. Οι

διχοτόμοι των γωνιών του Α και Δ τέμνουν τις διαγώνιές του ΒΔ και ΑΓ στα σημεία Κ και Λ αντίστοιχα. Να αποδείξετε ότι:

α) ΑΚ .l ΔΛ β) ΚΛ // ΒΓ Λ Υ Σ Η

α) Αφού ο ι ΑΚ και ΔΛ είναι διχοτόμοι των γωνιών Α και Δ αντίστοιχα τότε

Από το V βιβλίο των στοιχείων του Ευκλείδη

I _-\

Λ Λ Λ Λ Λ Λ ΔΑΚ = ΚΑΒ = ω και Γ ΔΛ = ΛΔΑ = φ

··'- ·, ·'· 'J' φ ·

t:r·"

Β

Γ

Αφού το τετράπλευρο ΑΒΓ Δ είναι παραλληλόγραμμο είναι Α + Δ = 1 80° ή 2ω + 2φ = 1 80 ή ω + φ = 90° άρα ΑΚ .l ΔΛ β) Εφ' όσον οι ΑΚ και ΔΛ είναι διχοτόμοι των γωνιών ΑΔΒ και ΑΔΓ αντίστοιχα, τότε ισχύουν: ΑΔ ΚΔ ΑΔ ΛΑ - = - και - = -ΑΒ ΚΒ ΓΔ ΛΚ αλλά ΑΒ = ΓΔ (ΑΒΓΔ είναι παραλληλόγραμμο) τότε από τις παραπάνω σχέσεις έχουμε ότι ΚΔ ΛΑ . . θ ' - = - , οποτε συμφωνα με το εωρημα του ΚΒ ΛΚ Θαλή έχουμε ΚΛ // ΑΔ.

Λl: Κ Η Σ Η 3 '1 Δίνεται κύκλος με διάμετρο (O,R) με

διάμετρο ΑΒ. Έστω Μ τυχαίο σημείο του κύκλου και Γ Δ μια χορδή κάθετη στην διάμετρο ΑΒ. Αν τα τμήματα ΜΓ και ΜΔ τέμνουν την διάμετρο ΑΒ στα σημεία Ε και Ζ αντίστοιχα, να δείξετε ότι τα σημεία Β και Α είναι αρμονικά συζυγή ως προς τα Ζ και Ε.

Λ V Σ Η

Επειδή είναι Γ Δ κάθετη στην ΑΒ τότε είναι Λ Λ

ΒΓ = ΒΔ, επομένως και Μ 1 = Μ2 , δηλαδή η ΜΒ είναι εσωτερική διχοτόμος της γωνίας Μ στο

ΜΖ ΒΖ τρίγωνο ΖΜΕ, άρα - = - ( 1 )

ΜΕ ΒΕ

ΕΥΚΛΕΙΔΗΣ Β ' 6 9 τ. l/50

Μ αθη ματικά για την Β ' Λυκείου

Γ

Ε �� • ) ·---- � . . ;) ··· -­Β

· - . <.'

Λ Επειδή ΑΜΒ =90° ως γωνία εγγεγραμμένη σε

ημικύκλιο είναι ΜΑ l_ ΜΒ δηλαδή η ΜΑ είναι εξωτερική διχοτόμος της γωνίας Μ στο τρίγωνο

, ΜΖ ΑΖ ΖΜΕ, αρα - = - (2)

ΜΕ ΑΕ Τέλος από τις σχέσεις ( 1 ) και (2) έχουμε :

ΒΖ ΑΖ , - = - , δηλαδη τα σημεία Β και Α είναι ΒΕ ΑΕ αρμονικά συζυγή ως προς τα Ζ και Ε .

Δίνεται ισοσκελές τρίγωνο ΑΒΓ (ΑΒ = ΑΓ), το ύψος του Γ Δ και ημιευθεία Γχ l_ ΑΓ, που τέμνει την πλευρά ΑΒ στο σημείο Ε. Να

δ 'ξ , ΒΔ ΓΔ

απο ει ετε οτι: - = - .

ΒΕ ΓΕ

Γ

Λ Λ Στο τρίγωνο ΒΓΕ έχουμε ότι: ΒΓΕ = 90° - Γ

Λ Λ και στο τρίγωνο ΒΔΓ έχουμε ότι: ΒΓ Δ = 90° - Β αλλά είναι Β = Γ αφού το τρίγωνο ΑΒΓ είναι

Λ Λ ισοσκελές επομένως είναι: ΒΓΕ = ΒΔΓ , δηλαδή η ΒΓ είναι διχοτόμος της γωνίας ΔΓΕ του τριγώνου ΔΓΕ επομένως σύμφωνα με το θεώρημα

ΒΔ ΓΔ εσωτερικής διχοτόμου έχουμε : - = -

ΒΕ ΓΕ

ΛΠ...: ; 1 Σ Η

φέρνουμε ευθεία παράλληλη της διχοτόμου ΒΔ που τέμνει την ΒΓ στο σημείο Ζ και την προέκταση της ΑΒ στο σημείο Ε. Να αποδείξετε ότι ΓΖ = ΑΕ.

• ·

Λόγω των παραλλήλων ΒΔ και ΕΜ από το θεώρημα του Θαλή έχουμε τις αναλογίες

ΑΒ ΑΔ ΓΖ ΓΜ - = - ( 1 ) και - = - (2) ΑΕ ΑΜ ΓΒ ΓΔ

πολλαπλασιάζοντας τις σχέσεις ( I ) και (2) έχουμε: ΑΒ . ΓΖ

= ΑΔ . ΓΜ

(3) ΑΕ ΓΒ ΑΜ ΓΔ

επειδή η ΒΔ είναι εσωτερική διχοτόμος του τριγώνου ΑΒΓ από το θεώρημα της εσωτερικής

. ΑΒ ΔΑ διχοτόμου έχουμε την αναλογια - = - με

ΒΓ ΔΓ βάση την αναλογία αυτή η σχέση (3)

ΓΖ ΓΜ δ

' - = -- επει η το Μ ΑΕ ΑΜ

απλοποιούμενη γίνεται:

ΓΖ ΓΜ είναι μέσο της ΑΓ έχουμε τελικά - = -- = 1

ΑΕ ΑΜ άρα ΓΖ = ΑΕ .

. \ Σ κ n η.: � ι Οι μη παράλληλες πλευρές ΑΔ και ΒΓ

τραπεζίου ΑΒΓ Δ τέμνονται στο σημείο Ο. Α ν είναι AB=12cm, ΓΔ = ΑΔ = 4cm και ΒΓ = 8cm, να υπολογίσετε τα μήκη των ευθυγράμμων τμημάτων ΟΔ και ΟΓ.

Τα τρίγωνα ΟΓΔ και ΟΑΒ έχουν την γωνία Ο Λ Λ

κοινή και ΟΓΔ = ΟΑΒ λόγω των παραλλήλων πλευρών του τραπεζίου, επομένως είναι όμοια

. ΟΔ ΟΓ ΓΔ . ΟΔ ΟΓ ΓΔ οποτε:

ΟΑ=

ΟΒ =

ΑΒ η

ΟΔ+ΑΔ =

ΟΓ+ΒΓ =

ΑΒ . ΟΔ οΓ 4 1 . η = = - = - επομενως έχουμε :

0Δ + 4 ΟΓ + 8 1 2 3 ΟΔ 1

-- = - :::::>30Δ=ΟΔ+4=>0Δ=2cm. 0Δ+4 3

ΟΓ 1 Δίνεται τρίγωνο ΑΒΓ και ΒΔ η διχοτόμος και = - => 30Γ = ΟΓ + 8 => ΟΓ = 4cm.

του. Από το μέσο Μ της πλευράς του ΑΓ ΟΓ + 8 3

ΕΥΚΛΕΙΔΗΣ Β ' 69 τ.l/51

Μαθηματικά για την Β ' Λυκείου

Δίνεται κύκλος κέντρου Ο και σημείο Ρ εξωτερικό του κύκλου. Από το σημείο Ρ φέρνουμε μια εφαπτόμενη του κύκλου ΡΑ και

, Αιf ΡΒ μια τεμνουσα ΡΒΓ. Να αποδείξετε ότι: -----=::2 = -

ΑΓ ΡΓ

Τα τρίγωνα ΡΑΒ και ΡΑΓ έχουν την γωνία Ρ Λ Λ

κοινή και Ρ ΑΒ = ΑΓΒ ( χορδή και εφαπτομένη) , , , , ΑΒ ΡΒ ΡΑ ,

επομενως ειναι ομοια, οποτε: - = - = - η ΑΓ ΡΑ ΡΓ

ΑΒ ΡΒ ΑΒ ΡΑ - = - ( ! ) και - = - (2) ΑΓ ΡΑ ΑΓ ΡΓ με πολλαπλασιασμό των σχέσεων ( 1 ) και (2)

, ΑΒ ΑΒ ΡΒ ΡΑ ΑΒ2 ΡΒ εχουμε : - · - = - · - => -- = -

ΑΓ ΑΓ ΡΑ ΡΓ ΑΓ2 ΡΓ

Σε τραπέζιο ΑΒΓ Δ οι μη παράλληλες πλευρές του ΑΔ και ΒΓ τέμνονται σε σημείο Κ. Από το σημείο Κ φέρνουμε ευθεία παράλληλη προς τις βάσεις του τραπεζίου, που τέμνει τις προεκτάσεις των διαγωνίων του τραπεζίου ΒΔ και ΑΓ στα σημεία Ε και Ζ αντίστοιχα. Να

δείξετε ότι: ΒΓ ΔΓ

-- - --

ΒΚ ΕΚ ΕΚ = ΚΖ

Λ α) Τα τρίγωνα ΒΓΔ και ΒΕΚ έχουν την γωνία Bl

Λ Λ κοινή και ΕΒΚ = ΔΓΒ λόγω των παραλλήλων ευθειών (ε) και ΓΔ. Συνεπώς είναι όμοια άρα ΒΓ ΔΓ , ,

-- = - β) Αναλογα με το πρώτο ερωτημα και ΒΚ ΕΚ

ΑΔ ΔΓ , , , , -- = - επειδη λογω των παραλληλων ευθειων ΑΚ ΚΖ

ΒΓ ΑΔ (ε), Γ Δ και ΑΒ ισχύει - - - από τις

ΒΚ ΑΚ ΔΓ ΔΓ

προηγούμενες σχέσεις προκύπτει ότι ΕΚ κz '

άρα ΕΚ = ΚΖ.

Δίνεται τρίγωνο ΑΒΓ και Μ τυχαίο σημείο της πλευράς του ΒΓ. Έστω Δ, Ε τα μέσα των πλευρών του ΑΒ και ΑΓ αντίστοιχα. Η ευθεία ΕΜ τέμνει την ΑΒ στο σημείο Λ και η ευθεία ΜΔ την ΑΓ στο σημείο Κ. Αν είναι ΑΘ//ΔΕ τότε

να αποδείξετε ότι: ΚΑ ΒΜ

-- - --ΚΕ ΔΕ

ΑΜ//ΚΛ

α) Τα Δ και Ε είναι τα μέσα των πλευρών του τριγώνου ΑΒΓ συνεπώς ΔΕ//ΒΓ. Αφού ΑΘ//ΔΕ

Λ Λ τότε θα είναι και ΑΘ//ΒΓ άρα Al = Β ως εντός

Λ Λ

εναλλάξ γωνίες, Δ ι = Δ2 ως κατακορυφή γωνίες και ΑΔ = ΒΔ, συνεπώς τα τρίγωνα ΑΘΔ και ΒΔΜ είναι ίσα. Τα τρίγωνα ΚΑΘ και ΚΔΕ είναι όμοια τότε ΚΑ ΑΘ

- = - και ΚΕ ΔΕ

από την ισότητα των τριγώνων ΑΘΔ και ΔΒΜ είναι ΑΘ=ΒΜ, οπότε η σχέση

, ΚΑ ΒΜ γινεται -- = --

ΚΕ ΔΕ β) Από την ομοιότητα των τριγώνων ΛΒΜ και

ΑΜ ΒΜ ΛΔΕ έχουμε -- = -- και με βάση το α)

ΑΕ ΔΕ ΚΑ ΑΜ

ερώτημα έχουμε -- - -- οπότε από το ΚΕ ΑΕ

Θεώρημα του Θαλή είναι ΑΜ//ΚΑ τα τρίγωνα ΑΔΓ και ΑΚΖ είναι όμοια οπότε

ΕΥΚΛΕΙΔΗΣ Β ' 69 τ. l/52

Μαθη ματικά για την Β ' Λυκείου

Δίνεται τρίγωνο ΑΒΓ και Μ το μέσο της πλευράς του ΒΓ. Από το σημείο Μ φέρνουμε τυχαία ευθεία (ε) που τέμνει τις ΑΒ και ΑΓ στα σημεία Ρ και Κ αντίστοιχα και από το σημείο Α ευθεία (η) παράλληλη της πλευράς ΒΓ του τριγώνου που τέμνει την ευθεία (ε) στο σημείο Ν. Να αποδείξετε ότι: Τα τρίγωνα ΡΑΝ και ΡΒΜ είναι όμοια ΡΝ·ΚΜ = ΡΜ·ΚΝ

α) Τα τρίγωνα ΡΑΝ και ΡΒΜ έχουν την γωνία Λ Λ Λ Ρ κοινή και Ν = Μ 2 λόγω των παραλλήλων (η)

, , , , ΡΝ ΑΝ και ΒΓ, επομενως ειναι ομοια, αρα -- = --

ΡΜ ΒΜ Λ Λ

β) Τα τρίγωνα ΚΑΝ και ΚΜΓ έχουν Κι = Κ 2 (ως Λ Λ

κατακορυφή γωνίες) και Ν = Μ ι επομένως είναι

, , ΚΝ ΑΝ λλ ' , ομοια, αρα -- = -- α α ΜΒ = ΜΓ αφου το

ΚΜ ΜΓ

Μ είναι μέσο της πλευράς ΒΓ, οπότε η παραπάνω

, , ΚΝ ΑΝ σχεση γραφεται: -- = -- .

ΚΜ ΒΜ

- - -- - - � -

//

Β �Ι( / / Μ - '• Γ

I / I / I / Υ H l

Τέλος από την παραπάνω σχέση και από το (α)

, , ΡΝ ΚΝ ερωτημα εχουμε: - = -- =:>ΡΝ·ΚΜ=ΡΜ·ΚΝ ΡΜ ΚΜ

Με βαθιά θλίψη η μαθηματική κοινότητα έχασε, φέτος το καλοκαίρι, στις 20 Αυγούστου, έναν υπέροχο άνθρωπο, έναν άριστο επιστήμονα, ένα μεγάλο δάσκαλο, ένα φίλο, τον Γιάννη Βρότσο.

Οι συγγενείς και φίλοι που τον συνόδευσαν στην τελευταία του κατοικία ήξεραν την αγάπη του για ζωή , το θάρρος του απέναντι στο θάνατο. Στην αγαπημένη του γυναίκα Σοφία και στη μικρή τους κόρη Ισαβέλλα αφιερώνουμε με συγκίνηση αυτή την κατάθεση τιμής, αυτά τα λίγα λόγια για τον υπέροχο αυτό άνθρωπο.

Ο Γιάννης Βρότσος γεννήθηκε στον Άγιο Νικόλαο Λακωνίας στη Μάνη και σπούδασε μαθηματικά στο Πανεπιστήμιο Αθηνών.

Υπήρξε μόνιμος επιστημονικός συνεργάτης της ΕΜΕ για θέματα Πληροφορικής και στυλοβάτης της διοργάνωσης της Α' Ολυμπιάδας Πληροφορικής στην Αθήνα. Εργάστηκε σχεδόν αδιάκοπα για 25 χρόνια προσφέροντας μεγάλο έργο στην ΕΜΕ.

Μόνιμος επιστημονικός συνεργάτης και καθηγητής των σεμιναρίων πληροφορικής της ΕΜΕ, έδωσε πνοή στην πρωτοπόρα προσπάθεια της Εταιρείας. Χιλιάδες ώρες διδασκαλίας από τον ακούραστο παιδαγωγό, δάσκαλο και επιστήμονα, τον βαθύ γνώστη της επιστήμης της πληροφορικής, στήριξαν επί σειρά ετών τα τμήματα Πληροφορικής της Ελληνικής Μαθηματικής Εταιρείας. Υπήρξε ο ακρογωνιαίος λίθος κάθε δραστηριότητας της ΕΜΕ σε θέματα πληροφορικής, όταν ακόμη ήταν ένας κλάδος όπου ελάχιστοι επιστήμονες είχαν τις δικές του γνώσεις και κατάρτιση .

Η διδακτική του καινοτομία, η αγάπη του για τα έξυπνα μαθηματικά των Η/Υ, οι ευφυείς ιδέες του έγιναν το κίνητρο για πάρα πολλούς νέους μαθηματικούς, σε δύσκολες εποχές ώστε να προχωρήσουν με θάρρος και βεβαιότητα σε ανώτερες μεταπτυχιακές σπουδές.

Σε αυτό τον ήρεμο, χαμογελαστό, ευχάριστο, μεθοδικό άνθρωπο και εκπαιδευτικό, με τον οξύ νου και το διεισδυτικό βλέμμα, που γοήτευε το ακροατήριό του με την ικανότητα να προλαβαίνει κάθε απορία, κάθε ερώτηση, που κρατούσε ζωντανό το ενδιαφέρον του κάθε μαθητή, που ήξερε να ενθαρρύνει με μια λέξη, ένα χαμόγελο, που μετέδιδε την αγάπη για τη γνώση με μια σπάνια απλότητα, οφείλεται το γεγονός ότι αμέτρητοι μαθηματικοί και άλλοι νέοι επιστήμονες παρακολούθησαν τα μαθήματά του και πέτυχαν στην επαγγελματική τους σταδιοδρομία, προχώρησαν με σιγουριά και έγιναν καθηγητές σε σχολεία και φροντιστήρια, στελέχη σε Τράπεζες, ΔΕΚΟ, Υπουργεία και σε πολλές μεγάλες εταιρείες του ιδιωτικού τομέα.

Όσοι είχαν την τύχη να είναι συνεργάτες του, όσοι ευτύχησαν να είναι φίλοι του, όσοι ευλογήθηκαν από τη διδασκαλία του θα τον θυμούνται πάντα με αγάπη .

ΕΥΚΛΕΙΔΗΣ Β ' 69 τ.l/53

Μαθηματικά για την Β ' Λυκείου

Μαθη ματικά κατεύθυνση ς Γενικά Θέματα Διανυσμάτων

Γκίνης Παναγιώτης - Χριστιάς Σπύρος

Η προέλευση της έννοιας του διανύσματος δεν έχει γίνει μέχρι σήμερα γνωστή. Εικάζεται ότι η πρώτη εμφάνισή

της οφείλεται σε ένα πόρισμα της εργασίας «Principia;> του /saac Newton, το 168 7. Στην εργασία αυτή εξετάζονται

εκτενώς κάποιες διανυσματικές ποσότητες (ταχύτητα, δύναμη κ. α.) αλλά όχι ακριβώς η έννοια του διανύσματος με τη

σημερινή της μορφή.

Η συστηματική μελέτη και η χρήση των διανυσμάτων αναπτύχθηκε ραγδαία στις πρώτες δεκαετίες του 19ου αιώνα,

μέσω των γεωμετρικών αναπαραστάσεων των μιγαδικών αριθμών. Βασικοί εμπνευστές αυτών των εργασιών ήταν οι

Carl Friedrich Gauss (1 777 - 1855), Wίlliam Rowan Hamilton (1805 - 1865), Joseph Louis Lagrange (1 736 -1813), Pierre Simon Laplace (1 749 - 182 7) και κυρίως ο James Clerk Maxwell (1831 - 18 79), ο οποίος

ταξινόμησε τις φυσικές ποσότητες σε μονοδιάστατες και διανυσματικές.

Η ανάπτυξη του Δ ιανυσματικού Λογισμού συνεχίσθηκε μέχρι τις αρχές του 200" αιώνα με την εργασία του Willard

Gibbs (1839 - 1903) στη θερμοδυναμική και τη διατριβή του Jean Frenet (1816 - 1990) στη θεωρία των

διαστημικών καμπυλών, η οποία περιέχει τους τύπους Frenet - Serret (γνωστό ως πλαίσιο τ.Ν.Β.). Σήμερα τα διανύσματα αποτελούν τη σύγχρονη γλώσσα της Φυσικής και των Εφαρμοσμένων Μαθηματικών, αλλά

ταυτόχρονα συνεχίζουν να κρατούν το εγγενές μαθηματικό ενδιαφέρον τους . .

Α. Ε Ρ!Η Η Σ Ε Ι Σ l:Ωl:TOV ΛΑΘΟVΣ

I . .Εάν ι α ι = ι β ι τότε α = β .

2 . Οποιαδήποτε διανύσματα α, β για τα οποία ισχύει; ια ι = ι β ι είναι ομόρροπα.

3 . Τα αντίθετα διανύσματα είναι αντίρροπα.

4 τ δ , α β , , , ' . α ιανυσματα ι αι ' ιβ ι εχουν ισα μετρα.

5. Για οποιαδήποτε διανύσματα α, β , γ αν το διάνυσμα α + β είναι συγγραμμικό με το διάνυσμα γ

τότε και τα διανύσματα α, β είναι συγγραμμικά με το διάνυσμα γ . 6. Δυο αντίθετα διανύσματα έχουν ίσα μέτρα και αντίθετους συντελεστές διεύθυνσης. 7 . Αν IΑΒ I = ψ8 - ψΑ τότε ΑΒ // ψ 'ψ .

8. Αν α = (3, 5) τότε α = 3i + 5j .

9 . Ισχύει ότι α- = α = α--? ι- ι 2 ι- ? ι

- 2 - ι β ι ι ο. Ισχύει η ισότητα : �' �I � ια ι για οποιαδήποτε διανύσματα ;, β με ; * ο και β * ο .

- -2 -1 1 . Ισχύει η ισότητα: α(�2

γ) = α · γ για οποιαδήποτε διανύσματα �' β, ; , με β =F Ο .

β

1 2 . Εάν α = (λ, l) και β = (l, μ) τότε συνθ= � λ + μ , όπου θ η γωνία των δυο διανυσμάτων. (λμ - 1 )2 + (λ + μ)2

Α Π ΑΝ Τ Η Σ Ε Ι Σ Σ τ J Σ Ε ΡΩΤ Η Σ Ε Ι Σ ΣΩΣΊΌ V - ΛΑΘΟΥΣ

I : I : I : I : I : I : I : I : I : I : I � I � ΕΥΚΛΕΙΔΗΣ Β ' 69 τ. l/54

Μαθη ματικά για την Β ' Λυκείου

\-:� 'Ε :Η Λ � " : Δίνεται τρίγωνο ΑΒΓ, Μ το μέσο της πλευράς ΑΒ και Δ, Ε δυο σημεία του επιπέδου του τριγώνου, τέτοια ώστε Μ μέσο του ΔΕ. Να

- - - -

αποδείξετε ότι: α) Γ Α + ΓΒ = Γ Δ + ΓΕ . IJ ) Για τυχαίο σημείο Κ του επιπέδου ισχύει

- - - -

ότι : ΚΑ + ΚΒ = ΚΔ + ΚΕ . Λ Y :l: i1 :

Α

Ε

ω Είναι ΓΔ + ΓΕ = ΓΑ + ΑΔ + ΓΒ + ΒΕ = = Γ Α + ΓΒ + Μ + ,Β'Ε = Γ Α + ΓΒ

(Στο τετράπλευρο ΑΔΒΕ οι διαγώνιοι διχοτομούνται. Άρα το τετράπλευρο ΑΔΒΕ είναι παραλληλόγραμμο και έτσι τα

--> -->

διανύσματα ΑΔ και ΒΕ είναι αντίθετα) . ΊΓ !1ΨJ !1 02.: Β ' : ΓA+ffi=2ΓM και ΓΔ+ΓΕ=2ΓΜ αφού Μ μέσο ΑΒ και ΔΕ. Επομένως Γ Α + ΓΒ = Γ Δ + ΓΕ ,

β ) Για το τυχαίο σημείο Κ του επιπέδου θα έχουμε ότι : - - - - - - - - - -

ΚΑ+ ΚΒ = ΚΔ + ΚΕ <:::::>ΚΑ-ΚΔ = ΚΕ - ΚΒ <:::::>ΔΑ= ΒΕ που ισχύει από το α) ερώτημα.

Δίνονται τα σημεία του επιπέδου Α, Β, Γ, Δ, Ε και Ο σημείο αναφοράς. Επίσης δίνεται ότι:

- - - - - 1 - 4 -ΟΑ = α, ΟΒ = β, ΟΓ = - α + -β,

5 5 - 2 - 1 - - 5 - 3 -ΟΔ = 3 α + 3 β, 0Ε = 2 α - 2 β

Να αποδείξετε ότι τα σημεία Α, Β, Γ, Δ, Ε είναι συνευθειακά. /\ Υ Π-Ι : Έχουμε ότι : ΑΒ = ΟΒ - ΟΑ = β - α - - - 1 - 4 - - 4 - 4 -ΑΓ = ΟΓ - ΟΑ = -α + -β - α = --α + -β =

5 5 5 5 4 - - 4 -= - (β - α) = -ΑΒ 5 5

- - - 2 - Ι - - Ι - Ι -ΑΔ = ΟΔ - ΟΑ = -α + -β - α = --α + -β = 3 3 3 3

Ι - - 1 -= )(β - α) = )ΑΒ - - - 5 - 3 - - 3 - 3 -ΑΕ = ΟΕ - ΟΑ = -α --β - α = -α --β = 2 2 2 2

= -lcβ - a) = -lAB 2 2

Άρα τα διανύσματα ΑΓ, ΑΔ, ΑΕ γράφονται σαν γραμμικός συνδυασμός του ίδιου διανύσματος ΑΒ , και επομένως τα σημεία Α, Β, Γ, Δ, Ε είναι συνευθειακά. Θ Ε Μ Α 3" : Δίνονται τα σημεία Α(4, 1 ) και Β(2,5). ιι) Να προσδιορίσετε σημείο Γ έτσι ώστε το

τρίγωνο ΑΒΓ να είναι ορθογώνιο και ισοσκελές.

β) Εάν Γ και Γ τα 2 σημεία που προκύπτουν από το ερώτημα α), να βρεθεί το μέτρο του

διανύσματος ΜΝ , όπου Μ το μέσο της ΒΓ και Ν το μέσο της ΒΓ και να δειχθεί ότι τα τετράπλευρα ΜΝΓ Α και ΜΝΑΓ είναι παραλληλόγραμμα.

Λ Υ Σ Η : α) Είναι ΑΒ= (-2,4) οπότε �� =vf4+ 16 =5o =2.fs.

Έστω το σημείο Γ(χ0 , ψ0 ) . Τότε ΑΓ = (χ 0 - 4, ψ0 - 1) με

IAΓ I = �( χ 0 - 4)2 + (ψ0 - Ι )2 .

Επίσης λ Ai3 = _i = -2 . 2

Πρέπει λ - · λ . = - Ι . <:::::> λ . = _!_ . ΑΓ ΑΒ ΛΓ 2

Ε , λ Ψο - Ι 4 Ά ιναι Ar' = -- , χ 0 ;;r:. • ρα

χ0 - 4 Ψο - Ι Ι -- =- <=:>2ψ0 -2= χ0 -4<=:>χ0 -2ψ0 =2 (1) . χ0 -4 2 Επίσης πρέπει IAΓI = J20 ή

�( χ ο - 4)2 + (Ψο - 1 )2 = J20 <:::::>

<:::::> ( Χ 0 - 4)2 + (Ψ ο - 1 )2 = 20 <:::::> <:::::> χ 02 - 8χ0 + Ι 6 + ψ02 - 2ψ0 + Ι = 20 <:::::> <=:> Χο2 + Ψο2 - 8χο - 2Ψο - 3 = 0 (2) Η λύση του συστήματος των ( Ι ) και (2) δίνει Γ(8,3 ) ή Γ(Ο,- Ι ) .

β ) Στο τρίγωνο ΑΒΓ με Μ(χ ι , Ψ ι ) μέσο της ΒΓ θα έχουμε ότι

8 + 2 3 + 5 ' Χ ι = -- = 5 και Ψι = -- = 4 δηλαδη Μ(5,4) . 2 2

ΕΥΚΛΕΙΔΗΣ Β ' 69 τ. l/55

Μαθηματικά για την Β ' Λυκείου

Στο τρίγωνο ΑΒΓ με Ν(χυ ψ2 ) μέσο της ΒΓ θα έχουμε ότι

0+ 2 -1 + 5 ' χ, =- = Ι και ψ7 =-- =2, δηλαδη Ν( 1 ,2) . - 2 - 2 'Ετσι: ΜΝ = (-4, -2) και jMNj = .J1 6 + 4 =

=fiδ =215 και λΜΙ'ί =� οπότε l\1N// ΑΓ και l\1N// ΑΓ' 2 και αφού έχουν ίσα μέτρα, τα ζητούμενα τετράπλευρα είναι παραλληλόγραμμα.

Θ Ε \ Ι Α 4' ' : - -Έστω τα διανύσματα α και β με ial = .J3 και l ί3 l = J6 . Α ν για οποιουσδήποτε

πραγματικούς αριθμούς μ, ν ισχύει ότι: - - - 1 -

(μα + ν β) .l (να - -μ β) , να υπολογισθεί το 2

- 1 - .J3 -μέτρο του διανύσματος u = -α + -β. 3 2

Λ Υ Π 1 : Αφού (μa + νβ) .l (νa - .!.μβ) έχουμε: 2 - - - Ι -(μα + νβ) · (να - -μβ) = Ο 2 -2 1 2- - ,- - Ι -2 <=>μνα --μ α· β+Vα· β--μνβ =0<=> 2 2

<=>μνl92 +( J -�μ2 Jα. β-�μv��2 =Ο<=>

<=>� # +( J -�μ2 Jα· ί3-��6=ο<=>( J -�μ2 Jα· ί3=ο Αφού η ισότητα ισχύει για κάθε πραγματικό αριθμό ν και μ θα πρέπει α · β = Ο . Έτσι έχουμε ότι

- -α } Δίνονται τα διανύσματα α=(2,-1) ιαιι β=(-2,5). - - - - - -Εάν 3χ+2ψ=α και 5χ+6ψ=β να βρεθεί το

συνημίτονο της γωνίας των διανυσμάτων - -χ και ψ. β ) Δίνονται τα διανύσματα u = λ χ + 2ψ και

; = 2� - λψ . Να βρεθεί η τιμή του λ ε �*+

ώστε τα διανύσματα u , ν να είναι κάθετα.

α) Λύνουμε το σύστημα {3� + 2� = � <=> {-�� -�Ψ �-3α . 5χ + 6ψ = β 5χ + 6ψ = β Προσθέτουμε κατά μέλη και έχουμε : -- - - - --4χ =-3α+β<=>-4χ = (-6,3) + (-2, 5) <=>χ = (2,-2)

και αντικαθιστώντας στην πρώτη εξίσωση

β ' ' - ( 2 5 ) Ά ' ρισκουμε οτι ψ = - ,- . ρα εχουμε : 2 - Λ - χ 0 ψ -4 - 5 -9 -9J82

συν( χ , ψ) � I " I IΨ I � -Γsf{ .J82 � 82

β } Είναι : u = (2λ, -2λ) + (-4, 5) = (2λ - 4, -2λ + 5) και - 5 -5 ν = (4, -4) - (-2λ,2λ) = (4 + 2λ, -4 -2λ) . Πρέπει u · ν = Ο

5 <=> (2λ - 4, -2λ + 5) ο (4 + 2λ, -4 - 2λ) = ο <=> 5 <=>(2λ-4) · (4+ 2λ) + (-2λ+5)(-4--λ) = 0<=> 2

<=>18λ2 -9λ-72 = 0<=>2λ2 -λ-8 =0<=> <=> \ = 1 + � Δεκτή λz = Ι -� Απορρίπτεται.

Θ ι;;: νι Α Δίνονται τα διανύσματα a, β, y με ια ι = 2,

l ί3 1 = 4 και lr 1 = 3 και (a ΛΙi) = 34π = {ί3 Λ Ύ) ενώ {Ύ Λ �) = � . Να βρεθεί η γωνία των

- - - - - -διανυσμάτων α+ β-γ και 2α+ β-2γ . . '� ι :ε w-ι Έστω (a+ β-:Υ ' 2a+ β-2:Υ) = θ . Είναι - - -- - - -(α+ β-γ) · (2α+ β-2γ) , συνθ I - - ι ι - -ι ο

Υπολογιζουμε τους α+ β-γ . 2α+ β-2γ όρους του κλάσματος.

(a+ β-Ύ) · (2a+ β-2:Υ) = 2 ιαι2 + aί3 - 2a:Y + I ί31 2 + +2aί3 - 2ί3:Υ - 2αγ - ί3Ύ + 2 Ι:Υ I 2 = = 8 + 3αβ - 4αγ - 3βγ + 1 6 + 1 8 = = 42 + 3 \a jί3 j \ συν 3; - 4 \a j:Υ j \ συν% - 3 \ί3 j:Υ j \ συν 3; =

� 42 + 3 j 4 [ -1) - 3 ,} { -1) �

ΕΥΚΛΕΙΔΗΣ Β ' 69 τ. l/56

Μαθη ματικά για την Β ' Λυκείου

= 42 - 12.J2 + 1 s.J2 = 42 + 6h. Ιa+ί3-Ύ 12 =1�2 +1�2 +1�2 +2αβ-2αγ-2βy=

3π π 3π =4+16+9+2 · 2 · 4·συν--2· 2 · 3 ·συν- -2 - 4 · 3 ·συν- = 4 2 4 =21+1{ �)-2{ �)=21+if2

Επομένως l a + β - ΎI = �29 + 4h. l:htί3-2Y 12 =��2 +1�2 +��2 +4αβ-&χγ-4βγ=

3π π 3π =4·4+16+36+4·2·4 ·συν--8·2 · 3 ·συν--4-4- 3 ·συν-= 4 2 4 =68-16f2+2if2=68+sJ2. 12CX+β-2Y 1 =�68+sJ2. 'Ετσι

3 /1(7+!2)

---> ---> --->

Δίνονται τα διανύσματα α = (2, - 4) και β ::;:. Ο

για τα οποία ισχύει ότι ι�- βl = 6 και

1 2 �+ 3 i3Ι = 4Μ . Ιi!( ) Ν α δειχθεί ότι α _l β και να βρεθεί το l β l .

Να βρεθεί το διάνυσμα της προβολής του - - -

διανύσματος ν = 2α - 3β πάνω στο

διάνυσμα α και το διάνυσμα της προβολής - -

του διανύσματος ν πάνω στο διάνυσμα β .

γ ) Ν α βρεθεί τ ο διάνυσμα β . . \ Υ Σ Η : J ) Είναι la l = �22 + (-4)2 = J20 = 2J5 . { Ι α - β 1 2 = 36 {α2 - 2aβ + β2 = 36 Ι2a + 3β1 2 = 224 Q 4a2 + 1 2aβ + 9β2 = 224 Q

{2ο -2aβ+ β2 = 36 {-2aβ+β2 = 16 Q Q Q sο+ Ι 2aβ+9β2 = 224 12aβ +9β2 = 144 { Ι sαβ -9β2 = -144

QΠΟλ/με την πρώτη με (-9) _ _ _ 2 12αβ+9β = 144 Προσθέτουμε κατά μέλη και έχουμε:

30αβ = 0Qαβ= 0Qα.lβ

- -(Αφού κανένα από τα α και β δεν είναι

μηδενικό) . Άρα l β l 2 = 36 - 20 = 1 6 Q l β l = 4 . β) Ισχύει ότι προ β- ν // α . Άρα υπάρχει λ Ε IR

α - - - - - -ώστε προβ . ν = λα. Όμως α · ν = α · προβ- ν

α α -2 - - -άρα α · ν = λα έτσι α · (2α - 3β) = λ · 20

-2 ή 2α - 3αβ = 20λ ή τελικά λ = 2 . Επομένως προ β- ν = 2a = ( 4, -8) . Επίσης

α

προ β β ν // β άρα υπάρχει κ Ε IR ώστε - - - -2 προββ ν = κβ Όμως β · ν = προββ ν. άρα β · ν = κβ

έτσι β . ν = Ι 6κ ή β(2a - 3β) = 1 6κ ή - 2 2αβ - 3β = 1 6κ ή τελικά κ = -3 .

Δίνεται τρίγωνο ΑΒΓ και σημείο Ο του επιπέδου του τριγώνου, για το οποίο ισχύει ότι ioΛI = ΙοΒΙ = Ιorl = ι και ΟΑ+ ΟΒ+ or = ο . ο) Να υπολογισθεί η γωνία που σχηματίζουν τα

---> --->

διανύσματα ΟΑ και ΟΒ . β ) Να δειχθεί ότι το τρίγωνο ΑΒΓ είναι ισόπλευρο.

� � � � � ____, �

Έχουμε ότι ΟΑ+ ΟΒ+ ΟΓ = Ο Q ΟΑ+ ΟΒ = -ΟΓ . τότε ΙοΛ+ ο8 1 = 1-or l = Ιor l . 'Ετσι

ΙοΛ+ ο 1 { = loΓ I 2 Q lo: { + lο ϊ { + 2οΛο8 = Ιοr Ι 2 ---> ---> ---> ---> 1

ή 1 + 1 + 20ΑΟΒ = Ι Q ΟΑΟΒ = -- . Ομοίως θα 2 ΕΥΚΛΕΙΔΗΣ Β ' 69 τ. l/57

Μ αθηματικά για την Β ' Λυκείου

---> ---> I ---> ---> Ι πάρουμε ότι ΟΑΟΓ = -- και ΟΒ ΟΓ = -- .

2 2 α) Έστω θ η ζητούμενη γωνία. Τότε

ioΛi loBI συνθ= -� ή συνθ= -�=συνΙ 20°.

Άρα η ζητούμενη γωνία είναι ίση με Ι 20°. f) ) Έχουμε ότι

IΆBI2 = IOB-OAI2 = IOBI2 + 10AI 2 - 20ΑΟΒ =

= I + I - 2(-�) = 3 . Άρα IABI =.J3 . Ομοια θα

πάρουμε ότι μ-rι = IBΓ I = .J3 . Άρα το τρίγωνο

είναι ισόπλευρο. 0 [ \I A 9" :

Δίνεται τρίγωνο ΑΒΓ με IAΉI = 4 , IAΓI = 6 , Μ

το μέσο της πλευράς ΒΓ και η γωνία μεταξύ ---> ---> π

των διανυσμάτων ΑΒ και ΑΓ ίση με - . 3

--->

υ) Να βρεθεί το μέτρο του διανύσματος ΑΜ .

β ) Να βρεθεί το μέτρο της προβολής του ---> --->

διανύσματος ΑΒ πάνω στο διάνυσμα ΑΜ .

Λ Υ Σ Η : ___, Ι ___, ___,

α ) Έχουμε ότι ΑΜ = - (ΑΒ+ ΑΓ) . 2

'Ετσι IM1 =� IAB+ ΑΓI �ΙΜ12 =� IAB+ 42 =

=�<IABI2 + IArl2 + 2ABAr) =� (l6 + 36 + 24) = 1 9 .

Άρα παίρνουμε ότι ΙΑΜ Ι = Jl9 . ---> β ) Η προβολή του διανύσματος ΑΒ πάνω στο

--->

διάνυσμα ΑΜ είναι διάνυσμα συγγραμμικό --->

με το διάνυσμα ΑΜ . ---> --->

Άρα θα έχουμε ότι προ β � ΑΒ = λ ΑΜ . ΑΜ

� � ---4- �

Ομως γνωρίζουμε ότι ΑΜΑΒ = ΑΜ προβ � ΑΒ .

---> --->

ΑΜ ---> 2 Έτσι προκύπτει ότι ΑΜ ΑΒ = λΑΜ ή

Ι ___, ___, ___, Ι ---> 2 ___, ___,

-(ΑΒ+ΑΓ)ΑΒ = Ι9λ ή - (ΑΒ + ΑΓΑΒ) = Ι9λ ή 2 2

_!_ ( Ι 6 + 1 2) = 1 9λ ή τελικά λ =

Ι 4 .

2 Ι 9

Θ F: Μ Α 10" : Έστω τρίγωνο ΑΒΓ, Μ το μέσο της πλευράς ΒΓ, Δ το μέσο της πλευράς ΑΓ, Ε το μέσο της πλευράς ΑΒ και G το βαρύκεντρο του τριγώνου.

---+ --7 --7 -+

u.) Να δειχθεί ότι GA+ GB+ GΓ = Ο .

I � -> ι � -> ι �ι --> ---> β) Να δειχθεί ότι B .. I GA+ .ru (;θ+ ιGΓ=Ο�

Το τρίγωνο ΑΒΓ είναι ισόπλευρο. Λ Yl:H :

--7 --7 --7 --7 --7

α) Έχουμε ότι GA+ GB+ GΓ = GA+ 2GM (Αφού Μ μέσο του ΒΓ). Από την ιδιότητα του

---> --->

βαρύκεντρου γνωρίζουμε ότι AG = 2GM . Έτσι προκύπτει ότι :

--7 --7 --7 --7 --7 --+ --7 --7

GA+ GB+ GΓ = GA+ 2GM = GA+ AG = Ο .

β ) 'Εστω ΙΒr Ι = λ, ΙΑrΙ = μ, I ABI = κ . Λόγω του

---> ---> --->

ερωτήματος α) θα έχουμε ότι GΓ = -GB- GA .

Τότε ΒΓ GA+ ΑΓ GB+ ΑΒ GΓ = Ο � I -> I -> I -> I -> I -> I -> -->

--7 -+ -+ -+ --+

λ GΑ+ μGΒ+ κ(-GA- GB) = Ο � --> --> -->

� (λ - κ) GΑ+ (μ - κ) GΒ = Ο . --> -->

Ομως τα διανύσματα GA και GB δεν είναι συγγραμμικά. Άρα πρέπει λ-κ=Ο και μ-κ=Ο. Έτσι προκύπτει ότι κ=λ=μ. Άρα το τρίγωνο ΑΒΓ είναι ισόπλευρο. Αντίστροφα : Έστω ότι το τρίγωνο είναι

ισόπλευρο. Τότε ΙΒr Ι = IAr l = IΆBI = α > Ο .

Έτσι θα έχουμε ότι

IB� GA+ I4 GΪ3+ μ�� Gr = α(GA+GB+Gn = α Ο = ο -+ -+ -+ -+

( GA+ GB+ GΓ = Ο λόγω του α) ερωτήματος) .

ΕΥΚΛΕΙΔΗΣ Β ' 69 τ. l/58

Μαθη ματικά για την Β ' Λυκείου

-->

Δίνονται τα διανύσματα α = (2, 1) και --> -->

β = (1 , - 1) . Να βρεθεί διάνυσμα γ συνεπίπεδο --> ....

των διανυσμάτων α και β για το οποίο ισχύουν --> ....

τα ακόλουθα: i) γ l. α , ii) Σχηματίζει αμβλεία

γωνία με το διάνυσμα β και iii) 1; 1 = �� � . ....

Το διάνυσμα γ θα γράφεται κατά μοναδικό τρόπο -->

ως γραμμικός συνδυασμός των διανυσμάτων α -->

και β . Άρα υπάρχουν μοναδικοί λ, μ ε iR τέτοιοι � � ---1> �

ώστε γ = λ α+ μ β . Έτσι παίρνουμε ότι γ =λ(2, --> ....

1 )+μ( 1 , - 1 )=(2λ+μ, λ-μ) . Αφού γ l. α θα έχουμε : --> -->

α γ = Ο . Άρα προκύπτει ότι : 4λ+2μ+λ-μ=Ο ή

5λ+μ=Ο ( 1 ) . Επίσης : 1; 1 =Η από όπου έχουμε:

)C2λ+μ)1 + (λ-μ)1 = J5 � .. . �5λ2 +2μ2 +2λμ=5 (2) .

Το σύστημα των ( 1 ) και (2) δίνει ως λύση τα ζεύγη

(λ, μ)= ( l , - %) ή (λ, μ)= ( -1 , %) . 'Ε τ σι ; =( - 1 , 2)

--> -->

ή γ =( 1 , -2 ) . Αφού το διάνυσμα γ σχηματίζει -->

αμβλεία γωνία με το διάνυσμα β θα πρέπει να --> --> -->

ισχύει ότι β γ < Ο . Όταν γ =( - 1 , 2) έχουμε ότι --7 --7 --7 ---t ---t

β γ = -3 ενώ όταν γ =( ! , -2) τότε β γ = 3 . Άρα -->

δεκτή λύση είναι το διάνυσμα γ =( - 1 , 2) .

Δίνονται τα σημεία Μ(χ, y) του επιπέδου για τα

οποία ισχύει ότι loi{ = 3 + 2y (όπου Ο η αρχή

των αξόνων). ιι ) Να βρεθεί ο γεωμετρικός τόπος

των σημείων Μ. β) Δίνονται τα σημεία Α(Ο, -1 ) και Β(Ο, 3) . Να υπολογισθεί η τιμή της

παράστασης Ρ= IAMI2 + IBi{ Δίνεται το

σημείο Γ(-3, 5). Να δειχθεί ότι 3 � lrM:I � 5 .

, :. Yl: !-� :

α) 'Εχουμε ότι ΙοΜ Ι 2 = 3 + 2y => χ2 +/ =3 +2y=>

=> χ2 + (y - 1/ = 4 => �χ2 + (y - 1)2 = 2 .

Όμως η ποσότητα �χ2 + (y - 1)2 αποτελεί το -->

μέτρο του διανύσματος ΚΜ όπου Κ είναι το σταθερό σημείο με συντεταγμένες (0, 1 ). Δηλαδή έχουμε ότι lκMI = 2 . Άρα το σημείο

Μ κινείται σε κύκλο με κέντρο το σημείο Κ( Ο, 1 ) και ακτίνα ρ=2 όπως φαίνεται και στο παρακάτω σχήμα :

--:--�-

_ ,_ I

--:' . Ι .. � : I Ι<.

Κ(Ο . 1 )ι

8(0. 3)

Μ(χ. γ)

- �-- - . - - ----,Γ�ιο. -1 1 � I

β } Τα σημεία Α(Ο,- 1 ) και Β(Ο,3 ) είναι aντιδιαμετρικά σημεία του κύκλου του ερωτήματος α) . Αφού τα σημεία Μ(χ, y) είναι σημεία του παραπάνω κύκλου θα έχουμε ότι το τρίγωνο ΒΜΑ είναι ορθογώνιο στο σημείο

Μ. 'Ετσι IM12 + �Β�2

= 142 =42 = 16 . Άρα P= l 6 .

--> --> -->

γ ) Ισχύει ότι ΓΜ = ΓΚ+ ΚΜ . Έτσι λόγω τριγωνικής ανισότητας θα έχουμε ότι

l lrκl - lκMII � Ιrκ + κΜI = lrM:I � lrκl + lκMI . Όμως rX=(3, -4) . 'Ετσι lrRJ =s ενώ lκMI = 2 .

Άρα ι s - 2 1 � ΙΓΜ Ι � 5 + 2 ή τελικά 3� ΙΓΜJ :::; 7 .

ΕΥΚΛΕΙΔΗΣ Β ' 69 τ. l/59

Μ81ΙΙJΙ8-ιι8ai ,.. I'� r 1'�1'1 l'•rι Α••Ι•rι

Ασκή σεις Γενικής Παιδείας Γ Λυκείου

Άσκηση J 'l Δίνεται η συνάρτηση f ( χ)= ln2 χ . ι) Να βρείτε το πεδίο ορισμού της f.

2) Να βρείτε το ρυθμό μεταβολής της f ως , ι

προς χ , οταν χ = - . e

3) Να βρείτε την εφαπτομένη στην καμπύ­

λη της f στο σημείο που έχει τεταγμένη ι και τε­

τμημένη μικρότερη της μονάδας.

Λ Υ Σ Η 1 . Για να ορίζεται η f ( χ ) πρέπει και αρκεί χ > Ο .

Άρα το σύνολο ορισμού της είναι το Α = (Ο, +οο) . 2 . f' (x ) = ( ln2 x)' = 2ln x · ( ln x)' = 2 ln x · _!_ = 2 ln x

χ χ

( ) 2 1η _!_ f' ± = _!_ e = 2e · ( - 1 ) = -2e

e 3 . f ( χ ) = 1 <=> !η 2 χ = 1 <=> !η χ = 1 ή

ι 1

' 1 1 , 1 η χ = - <::::> x = e η χ = - <::::> χ = - , αφου - < 1

και e > 1 .

e e e

Η εφαπτομένη είναι y = λχ + β με

λ = f ' (±) = -2e άρα y = -2ex + β .

Για να διέρχεται από το σημείο ( ± , 1) πρέπει

και αρκεί 1 = -2e_!_ + β δηλαδή β = 3 . e

Άρα y = -2ex + 3 .

Αν. Φανέλη

Άσκηση 2'�

Δ , , r ( ) xz + 4χ ινεται η συναρτηση χ = . χ + 3

ι) Να βρείτε το πεδίο ορισμού της f.

2) Να βρείτε τα όρια:

lim f (x) και lim f{x)

χ--+3 χ--+0 χ 3) Αν χ > -3 να αποδείξετε ότι η f δεν έχει

ακρότατα.

ΛΥΣΗ 1 . Για να ορίζεται η f (χ) πρέπει και αρκεί

χ + 3 * Ο δηλαδή χ * -3 . Άρα το σύνολο ορισμού της είναι το Α = ( -οο, -3) υ ( -3, +οο) .

2 . lim (χ + 3) = 6 i:- O, lim ( x2 + 4χ ) = 32 + 4 · 3 = 2 1 χ --)> 3 χ --7 3

, 1 . f ( ) 2 1 7 οποτε ιm χ = - = -

χ --+ 3 6 2 χ2 + 4χ χ + 3 χ 2 + 4χ χ ( χ + 4) χ + 4

χ χ = χ ( χ + 3) = χ ( χ + 3) - χ + 3 lim( χ + 3) = 3 * Ο, lim( χ + 4) = 4 άρα lim f ( χ ) = � . χ....Ο χ....Ο χ-->0 χ 3 3 . f' ( x ) = ( x2 + 4χ )' = χ + 3

- (χ2 + 4χ)' ( χ + 3) - (χ2 + 4χ) ( χ + 3)' - (χ + 3)2 ( 2χ + 4) ( χ + 3) - ( χ2 + 4χ ) ( 1 + 0)

(χ + 3 )2 2χ2 + 6χ + 4χ + 1 2 - χ2 - 4χ

( χ + 3 )2

=

ΕΥΚΛΕΙΔΗΣ Β ' 69 τ. l/60

Μαθη ματικά για την Γ Λυκείου

χ2 + 6χ + 1 2 0 2 > ( χ + 3)

Αφού Δ = 62 - 4 · 1 2 = 36 - 48 = - 1 2 < Ο , άρα θα είναι χ 2 + 6χ + 1 2 > Ο για κάθε χ Ε IR οπότε f1 ( χ ) > Ο για κάθε χ > -3

Άρα f γνησίως αύξουσα στο (-3, +οο) , οπότε

οποιαδήποτε τιμή της f ( χ0 ) αποκλείεται να είναι

ακρότατο στο (-3, +οο) . �

η .ι ;ι · η ; αν -3 < Χ ι < χ0 τότε f( Χ ι ) < f( χ0 ) , δηλαδή το f ( χ0 ) δεν είναι ελάχιστο. Αν

χ0 < χ 2 τότε f ( χ0 ) < f ( χ 2 ) δεν είναι ούτε μέγιστο .

Έστω ότι η γραφική παράσταση της συνάρ­τησης f ( χ) = ax3 - 3χ2 + βχ - 2008, χ Ε R , διέρ-

χεται από το σημείο { 3,-1 999) και η εφαπτο­

μένη της στο σημείο (Ο, f {Ο)) έχει συντελεστή

διεύθυνσης 9. ι) Ν α βρεθούν τα α, β.

ι 2) Αν α = - και β = 9 να εξετάσετε την f

3 ως προς την μονοτονία και τα ακρότατα.

3) Ν α λυθεί η ανίσωση χ3 - - 3χ 2 + 9χ - 9 > Ο χρησιμοποιώντας τα συ-3

μπεράσματα του δεύτερου ερωτήματος.

Ι . f ( 3 ) = - 1 999 => α · 33 - 3 · 32 + β · 3 - 2008 = - 1 999 =>

27a - 27 + 3β - 2008 = - 1 999 => 27a + 3β = 36 => 9a + β = 1 2 ( 1 )

Έχουμε:

f' ( x ) = (ax3 - 3χ 2 + βχ - 2008)' = 3ax2 - 6χ + β οπότε f' (Ο) = β

Η εφαπτομένη στο χ0 = Ο έχει συντελεστή δι­

εύθυνσης 9 άρα f' (Ο) = 9 => β = 9 Οπότε ( Ι ) => 9a + 9 = 1 2 => 9a = 3 => a = .!. . 3

1 2 . Για a = - και β = 9 η συνάρτηση είναι 3 f ( χ ) = .!. χ3 - 3χ 2 + 9χ - 2008 ' 3 x E R και

f' ( x ) = x2 - 6χ + 9 , άρα f' ( x ) = ( x - 3)2 ;::: ο . 0-

πότε η f είναι γνησίως αύξουσα στο IR και δεν παρουσιάζει ακρότατα. Πράγματι αφού η f παρα­γωγίζεται στο ( -οο, 3] και f1 ( χ ) > Ο στο

( -οο, 3) θα είναι f γνησίως αύξουσα στο ( -οο, 3] . Επίσης αφού η f στο [3, +οο) και f1 ( χ ) > Ο στο

(3, +οο) θα είναι f γνησίως αύξουσα στο [3, +οο) . Άρα f γνησίως αύξουσα στο ( -οο, 3] u [3, +οο) = IR .

ΓΥ ροσοχι� : Α ν είχαμε f γνησίως αύξουσα στο ( -οο, 3] και στο (3, +οο) τότε δεν είναι σίγουρο

ότι f γνησίως αύξουσα στο ( -οο, 3] u (3, +οο) = IR (βλέπε σχήμα). Ομοίως όταν f γνησίως αύξουσα στο ( -οο, 3) και στο [3, +οο) ή όταν f γνησίως αύξουσα στο (-οο, 3) και στο (3, +οο) .

3 χ ? 3 . - - 3χ- + 9χ - 9 > 0 <=> 3 χ3 ? --3χ - + 9χ > 9 <::::> 3 χ3 ? -- 3χ - + 9χ - 2008 > 9 - 2008 <=> 3 χ3 ? -- 3χ - + 9χ - 2008 > - 1 999 <=> 3 f ( x ) > f (O) <=> χ > Ο , αφού f γνησίως αύξουσα.

Δίνεται η συνάρτηση

ΕΥΚΛΕΙΔΗΣ Β ' 69 τ. l /61

Μαθηματικά για την Γ Λυκείου

r ( x) = ημχ

, χ ε (ο,π) .

ημχ + συνχ 2

1 ) Να βρείτε το όριο lim f ( x ) π

x ---t-6

2) Να βρείτε την f' ( χ) .

3) Α ν χ ε (: , �) να δείξετε ότι f ( χ) > � . :\ \ ' �

1 Α . 1 . ( ) π π ι .J3 0 . φου ιm ημχ+συνχ =ημ- +συν- =- +- "i= '� 6 6 2 2

6

θα έχουμε 1 2 1 J3 - ι JJ - 1

lim f ( χ ) = J3 - -Η� 1 3 1 + J3 - 3=1 - -2-

- + -2 2 2 . f' ( x ) - [ ημχ ) ' -ημχ + συνχ

_ ( ημχ )' ( ημχ + συνχ) -ημχ ( ημχ + συνχ ) ' _ -

? -

( ημχ +συνχγ συνχ ( ημχ + συνχ ) - ημχ ( συνχ - ημχ )

( ημχ + συνχ )2 � + συν2χ� + ημ2χ

( ημχ + συνχ )2 1

( ημχ + συνχ γ . 3 . Αφού f' ( χ ) > Ο η f είναι γνησίως αύξουσα. J2 Άρα χ > �� f( χ) > f( � J � f( χ ) > J2 2 J2 �

J1 72 ι f ( x ) > � � f ( x ) > - . 2 J1. 2

2

- +-2 2

Ένα ορθογώνιο φύλλο χαρτιού έχει εμ­

βαδό 600cm2 • Τα περιθώρια πάνω και κάτω

είναι 4cm ενώ δεξιά και αριστερά είναι

3cm . Να βρείτε τις διαστάσεις του φύλλου

για τις οποίες η ωφέλιμη επιφάνεια είναι μέ­

γιστη.

Έστω χ , y οι διαστάσεις του φύλλου . Τό­

τε Ε = χ · y = 600 άρα y = 600 . Οι διαστάσεις χ της ωφέλιμης επιφάνειας είναι: α = χ - 6 και β = y - 8 οπότε το εμβαδό της θα είναι

Ε 1 = α · β = ( χ - 6) ( y - 8) = ( 600 J 3600 ( χ - 6) � - 8 = 600 - 8χ --χ- + 48 = 3600 -8χ --- + 648 , με χ > 6, y > 8 χ

Άρα:

χ

4 α 3

β Υ

E' ( x ) = -8 - 3600 (-�J = -8 + 36?0 = -8χ 2 + 3600 χ - χ - χ 2

, οπότε Ε' ( χ ) = Ο � -8χ 2 + 3600 = 0 � 8χ 2 = 3600 �

χ 6 Ε'( χ)

χ 2 = 450 � Ε(χ) χ = .J 450 , αφού χ > 6 και

+ 450 +ω ο

Ε1 ( χ ) > ο � ( χ -F450 ) ( x + F450 ) > Ο � χ > F450 ενώ Ε1 ( χ ) < Ο � χ < F450"

Άρα γίνεται μέγιστη για χ = F450" = 2 ι , 2 1

ΕΥΚΛΕΙΔΗΣ Β ' 69 τ. l/62

Μαθη ματικά για την Γ Λυκείου

Μιγαδικοί Αριθ μοί (γεωμετρικοί τόποι - μέγιστο ελάχιστο μέτρο - γεωμετρική

ερμηνεία) - Α ντίστροφη συνάρτηση και μονοτονία. Τσανής Νίκος - Πιτσάβας Θανάσης

Α. Λ ΥΜΕΝΑ ΘΕΜΑΤΑ - ΠΑΡΑΤΗΡΗΣΕΙΣ

Θέμα Ι ο

Έστω οι αριθμοί z ε C για τους οποίους ισχύει jz - 1 + i j = jz - 4i j (1) . Τότε να βρεθεί:

α. Ο γεωμετρικός τόπος των εικόνων Μ των

μιγαδικών z β. Η ελάχιστη τιμή του j z j γ. Ο μιγαδικός z με το ελάχιστο μέτρο δ. Έστω f(w)= jw - 1 + i j + jw - 4ί j . Τότε να

βρεθεί ο γεωμετρικός τόπος των μιγαδικών

w ώστε η παράσταση f(w) να παίρνει την

ελάχιστη τιμή.

ε. Από τους μιγαδικούς w του ερωτήματος (δ)

να βρείτε αυτόν για τον οποίο ισχύει jw - 1 + i j = jw - 4i j .

Λίιση α. Έστω z=xo+ψ0i , τότε Μ(χο ,ψ0) και: ( Ι ) <=> j z - ( Ι - i) j = j z - 4i j <=> (ΜΑ) = (ΜΒ) , όπου A( l ,- 1 ) Β(Ο,4) . Άρα ο γεωμετρικός τόπος των Μ είναι η μεσοκάθετος του ΑΒ. Εξάλλου ( Ι ) <=> j x 0 - 1 + (ψ0 - l) i j = = jx 0 + (ψ0 - 4)i j <=:>χο-5ψο+7=0, δηλαδή η μεσοκά-θετος του ΑΒ είναι η (ε) : χ-5ψ+7=0.

Β ψ

χ ' Α χ

ψ '

_ ι ο - s . ο + 7 1 7 d(Ο,ε)- r;:;-;:; r::::; . ν 12 + 52 ν26

γ. Ο μιγαδικός z με το ελάχιστο μέτρο έχει εικόνα το ίχνος της καθέτου από το 00,0) στη (ε) : , έστω Μι . 'Εχουμε λε=.!_ <=> λοΜ ι=-5 . Άρα: 0Μ ι :ψ=-5χ, 5 οπότε οι συντεταγμένες του Μ 1 επαληθεύουν το σύστημα: ψ = -5χ } ' 7 35 , δηλαδη χ=-- , ψ=- . χ - 5ψ + 7 = ο 26 26

Ά 7 35 . ρα z=-- +- ι 26 26

δ. Α ν Ρ η εικόνα του w τότε f(w)= jw - 1 + i j + jw - 4i j = (ΡΑ)+(ΡΒ) . Το άθροι-σμα των αποστάσεων (ΡΑ)+(ΡΒ) 2 (ΑΒ) παίρνει ελάχιστη τιμή μόνο όταν το Ρ είναι σημείο του ευ­θύγραμμου τμήματος ΑΒ ε. Εφόσον jw - 1 + ij = jw - 4ij , τότε η εικόνα του w ανήκει στη μεσοκάθετο του ΑΒ. Άρα η εικόνα του w είναι το μέσον Κ του ΑΒ, δηλαδή Κ( .!_ , l ) και

2 2 1 3 . w=- +- ι 2 2

Π αρατή ρη ση I Όταν ο μιγαδικός z κινείται σε ευθεία τότε, το ε­λάχιστο μέτρο είναι η απόσταση του 0(0,0) από την ευθεία, και ο μιγαδικός με το ελάχιστο μέτρο έχει εικόνα το ίχνος της καθέτου από το 0(0,0) στην ευθεία. Θέμα 2 Έστω μιγαδικός z για το οποίο ισχύει jz - 3 - 3i j =2 τότε να βρεθεί:

β. Εφόσον j z j =(ΟΜ), το ελάχιστο j z j είναι η από- α. Ο γεωμετρικός τόπος των εικόνων Μ των μι­

σταση του 0(0,0) από την (ε), δηλαδή η γαδικών z

ΕΥΚΛΕΙΔΗΣ Β ' 69 τ. Ι/63

Μαθηματικά για την Γ Λυκείου

β. Η ελάχιστη και μέγιστη τιμή του l z l γ. Οι μιγαδικοί z με το ελάχιστο και μέγιστο

μέτρο

δ. Αν για του μιγαδικούς z1 , z2 ισχύει

lz ι - 3 - 3i l = lz2 - 3 - 3i l =2 να βρεθεί η μέγιστη

τιμή της παράστασης lz ι - z2 1 ι\iιση α. Έχουμε: l z - 3 - 3i l =2 <=> Jz - (3 + 3i )j = 2 <=> (ΚΜ) = 2 <=> Μ ε ( c ) , όπου ( c ) ο κύκλος με κέντρο Κ(3 ,3) και ακτίνα ρ=2, δηλαδή ο (c) : ( χ-3)2 + (ψ-3)2 =4

I - �--- --�---� i I

I χ · ____ ο_Ψ_ - - χ - _j β. Εφόσον l z l =(ΟΜ) και I COK) - (KM)I � (ΟΜ) � (ΟΚ)+(ΚΜ) =>

I COK) - ρl � l z l � (ΟΚ)+ρ με (ΟΚ)= -./9 + 9 =3 .fi θα έχουμε :

max( lz l )=(ΟΚ)+ρ=3 .fi +2 min( l z l )=(ΟΚ)-ρ=3 .fi -2

γ. Οι μιγαδικοί z με το ελάχιστο και μέγιστο μέτρο έχουν εικόνες Μ ι , Μ2 αντιστοίχως τα σημεία το­μής της ΟΚ με το κύκλο ( c ) :

'Ε χουμε λοκ= 3 - 0 = 1 , οπότε ΟΚ: ψ=χ 3 - 0 Οι συντεταγμένες των Μ ι , Μ2, επαληθεύουν

το σύστημα: ? ? , δηλαδη Μ ι (3-.fi , ψ = χ

} ' (χ - 3) - + (ψ - 3)- = 4

ψ=3-.fi ) και Μ2 (3+ .fi , 3+ .fi ) Ο μιγαδικός με ελάχιστο μέτρο είναι ο z ι=(3-

.fi ) +(3-.fi )i και με μέγιστο μέτρο ο z2=(3+ .fi )+(3+ .fi )i δ. Αν ο z ι έχει εικόνα Α, και ο z2 έχει εικόνα Β, τότε ισχύει l z 1 - 3 - 3i l = lz2 - 3 - 3i l =2 δηλαδή τα

Α,Β βρίσκονται στον κύκλο του ερωτήματος (α). Εφόσον lz 1 - z2 1 = (ΑΒ), η μέγιστη τιμή της

παράστασης είναι ίση με τη διάμετρο του κύκλου, δηλαδή max C lz 1 - z2 1 ) = 2ρ = 4 �lί !Π �ΗRη� Ι11' Ιl σΙi! 2 Α ν ο μιγαδικός z κινείται σε κύκλο με κέντρο Κ και ακτίνα ρ, τότε το ελάχιστο μέτρο είναι ίσο με I (OK) - ρl και το μέγιστο μέτρο είναι ίσο με το (ΟΚ)+ρ, επίσης οι μιγαδικοί με το ελάχιστο και μέγιστο μέτρο έχουν εικόνες τα σημεία τομής της ΟΚ με τον κύκλο.

Αν για τους μιγαδικούς z.w ισχύουν οι σχέσεις lz - 31 =2 και 1 ( 1 - i)(w - 6i) l = 1(1 + i)(w - 3)1 τότε

να βρείτε:

α. Τον γεωμετρικό τόπο των εικόνων των μιγα­

δικών αριθμών z β. Τον γεωμετρικό τόπο των εικόνων των μιγα­

δικών αριθμών w γ. Την ελάχιστη τιμή του lw l δ. Την ελάχιστη τιμη του lz - wl Λiιση α. Αν Μ η εικόνα του z και Κ(3 ,0), τότε: I z - 3 1 = 2 <=> ( ΚΜ) = 2 . Άρα ο γεωμετρικός τόπος των Μ είναι ο κύκλος με κέντρο Κ(3 ,0) και ακτίνα ρ=2, δηλαδή ο ( c ) : (χ-3)2+ψ2=4 β. Έχουμε: 1 0 - i)(w - 6i) l = 1 ( 1 + i)(w - 3) 1 <=> 1 1 - i l lw - 6i l = 1 1 + i l lw - 31 <=> lw - 6i l = lw - 31 Αν Α(Ο,6) και 8(3,0) , τότε ο γεωμετρικός τόπος των εικόνων Ρ των w=α+βί είναι η μεσοκάθετος του ΑΒ

Εξάλλου : lw - 6i l = lw - 31 <=> Ι α + (β - 6)i l = ι c α - 3) + βi l <=> 3α - 4β + 9 = ο .

Άρα ο γεωμετρικός τόπος του Ρ, δηλαδή η μεσο-

Γ ψ ε

I i i

i χ ' χ I I

_ _ _ _ j ψ ι__ ______ ____ .. - - · · · --

ΕΥΚΛΕΙΔΗΣ Β ' 69 τ. Ι /64

Μαθη ματικά για την Γ Λυκείου

κάθετος του ΑΒ είναι η (ε) : 3χ--4ψ+9=0. γ. Εφόσον οι εικόνες του w βρίσκονται στην ευ­θεία ε : 3χ--4ψ+9=0 και lw l =(ΟΡ) το ελάχιστο μέ-τρο είναι ίσο με την απόσταση του 0(0,0) από την

13 .0 - 4.0 + 91 _ 9 (ε), δηλαδή την d(Ο,ε) - -.J32 + 42 5 Άρα min( lw l )=2. 5 δ. Εφόσον l z - w l = (ΜΡ) φέρω την κάθετη από το Κ στην (ε) τέμνει τον (c) : στο Μ Ι και την ε: στο P l , τότε για οποιαδήποτε σημεία Μ,Ρ ισχύει

(ΚΜ)+(ΡΜ) 2: (ΚΡ) 2: (ΚΡ 1 )=:>(ΚΜ)+(ΡΜ) 2: (ΚΜΙ ) +(Μ1Ρ 1 ) => ρ+(ΡΜ) 2: ρ+(Μ 1Ρ 1 ) =>

(ΡΜ) 2: (Μ 1Ρ 1 ) . Άρα η ελάχιστη τιμή του ΜΡ είναι ίση με (Μ 1 Ρ 1 )=d(Κ,ε )-ρ

' 13 . 3 - 4.0 + 91 1 8 1 8 Αλλα d(Κ,ε)= = - =:>(Μ1Ρ 1 ) =--2 .J32 + 42 5 5 =� =:>min( l z - w l )=� . 5 5

Ελάχιστη απόσταση των εικόνων μιγαδικών, όταν ο ένας βρίσκεται σε ευθεία (ε) και ο άλλος σε κύ­κλο με κέντρο Κ και ακτίνα ρ όταν ευθεία κύκλος δεν έχουν κοινά σημεία είναι ίση με d(κ,ε)-ρ, ενώ όταν ευθεία και κύκλος έχουν κοινά σημεία είναι ίση με Ο .

Θi:�ω -' Αν για τους μιγαδικούς z,w ισχύει l2z - 4i l =2 και lw - 3 + 3i l =2 τότε να βρείτε

α. Τους γεωμετρικούς τόπους των εικόνων των μιγαδικών z και w β. Την ελάχιστη και μέγιστη τιμή της παρά­στασης A= lz - w l γ. Έστω η παράσταση Β= lz + 3 + 2i l να δείξετε ότι 4 :<;; Β :<;; 6

α. Έχουμε: l 2z - 4i l =2 �2 lz - 2i l =2� lz - 2i l = 1 Άρα ο γεωμετρικός τόπος των M(z) είναι ο κύκλος με κέντρο Κ(Ο, 2) και ακτίνα ρ= l , δηλαδή ο C l : χ2+(ψ-2)2= 1

Επίσης: lw - 3 + 3i l =2 � lw - (3 - 3i ) l = 2 . Άρα ο γεωμετρικός τόπος των P(w) είναι ο κύκλος με κέ­ντρο Λ(3 , -3) και ακτίνα R=2, δηλαδή ο C2 : (χ-3)2+(ψ+3)2=4

β. A= lz - w i =(MP) ,ΚΛ=J32 + (-3 - 2)2 =J34 >R+ρ και εφόσον Μ ε C 1 , Ρ ε C2 , θα έχουμε

γ. Έχουμε:

max(A)=ΚΛ+R+ρ=J34 +3 min(A)=ΚΛ-R-ρ= J34 -3

B= lz + 3 + 2i l = lz - 2i + 3 + 2i + 2i l = lz - 2i + 3 + 4i l και l l z - 2i l - l3 + 4i l l :<;; l z - 2i + 3 + 4i l :<;; l z - 2i l + l3 + 4i l , οπότε I I - 5 1 :<;; Β :<;; 1 +5 . Άρα: 4 :<;; Β :<;; 6

* Οι αριθμοί 4, 6 είναι κάτω και άνω φράγμα της παράστασης Β . i-i αμαη� i ΙΨΠ� "� Έστω ότι οι εικόνες δυο μιγαδικών βρίσκονται σε κύκλο C 1 με κέντρο Κ και ακτίνα R και κύκλο C2 με κέντρο Λ και ακτίνα ρ με R 2: ρ Αν ΚΛ>R+ρ τότε η μέγιστη απόσταση των εικό­νων είναι ίση με ΚΛ+R+ρ και ελάχιστη τιμή της απόστασης είναι ίση με ΚΛ-R-ρ

Α ν R -ρ :<;; ΚΛ :<;; R +ρ τότε μέγιστη απόσταση των εικόνων είναι ίση με ΚΛ+R+ρ ενώ η ελάχιστη τιμή είναι ίση με Ο

Αν ΚΛ<R-ρ τότε μέγιστη απόσταση των εικό­νων είναι ίση με ΚΛ+R+ρ και η ελάχιστη τιμή εί­ναι ίση R-(ΚΛ)-ρ Γi ιφαη�ρηση 5 Όταν δίνεται ανισότητα μέτρον l z - z 1 1 :<;; α και θέ-λουμε να δείξουμε ότι,m :<;; l z - z2 1 :<;; Μ, τότε έχου­με lz - z2 1 = lz-Zι -� +Zι l = l (z -z1 ) + (z1 - z2 )1 οπότε

ΕΥΚΛΕΙΔΗΣ Β' 69 τ. l/65

Μαθη ματικά για την Γ Λυκείου

από την τριγωνική ανισότητα παίρνουμε l l z - z ι l -l l z ι - z2 l :::; ! (z - z ι ) + (zι - z2 ) l :::; l z - z ι l + l z ι - z2 1 Θέμα 5

Α. Α ν για τους μιγαδικούς z ισχύει lz + Si l + lz - Si l =26 (1 ) , τότε να βρεθεί :

α. Ο γεωμετρικός τόπος των εικόνων των μιγα­δικών z β. Η μέγιστη και η ελάχιστη τιμή του lz l γ. Το μέγιστο και το ελάχιστο της παράστασης lz ι - Ζ2 1 , όταν οι μιγαδικοί zι ,z2 αληθεύουν την

σχέση (1) Β. Α ν για τους μιγαδικούς αριθμούς w ισχύει lw + Si l - lw - Si l =8 (2), τότε να βρεθεί

α. Ο γεωμετρικός τόπος των εικόνων των μιγα­δικών w β. Η ελάχιστη τιμή του lw l .\ ίιση Α. α. Αν Μ η εικόνα του z και Γ(Ο,5) και Γ(Ο,-5) , τότε ( I )�(ΜΓ) +(ΜΓ)=26. Εφόσον (ΓΓ) = 1 0<26, ο γεωμετρικός τόπος των Μ είναι η έλλειψη (c) με εστίες Γ, Γ και μεγάλο άξονα 2α=26, δηλαδή α= 1 3 και εστιακή απόσταση (ΓΓ)=2γ= Ι Ο, δηλαδή γ =5 και β= �α2 - γ2 = .Jί44 = 1 2 .

ο 7

Εφόσον Γ, Γ εψψ ' θα είναι (c) : � + ψ- = 1 1 44 1 69 β. Για κάθε σημείο Μ(χ,ψ) της έλλειψης (c) έχου-

2 2 7 7 ο 7 (ΟΜ ) '

με 1 =� +� :::; � + ψ- = χ - + ψ- = -1 44 1 69 1 44 1 44 1 44 1 44 '

οπότε (ΟΜ) ;;::: 1 2 (το ίσον στις θέσεις Β, Β ') και 1 = χ 2 + ψ2 > χ 2 + ψ2 = χ 2 + ψ2 = (ΟΜ)2

1 44 1 69 1 69 1 69 1 69 1 69 '

Α ψ

χ ' χ

Α ' ψ

οπότε ( ΟΜ) :::; 1 3 (το ίσον στις θέσεις Α, Α ') Άρα min( l z l )=β= 1 2 και max( l z l )=α= 1 3 . Συνέπεια των παραπάνω είναι το ότι ο μεν κύκλος (0, β) εί­ναι στο εσωτερικό της έλλειψης ενώ ο κύκλος (0, α) είναι στο εξωτερικό της, εκτός φυσικά από τα σημεία Α, Α ' , Β, Β ' που είναι κοινά. γ. Εφόσον οι μιγαδικοί z 1 ,z2 αληθεύουν την σχέση ( 1 ), οι εικόνες τους Μ 1 , Μ2 ανήκουν στην έλλειψη C, οπότε ( Μ1 Μ2 ) :::; ( ΟΜ1 ) + ( ΟΜ2 ) :::; 1 3 + 1 3 = 26 . Άρα max( l z 1 - z2 1 )=2α=26, ενώ lz 1 - z2 1 ;;::: Ο με το ίσον όταν z1 = z2 . Άρα min lz 1 - z2 1 =0. Β. α. Αν Ρ η εικόνα του w και Γ(Ο,5) Γ(Ο,-5), τό­τε (2)�(ΡΓ)-(ΡΓ)=8 . Εφόσον (ΓΓ)= Ι Ο > 8 , ο γεωμετρικός τόπος των Ρ θα είναι ο κλάδος υ­περβολής με εστίες Γ, Γ και άξονα 2α=8 δηλαδή α=4, γ=5 και β= �γ2 - α2 = J9 = 3 .

Γ ψ

χ ' ο χ

ψ

Εφόσον (ΡΓ)>(ΡΓ) και Γ, Γ εψψ ' θα είναι ο άνω 2 2 κλάδος της υπερβολής C: .!.____� = Ι 25 9

β. Εφόσον lw l =(ΟΡ) , η ελάχιστη τιμή του (ΟΡ) είναι η (ΟΑ) διότι για κάθε άλλη θέση του Ρ το τρίγωνο ΟΡΑ είναι αμβλυγώνιο στο Α, οπότε (ΟΡ)>(ΟΑ)=α (* ) . Άρα min( lw l )=α=4 (*) Γενικά για κάθε σημείο Ρ(χ,ψ) του άνω κλάδου

7 ?

β λ ' ψ- χ- I δ ' ' της υπερ ο ης -, - -7 = , ιαφορετικο απο το Α, α- β-έχουμε ψ>α, (ΟΡ)2=χ2+ψ2, (ΡΑ)2=χ2+(ψ-α)2 , οπό­τε (ΟΑ)2 +(ΡΑ)2 = α2 + χ2 + (ψ-α)2 = χ2 +� +2α(ψ-α) = (ΟΡ)2 + 2α (ψ - α) > (ΟΡ)2 , αφού ψ > α . Άρα το τρίγωνο ΟΡΑ είναι αμβλυγώνιο στο Α.

ΕΥΚΛΕΙΔΗΣ Β' 69 τ. l/66

Μαθηματικά για την Γ Λυκείου

Θi:[U!U 6 Α ν για τους μιγαδικούς z ισχύει (z-2i)4 = 81 (z+2i)4 (1 ) τότε να δείξετε α. Οι εικόνες των μιγαδικών z βρίσκονται σε κύκλο τον οποίο να προσδιορίσετε β. Να βρεθεί η μέγιστη και η ελάχιστη τιμή του l z l . . Ηιση α. Έστω z=xo+ψ0i και Μ η εικόνα του, τότε: (z-2i)4= 8 ι (z+2i)4 � l<z - 2i)4 1 = Ι8 ι (z + 2i)4 1 � lz - 2i l 4= l3(z + 2i) l 4� lz - zi l =3 lz + zi l � lz - 2i l 2=9 l z + 2i l 2�χ<?+(ψ0-2)2=9 [χ02+(ψ0+2)2] �χ02+(ψ0+

2_ )2=2.. Άρα οι εικόνες Μ των μιγαδι-2 4

' β ' ' λ ( ) 2 ( 5 )2 9 κων z, ρισκονται στον κυκ ο c : χ + ψ+- =-2 4 , Κ(Ο -5

) , 3 με κεντρο , 2 και ακτινα ρ= 2 . β. Πρέπει πρώτα να προσδιορίσουμε τους μιγαδι­κούς z, γιατί προφανώς δεν είναι όλα τα σημεία του C, αλλά μόνο 4 από αυτά, αφού : (z-2i)4 = 8 1 (z+2i)4 <=> (z-2i)2 = 9(z+2 i)2 ή (z-2 i)2 = - 9(z+2i)2 <=>z-2i = ±3(z+2i) ' 2 " 3 " ( 2 " ) {--4· . 6 8 . 6 8 . } η z- ι = ± ι z+ ι <=> z ε ι - ι - - - - ι - - - ι . ' ' 5 5

, 5 5

' . 6 8 . Για τις τιμες z 1= --4ι, z2= -ι , z3= - - - - ι και 5 5

6 8 . ' z4=- - - ι εχουμε: 5 5

l z3 1 = 1 z4 1 = 2 .

χ '

ψ

χ

i ___j

Άρα μέγιστο μέτρο έχει ο z 1 και ελάχιστο ο z2 . rπ ���or:o χ Η :

και

Στην περίπτωση μας έτυχε το μέγιστο και το ελά­χιστο μέτρο του z να αντιστοιχούν στη μέγιστη

απόσταση (ΟΑ) και την ελάχιστη απόσταση (ΟΒ) του Ο από τον κύκλο ( c ) , διότι

5 3 χΑ = χκ -ρ=-2 -2 =-4 και 5 3

Χ 13 = χ κ + ρ = -2 +2 = - ι , δηλαδή A(z 1 ) και Β ( z2 ) . Αυτό όμως δεν συμβαίνει πάντοτε διότι πιθανόν κανένας από τους z 1 , z2 , z3 , Ζ4 να μην α­πεικονίζεται στο Α ή στο Β . i ! cφαη)ρηση 6 Όταν δίνεται Ζ ι v=z2 ν ( ι ) , τότε από ιδιότητες μέ­τρων έχουμε:

( l )� lz ι v l = lz2 ν ι � lz ι l v= l z2 1 ν � lz ι l = l z2 1 · . . Θi:μυ 7 Έστω μιγαδικοί z1 , z2 με εικόνες εσωτερικά ση­μεία του μοναδιαίου κυκλικού δίσκου εκτός του 0(0,0) και η συνάρτηση F(x)= lz ι l '+2 lz 2 1 '-3 α. Να δείξετε lz ι - z2 1 <2

β. Να δείξετε ότι F αντιστρέφεται γ. Να λυθεί η ανίσωση :

l z , l x2+J- 2 lz z l 4' � l z ι l 4'-2 lz z l x2+J

. ΗJση Ο μοναδιαίος κυκλικός δίσκος έχει εξίσωση C: χ2+ψ2 5 ι , κέντρο 0(0,0) και ακτίνα ρ= ι α. Έστω Α η εικόνα του z 1 και Β η εικόνα του z2 , τότε l z 1 - z2 1 =(AB) . Εφόσον Α, Β εσωτερικά σημεία του κυκλικού δί­σκου, θα είναι (ΑΒ) <2ρ, δηλαδή l z ι - z2 1 <2 . β. Αν l z ι l =α τότε Ο< α< 1 και l z2 1 =β τότε Ο< β< ι F(x)= lz ι l '+2 l z2 1 '-3=α'+2β'-3 , με σύνολο ορισμού A=R. Για κάθε χ 1 , χ 2 ε R έχουμε: χ 1 <χ2�α' 1>α'2 και β' ι >β'2 � α χ ι+2 β' ι> α χ2 +2 β'2 � �α' 1 +2 β' 1-3> α '2 +2β'2-3� F(χ ι ) >f(x2) . Άρα

ν

F .,!, στο R, οπότε F « ι - ι » στο R. Επομένως η F

αντιστρέφεται. γ. l z ι l χ2+3_ 2 lz2 1 4x � lz ι l 4x_2 l z2 1 '2+3 <=> lz ι l x2+3+2 lz2 1 x2+3 � lz ι l 4'+2 l z2 1 4x<=> lz ι l x2+3+2 l z2 1 χ2+3 -3 � lz ι l 4'+2 l z2 1 4x_3<=>

F � F(x2+3) � F(4x) <=> χ2+3 5 4χ<::::>χ2--4χ+3 5 ο

<::::>χ ε [ ι ,3 ]

ΕΥΚΛΕΙΔΗΣ Β ' 6 9 τ. Ι /67

Μαθη ματικά για την Γ Λυκείου

Έστω οι μιγαδικοί z,w για τους οποίους ισχύει jz - 3 - zi j + jw - 5 - 9i j =Ο και η συνάρτηση

F :R � R γνησίως μονότονη α. Ν α βρείτε τις εικόνες Α, Β των z, w β. Α ν F διέρχεται από τα Α,Β να βρείτε το είδος της μονοτονίας γ. Να λύσετε την εξίσωση F(2+Γ1 (χ2+χ))=9 δ. Να λύσετε την ανίσωση F(Γ1 (χ2-8χ)-2)<2, όταν γνωρίζετε ότι F, Γ1 έχουν το ίδιο είδος μο­νοτονίας.

α. jz - 3 - zij + jw - 5 - 9i j =0<:::> jz - 3 - zi j =Ο και

α. Η μέγιστη τιμή της παράστασης j z 1 + z2 j

β. Η τιμή του z2 για την οποία η τιμή της παρά­στασης j z 1 + z2 j γίνεται μέγιστη .

Υπ{Jδπtη : Σύμφωνα με την παρατήρηση 5 Θ{:μυ 2

. z2 - 1 , Αν zεC με z :;t: ψΙ και F(z)=--_- τοτε z + z

α. Να δείξετε ότι F(z) = F( z )

jw - 5 - 9i j =O<::>z-3-2i=O και w-5-9i=O β. Να δείξετε ότι j F(z) j = jF(z)j <:::> z=3+2i και w=5+9i . Εικόνα του z είναι το Α(3 ,2) και του w το Β(5 ,9) .

β. AεCF <::>F(3)=2 και BεCF<::>F(5)=9 . Εφόσον η F είναι γνησίως μονότονη με 3<5 και F(3) <F(5), η F θα είναι γνησίως αύξουσα στο R

γ. Εφόσον η F είναι γνησίως αύξουσα στο R, θα είναι και « 1- 1 » στο R, άρα η F αντιστρέφεται. F(3)=2 <:::>F- 1 (2)=3 και F(5)=9<::>F- 1 (9)=5 F(2+F- 1 (x2+x))=9<::> F(2+F- 1 (x2+x))=F(5) <:::>2+ F- 1 (x2+x)=5<::> F- 1 (x2+x)=3<::> F- 1 (x2+x)= F-1 (2) <::>χ2+χ=2<::> x2+x-2=0<::>x= l ή χ=-2 εφόσον F « 1-1 » και F-1 « 1- 1 ».

δ. Η F είναι γνησίως αύξουσα στο R, οπότε και η F - Ι θα είναι γνησίως αύξουσα στο R. Άρα: F (F - 1 (χ2 -8x)-2)<2<::>F(F - ι (χ2 -8χ)-2) <F(3 ) <:::> F - 1 (x2-8x)-2<3<::>F - 1 (x2-8x) <5<:::> F - 1 (χ2 -8x)<F - 1 (9)<::>χ2-8χ<9<::> χ2 -Sx-9<0<:::> χ ε (- 1 ,9)

Β. nτ ι� � Ί> .Η. � " > Χ \� �'" θ fί.� \>'Υ Λ Τ Λ - Υ Π ΟΔ Ε Ι -Ξ ΊΤ i .

Α. Αν zεC και j z - 1 - i j <5 να δείξετε ότι l O< j z - 1 0 - 1 3i j <20

γ. Να βρεθεί ο γεωμετρικός τόπος των εικόνων Μ του z αν F(z) φανταστικός

δ. Αν j z j =3 να βρεθεί ο γεωμετρικός τόπος. των εικόνων Ρ του g(z) αν ισχύει g(z)=z+2-i

1ι Ίτ6δηξη : δ) Έστω g(z)=w=x+ψi Ρ(χ,ψ), g(z)=z+2-i<::>g(z)-2+i=z<::> jg(z) - 2 + i j = j z j <:::> j w - 2 + i j = 3 , τότε ο γεωμετρικός τόπος των Ρ είναι κύκλος με κέντρο Κ(2,- 1 ) και ακτίνα ρ=3 .

Θi:μα 3 Α. Αν για τους μιγαδικούς z, w ισχύει j z j = jw j τότε να δείξετε

' z + w )20 1 0 ' ' β α. Ο αριθμος u=(-- ειναι πραγματικος . z - w

' ( z + w )2009 ' ' ο αριθμος ν= -- ειναι φανταστικος z - w

Β. Αν για τους μιγαδικούς z 1 , z2, z3 ισχύει j z 1 I = 1 , j z2 j =2, j z3 j =4 τότε να δείξετε

2 Υπ6.ι)υξη : Α. jz j = jw j =ρ<::> . . . . . . . . z =Ε_ . . . . . · ·

z Β. Αν z 1 ,z2 εC με z1=8+6i και j z2 j =5 τότε να βρεθεί: Β. α. Να δείξετε j z 1 + z2 + z3 j :;t:O ή με εις άτοπο . . .

ΕΥΚΛΕΙΔΗΣ Β ' 69 τ. t /68

Μαθη ματικά για την Γ Λυκείου

Σχετικά με ένα θέμα των πανελληνίων εξετάσεων 2008

Του Αντώνη Κυριακόπουλου

Πρόκειται για το δεύτερο θέμα των μαθηματικών θετικής και τεχνολογικής κατεύθυνσης. Το θέμα αυτό

έχει ως εξής:

Θ Ε Μ Λ 2° Α ν για τους μιγαδικούς αριθμούς z και w ι­

σχύουν /(i + 2.J2)z / = 6 και /w-(1 - i)/ = /w-(3-3i)/

τότε να βρείτε: α. Το γεωμετρικό τόπο των εικόνων των μιγα­δικών αριθμών z. β. Το γεωμετρικό τόπο των εικόνων των μιγα­δικών αριθμών w. γ. Την ελάχιστη τιμή του /w / . δ. Την ελάχιστη τιμή του / z - w/ .

Λ Υ� Η . υ ) Έχουμε : / (i + 2h)z / = 6 � / i + 2h / . / z/ = 6 � 3/z/ = 6 � / z/ = 2.

Άρα, ο ζητούμενος γεωμετρικός τόπος είναι ο κύ­κλος C με κέντρο το σημείο 0(0,0) και ακτίνα ρ=2 ( στο σχήμα, που έστειλε στα βαθμολογικά κέντρα η επιτροπή εξετάσεων, τον κύκλο τον έχει με ακτί­να 1 . Δεν είναι σπουδαίο, αλλά δείχνει την προ­χειρότητα με την οποία εργάζονται ).

I /

,/ // r !

)'

ο

Ν · ·· ... ii/�.:I(-2 , 2)

/ .

-,� '1' r β) Π ρώτος τρ6πος ( Γεωμετρ ικά) . Ο ζητούμε­νους γεωμετρικό τόπος είναι η μεσοκάθετος ε του ευθυγράμμου τμήματος με άκρα τα σημεία A( l ,- 1 )

και Β(3,-3 ) . Το μέσο του ευθυγράμμου τμήματος ΑΒ είναι Μ (2,-2) . ο συντελεστής διεύθυνσης της

(-3) - (- 1 ) ευθείας ΑΒ είναι : λ Α Β = = - 1 . 3 - 1

Άρα ο συντελεστής διεύθυνσης της ευθείας ε εί­ναι λc = Ι . Και επειδή η ε περνάει από το σημείο Μ (2 ,-2), η εξίσωση της είναι:

y - ( -2) = 1 . ( χ - 2) � y = χ - 4. Δεύη:ρος τρ6πος ( Λλγεβρ ικιΊ) .

Θέτουμε: w = χ + yi , x,y Ε IR . 'Έτσι , έχουμε: /w - (1 - i)/ = /w - (3 - 3i) / �

� /(χ - 1) + (y + 1)i / = / ( χ - 3) + (y + 3)i / �

� �( χ - 1 )2 + ( y + l)2 = �( χ - 3)2 + ( y + 3γ � � . . . � y = x - 4. Άρα , ο ζητούμενους γεωμετρικό τόπος είναι η

ευθεία ε:y=χ-4. γ) Π ρώτος τρόπος ( Γ αuμι:τ ρ ικιί ) . Η ελάχιστη τι­μή του /w / είναι ίση με την απόσταση (ΟΜ) της αρχής των αξόνων Ο από την ευθεία ε:y=χ-4. Άρα ,η ελάχιστη τιμή του /w / είναι :

(ΟΜ) = �( 2 - 0)2 + (-2 - 0)2 = 2J2. ΣΧΟΛ ΙΟ. Η λύση αυτή είναι δεκτή ,όχι γιατί βλέ­

πουμε από το σχήμα ότι ΟΜ�ΟΛ, για κά-

θε σημείο Λ της ευθείας ε, αλλά γιατί αυτό το έχουν αποδείξει οι μαθητές στη Γεωμε­τρία που διδάσκονται.

Λεί)τψος τρόπος ( Αλγεβ ρ ιι<6: ) . Στο προηγούμενο ερώτημα θέσαμε w = x+yi, x,y Ε IR και βρήκαμε ότι y=x-4 . 'Έτσι έχουμε: w=x+(x-4), Χ Ε IR . Συνεπώς: /w / = �x2 + ( χ - 4)2 = J2.��x2,-----4-x-+-8 . Γνωρίζουμε ότι το τριώνυμο f(x) = χ 2 - 4χ + 8

έχει ελάχιστη τιμή για χ = - -4 = 2 , ίση με f(2)=4-2

8+8=4. Άρα, έχουμε για κάθε χ Ε IR :

ΕΥΚΛΕΙΔΗΣ Β ' 69 τ. l/69

Μαθηματικά για την Γ Λυκείου

lw l � J2.J4 = 2J2 , με το ίσον για χ=2 .

Συνεπώς η ελάχιστη τιμή του l w l είναι 2J2 . δ ) Παραθέτω τη λύση που συνέταξε η επιτροπή εξετάσεων και απέστειλε στα βαθμολογικά κέντρα ( ενδεικτικές απαντήσεις) : «Η ελάχιστη τιμή του l z - w l ισούται με το μήκος του τμήματος ΜΝ.

Έχουμε : (ΜΝ)=(ΟΜ)-(ΟΝ)= 2J2 -2». Αυτό που λένε δεν το αποδεικνύουν ούτε Αλ­

γεβρικά ,ούτε Γεωμετρικά ,αλλά το στηρίζουν α­ποκλειστικά και μόνο στην εποπτεία (αυτό που βλέπουν). Σημειώνω ότι στη Γεωμετρία του Λυ­κείου δεν αναφέρεται η έννοια τις απόστασης ευ­

θείας και κύκλου. Και το ότι ισχύει ΜΝ :::;ΚΛ, για

κάθε σημείο Κ του κύκλου C και για κάθε σημείο Λ ευθείας ε , δεν αναφέρεται ούτε σαν άσκηση .

Η εποπτεία στα μαθηματικά είναι χρήσιμη για να κατανοούμε περισσότερο τις διάφορες μαθηματικές έννοιες. Επίσης μας βοηθάει στην αναζήτηση λύ­σεων. Π οτi: όμως δεν στηρίζουμε μία λί>ση στψ ι :ποπτr. ία. Α ν είχαν επιλέξει ως ένα θέμα να αποδεί­ξουν το θεώρημα ενδιάμεσων τιμών (Σχολικό βι­βλίο Μαθηματικών Κατεύθυνσης, σ. 1 94) και ένας μαθητής στηριζόταν μόνο στο σχήμα που έχει το σχολικό βιβλίο στην ίδια σελίδα, θα ήταν σωστή η

απόδειξη ; Και βέβαια θα μου πείτε όχι. Τότε γιατί τη δική τους λύση είναι σωστή ; Ίσως μου πείτε ότι η άσκηση αναφερόταν στους Μιγαδικούς Αριθ­μούς. Και λοιπόν; Οι Μιγαδικοί Αριθμοί είναι μα­θηματικά δεύτερης κατηγορίας; • Λοιπόν,θα έπρεπε η επιτροπή στη λύση της

πρώτα να αποδείξει γεωμετρικά την παραπάνω

σχέση (ΚΛ;?:ΟΛ-ΟΚ;?:ΟΜ-ΟΝ=ΜΝ) και μετά

να πει αυτά που λέει. Λ.λγΕβρ ική λίιση . Λαμβάνοντας υπόψη και τα προηγούμενα ερωτήματα, δηλαδή ότι: l z l = 2 και

l w l � 2Ji(> 2) , έχουμε:

l z - w l = l z + (-w)l � l l z l - 1-w l l = 1 2 - lw l l =

= l lw l - 21 = lw l - 2 :2: 2J2 - 2 Άρα: l z - w l � 2J2 - 2 . Τώρα θα πρέπει να εξετά­

σουμε αν υπάρχουν αριθμοί z και ω για τους οποί­ους να ισχύει η ισότητα και βέβαια να πληρούν και

τις δοσμένες συνθήκες ( γιατί π.χ. ισχύει: ημχ;?:-5 ,

για κάθε χ ε IR. , αλλά το -5 δεν είναι ελάχιστη τι­μή του ημχ). Όπως μπορούμε να βρούμε, τέτοιοι αριθμοί υπάρχουν και είναι οι εξής: z = J2 - i.J2 και w = 2 - 2i . Άρα η ελάχιστη τιμή του l z - ωl είναι 2J2 - 2.

Οι aντίστροφες συναρτήσεις r Ρ

και τα Ί(οινα τους σημεια των Αντώνη Κυριακόπουλου, Νίκου Φωτιάδη Δρ. Μαθηματικών

Τα τελευταία χρόνια κυκλοφόρησε ένα βιβλιαράκι στο οποίο ο συγγραφέας του κ. Πετράκης αναφέ­ρει ότι τα κοινά σημεία μιας συνάρτησης f και της αντίστροφής της βρίσκονται μόνο πάνω στην ευθεία

y = χ . Αυτό είναι λάθος. Παρ' όλες τις εξηγήσεις που δόθηκαν στον κ. Πετράκη τόσο στη Μαθηματική Εταιρεία όσο και αλλού, ο ίδιος εξακολουθεί να διαστρεβλώνει την αλήθεια εμμένοντας στους ισχυρι­σμούς του . Αποτέλεσμα αυτών των ισχυρισμών του ήταν να παρασυρθούν ορισμένοι συνάδελφοι μαθη­ματικοί και ενδεχομένως να δίδαξαν εσφαλμένα στους μαθητές τους ότι τα κοινά σημεία των γραφικών παραστάσεων μιας συνάρτησης και της αντίστροφής της βρίσκονται πάνω στην ευθεία y = χ και μόνο εκεί. Με το άρθρο μας αυτό θα επιχειρήσουμε να αποκαταστήσουμε την αλήθεια.

U . Λ ΥΠ l: τ J)ΟΦ f:Σ :L Y \ AYΠ-n : E i l: Ορισμός. Μια συνάρτηση f : Α � R λέμε

ότι είναι 1 - 1 αν και μόνο αν: Για κάθε χ1 , χ2 Ε Α , ισχύει :

Χι * Xz � /(xι ) * f(xz ) · Ισοδύναμα: Για κάθε Χι , χ2 Ε Α , ισχύει:

f(xι ) = f(xz ) � xι = Xz · • Έστω ότι μια συνάρτηση f : Α � R είναι 1 -

1 . Τότε, για κάθε y Ε f(A) , υπάρχει ένας α­κριβώς αριθμός χ Ε Α με f(x) = y . Έτσι, τότε ορίζεται μια νέα συνάρτηση, την οποία συμβολίζουμε με ι- ) ' ως εξής:

ΕΥΚΛΕΙΔΗΣ Β ' 69 τ. l/70

Μαθη ματικά για την Γ Λυκείου

/-' : f(A) � JR με f-' (y) = x � f(x) = y, για κάθε χ Ε Α και για κάθε y Ε /(Α).

Η συνάρτηση αυτή /-' ονομάζεται αντί­στροφη της f .

Έτσι, αν μια συνάρτηση f είναι 1 - 1 τότε ορί-

ζεται η αντιστροφή της συνάρτηση /-ι . Στην πε­ρίπτωση αυτή λέμε ότι η f αντιστρέφεται.

Ισχύουν: ι- ι (/(χ)) = χ ' για κάθε χ Ε Α και

/(/- ι (y)) = y , για κάθε y Ε /(Α). Π ιφ 6.6*ημυ.. Θεωρούμε τη συνάρτηση

f(x) = -χ3 , χ Ε JR . Η συνάρτηση αυτή, όπως βρίσκουμε εύκολα, είναι 1 - 1 και επομένως αντι­στρέφεται. Το σύνολο τιμών της βρίσκουμε ότι εί-ναι j(JR) = JR. Για να βρούμε τον τύπο της /- ι λύνουμε την εξίσωση y = -χ3 ως προς χ . Αν

y < Ο έχουμε χ = h , ενώ αν y � Ο έχουμε

, {h' αν y < Ο χ = -ifY . Δηλαδη : χ =

-ifY, αν y � Ο Έτσι, ο τύπος της aντίστροφης συνάρτησης /- ι

- ι {h' αν y < Ο είναι : f (y) =

-ifY, αν y � Ο Στην αντίστροφη συνάρτηση /- ι οι ρόλοι των

μεταβλητών χ και y έχουν εναλλαχθεί. Εδώ, η ανεξάρτητη μεταβλητή είναι το y και η εξαρτη­μένη μεταβλητή είναι το χ . Επειδή συνηθίζεται η ανεξάρτητη μεταβλητή να συμβόλιζε με χ και η εξαρτημένη μεταβλητή με y , τον τύπο της /-ι

ι {�' αν χ < Ο

τον γράφουμε : f- (χ) = > Γ -'Vx , αν χ � Ο 2 . η .ι. -.: ! �{ [ 2: Π Ι>(ΗΑ2: Ε Ι Σ

� μι'ηνση 1 . Αν μια συνάρτηση f : Α � JR είναι γνησίως μονότονη τότε είναι 1 - 1 .

,\:t (Jδ .;: ιξη . Έστω ότι f ! Α ( όμοια εργαζόμαστε

αν f J Α ). Θεωρούμε δύο αριθμούς Χι, χ2 Ε Α με

Χι =F χ2 . Τότε έχουμε: {Χι < Χ2 {/(χι ) < /(χ2 ) ή => ή => /(xι ) =F .f(x2 ).

Χι > Χ2 /(χι ) > f(x2 ) Άρα, η .f είναι 1 - 1 .

l.:ιj μt:ίιωση . Το αντίστροφο δεν ισχύει.

Π ρόταση 2 . Αν μια συνάρτηση f : Α � JR α­ντιστρέφεται, τότε οι γραφικές παραστάσεις C f της f και C Γ' της /-ι είναι συμμετρικές

ως προς την ευθεία y = χ (διχοτόμος της 1 ης και της 3ης γωνίας των αξόνων). Απόiiηξη . Έστω ότι μια συνάρτηση f : Α � JR αντιστρέφεται. Έχουμε: Μ( α, β) ε ς �β= f(α) �

� α = .r- ι (β) � Μ '(β, α) Ε CΓ, . Και επειδή τα σημεία Μ(α, β) και Μ'(β, α) εί­ναι συμμετρικά προς την ευθεία y = χ , έπεται ότι

οι γραμμές C1 και CΓ, είναι συμμετρικές ως

προς την ευθεία αυτή y = χ . Π ρόταση 3 . Έστω ότι μια συνάρτηση f : A�JR είναι γνησίως αύξουσα (οπότε αντιστρέφεται). Τότε τα κοινά σημεία των C1 και CΓ, (αν υ-

πάρχουν) ανήκουν στην ευθεία y = χ . Απόδειξη . Έστω ότι M(x0 , y0 ) είναι ένα κοινό

σημείο των γραμμών C/ και CΓ, . Τότε:

χ0 ε (A n f(A)) και /(χ0 ) = /- ι (χ0 ) ( 1 ) .

Θα δείξουμε ότι: f(x0 ) = χ0 . Πράγματι, έστω

ότι: /(χ0 ) > χ0 • Τότε, από την ( 1 ) και επειδή

Ι ! Α ' έχουμε: f(xo ) > Χο => ι- ι (χο ) > Χο =>

=> /(/- ι (χ0 )) > f(x0 ) => χ0 > f(x0 ) που είναι άτοπο. Όμοια φτάνουμε σε άτοπο αν υποθέσουμε ότι /(Xr,) < χ0 • Άρα /(χ0) = χ0 και συνεπώς το ση-

μείο Μ(χ0,.Υ(>) ανήκει στην ευθεία y =x . Ση μείωση . Όπως θα δούμε παρακάτω, οι παραπά­νω πρόταση δεν ισχύει αν η συνάρτηση f είναι γνησίως φθίνουσα. 3 . KO I N A Σ Η Μ Ε Ι Α ΤΩΝ ΓΡΑΦ Ι ΚΩl\ Π Α­ΡΑΣΤΆΣ ΕΩΝ ΔΥΟ Α Ν τJ ΣΤ ΡΟΦΩΝ ΣΥ­Ν Α ΡΤ Η Σ ΕΩ Ν

Οι συντεταγμένες των κοινών σημείων των γραφικών παραστάσεων c/ και CΓ, δύο αντι-

στρόφων συναρτήσεων f : Α � JR και

/-ι : f(A) � JR , είναι οι πραγματικές λύσεις του

{Υ = /(χ) συστήματος: ·

- ι με χ ε (A n f(A)) .

Υ = j (χ) Όσες πραγματικές λύσεις έχει το σύστημα αυτό,

τόσα είναι και τα κοινά σημεία των δύο αυτών γραμ­μών. Αν το σύστημα αυτό είναι αδύνατο, τότε οι γραμμές αυτές δεν έχουν κοινό σημείο. Συνεπώς:

ΕΥΚΛΕ Ι ΔΗΣ Β ' 69 τ. Ι /7 1

Μαθη ματικά για την Γ Λυκείου

Οι γραφικές παραστάσεις δύο αντιστρόφων συναντήσεων μπορεί να μην έχουν κοινά σημεί­α. Μπορεί όμως να έχουν ένα ή περισσότερα. Στην περίπτωση που έχουν κοινά σημεία αυτά δεν ανήκουν αναγκαία στη διχοτόμο y = χ (δεν υπάρχει τέτοιο θεώρημα), εκτός αν η f είναι γνησίως αύξουσα (πρόταση 3).

Αναφέρουμε τα παρακάτω παραδείγματα: 1) Όπως είδαμε παραπάνω, η συνάρτηση : f (χ) = -χ3 , χ Ε JR. , έχει αντίστροφη τη συ-

, _1 {�' αν χ < Ο ναρτηση : f (χ) = 1 Γ .

-ν χ ' αν χ 2': ο Όπως μπορούμε να βρούμε εύκολα, οι γραφικές

τους παραστάσεις έχουν τρία κοινά σημεία, τα ε­ξής: ( - 1, 1) , (1 , - 1) και (0, Ο) . Από αυτά μόνο το (0, Ο) ανήκει στην ευθεία y = χ . Δεν εφαρμόζε­ται η πρόταση 3, γιατί η f δεν είναι γνησίως αύξουσα. Είναι γνησίως φθίνουσα.

Οι γραφικές παραστάσεις των δύο αυτών συ­ναρτήσεων δίνονται στο παρακάτω σχήμα:

2) Η συνάρτηση f : (0, +οο) ----; 1R. με f(x) = \η χ έχει αντίστροφη τη συνάρτηση ι- Ι : JR. ----; JR. με

f-1 (χ) = ex . Οι γραφικές τους παραστάσεις (βλέπε παρακάτω σχήμα) δεν έχουν κανένα κοινό σημείο.

3) Η συνάρτηση f (χ) = 2χ - 3 , χ Ε 1R. έχει αντί-

ι χ + 3 στροφη τη συνάρτηση f- (χ) = -- , χ Ε JR. . Οι

2

γραφικές τους παραστάσεις έχουν ένα μοναδικό κοινό σημείο, το (3 ,3) . Το σημείο αυτό ανήκει στην ευθεία y = χ . 4) Η συνάρτηση f(x) = 4 - χ, χ Ε 1R. έχει αντί­

στροφη τη συνάρτηση ι-' (χ) = 4 - χ ' χ Ε JR. . Οι γραφικές τους παραστάσεις έχουν όλα τα σημεία τους κοινά (ταυτίζονται, άπειρα κοινά σημεία). Από αυτά μόνο ένα ανήκει στην ευθεία y = χ , το (2,2).

!Πi Θεωρεί διανυσματικές εξισώ­σεις r{ ( t ) = . . . Καt rΓ1 ( t ) = . . . των γραμ-

μών Cr και CΓι , αντιστοίχως και για να βρει

τα κοινά τους σημεία λύνει την εξίσωση : rr (t) = rΓI (t) . Αυτό είναι λάθος. Γιατί, για

να βρούμε τα κοινά σημεία των δύο αυτών γραμμών, πρέπει και αρκεί να βρούμε όλα τα ζεύγη ( t, , t2 ) με t, Ε D(r1 ), t2 Ε D(rΓι ) ,

για τα ΟΠΟία ισχύει r1 (t1 ) = rΓ1 (t2 ) . (γι' αυ­

τό χάνει τα σημεία που είναι έξω από την ευ­θεία y = χ ).

,· Δ;.:iJη;�HJ Για να αποδείξει ότι η συ-νάρτηση f(x) = 4 - χ , χ Ε JR. δεν είναι ίση με την αντίστροφή της: ι- Ι (χ) = 4 - χ ' χ Ε JR. ( ! ! ! ) υποθέτει ότι είναι ίσες και θεω­ρεί τις εξής διανυσματικές εξισώσεις των C1 και CΓι : � (t) = t · f + (4 -t) .] , t E JR. ( 1 )

Και rΓ1 (t) = ( 4 - t ) · Τ + f · ] , t Ε JR. (2)

Βρίσκει ότι: F(t) = 1 · τ - 1 · J = (1, - 1) και FΓ

I (t) = - 1 · τ + 1 · J = ( - 1, 1) και φθάνει, όπως λέει, στο «καταπληκτικό» συμπέρασμα ότι «τα εφαπτόμενα διανύσματα σε οποιοδήποτε σημείο των γραφικών τους παραστάσεων κατά τη φορά διαγραφής τους, είναι αντίθετα! ! ! Α ΤΟΠΟ» . Αυτό όμως, όχι μόνο δεν είναι καταπληκτικό

και άτοπο, αλλά σύμφωνα με τη Διαφορική Γεωμετρία, είναι αναμενόμενο. Πράγματι η ε­ξίσωση (2) προκύπτει από την ( 1 ) θέτοντας όπου t το 4 - t (αλλαγή παραμέτρου). Έτσι, οι εξι­σώσεις ( 1 ) και (2) παριστάνουν την ίδια ευθεία

και επειδή d( 4 - t) =

- 1 < Ο, την διαγράφουν

dt κατά αντίθετες φορές. Άρα, τα εφαπτόμενα δια­νύσματα σε οποιαδήποτε σημεία τους κατά την φορά διαγραφής τους είναι αντίθετα. Που είναι

ΕΥΚΛΕΙΔΗΣ Β ' 69 τ. l /72

Μαθηματικά για την Γ Λυκείου

το καταπληκτικό; Που είναι το άτοπο; Το ότι οι εξισώσεις ( 1 ) και (2) παριστάνουν

την ίδια γραμμή (ευθεία) δε σημαίνει ότι για το ίδιο t από τις εξισώσεις αυτές παίρνουμε το ίδιο σημείο της γραμμής αυτής. Α ν ένα σημεί­ο, της γραμμής αυτής, προκύπτει από την ( 1 ) για t = !0 , από την (2) το ίδιο σημείο προκύ-πτει για t = 4 - t0 • Μόνο για t = 2 και από τις δύο εξισώσεις παίρνουμε το ίδιο σημείο, το (2,2), το οποίο ο κ. Πετράκης λέει ότι είναι το μοναδικό κοινό τους σημείο . Αλλά και στο σημείο αυτό τα εφαπτόμενα διανύσματα είναι αντίθετα, άτοπο όπως μας λέει. Άρα, ο ίδιος απέδειξε ότι «το μοναδικό κοινό σημείο (2,2)

των δύο γραμμών C 1 και C Γι δεν είναι

κοινό σημείο αυτών»! ! ! • Τρ ίτο λ{ιθος. Μπερδεύει τα αναλλοίωτα ση­

μεία ενός γραμμικού μετασχηματισμού με τα κοινά σημεία μιας γραμμής και της εικόνας της μέσω του γραμμικού αυτού μετασχηματισμού.

• Τέταρτο λάθος. Ισχυρίζεται ότι οι περιορισμοί δύο αντιστρόφων συναρτήσεων οφείλουν να είναι επίσης aντίστροφες συναρτήσεις.

Αυτό είναι λάθος. Τέτοιο θεώρημα στα μα­θηματικά δεν υπάρχει.

• Π έμπτο λάθος. Ισχυρίζεται ότι δύο συναρτή­σεις με το ίδιο σύνολο αφίξεως και το ίδιο γράφημα δεν είναι ίσες. ! ! !

• ' Ε κτο λάΟος. Ισχυρίζεται ότι οι γραφικές παρα­στάσεις δύο aντίστροφων συναρτήσεων μπορεί να διέρχονται από το ίδιο σημείο του επιπέδου, χωρίς το σημείο αυτό να είναι κοινό τους ση­μείο ! ! ! ( εισήγησή του στη 2η μαθηματική ε­βδομάδα Θεσσαλονίκης , σελίδα 206 ) .

Κυκλοφορούν γ ι α τιs τρε ι s τό ξε ι s του γυμνασίου από τι s εκδόσε ιs Μ Α Θ Η Μ Ατ Ι Κ Η Β Ι Β Λ Ι ΟΘ Η Κ Η

Συγγροφεis: ΕΛΕΝΗ MHHIOY ΛΕΟΝΙΔΑΣ θΑΡΡΑΛΙΔΗΣ ΑΝΑΠΑΣΙΑ τΣΙΡΟΠΑΝΝ Η θΕΟΔΟΡΟΣ ΛΑΖΑΡΙΔΗΣ ΣΟΤΗΡΗΣ τΟΥΒΛΑ ΙΖΗΣ

www. νafeiadisbooks.gr

Διαλtττη 27 · 546 2 1 Θεσσαλονίκη Τηλ. -Faχ: 231 0 263 1 63

Αλλάζουμε το τοπίο στο εκπαιδευτικό βιβλίο

ΜΑΘΗ ΜΑΥΙ ΚΑ Για τη Θετική - Τεχνολογική

κατεύθυνση Γ' ΛΥΚΕΙΟΥ

Ιωάννη<; Γ αρατζιώτηι; Παύλοι; Μάοτακαι;

ετοιμάζεται ο μ· τόμο(

ΜΑΘΗΜΑΥΙΚΑ Για τη Θετική - Τεχνολογική

Κατεύθυνση Β' ΛΥΚΕΙΟΥ

Παναγιώτη<; Δουληγέρη<;

�� r:κ�fι:Έ��

www. kedrosed u .com ξε ασεις, οργανωμένα με τη uλλω ε

ΑΛΓΕΒΡΑ Α' ΛΥΚΕΙΟΥ

Ιωάννη<; Γαρατζιώτηι; Παύλοι; Μάοτακαι;

ΕΚΔΟΣΕΙΣ Κ ΕΔΡΟΣ: Γ. ΓΕΝ ΝΑΔΙΟΥ 3, ΑΘ ΗΝΑ 1 06 78 • ΤΗΛ. : 2 1 0 38.09.71 2 • e-mai l : [email protected]

ΕΥΚΛΕΙΔΗΣ Β' 69 τ. l/73

Μαθηματικά για την Γ Λυκείου

5. Ε Π Ι Λ Ο ΓΟ l: Αναρωτιέται κανένας τι είναι σωστό από αυτά που ισχυρίζεται ο κ. Πετράκης. Από αυτά που γράφει στο βιβλιαράκι που κυκλοφόρησε και αλλού, έχουμε σχηματίσει την εντύπωση ότι ο κ. Πετράκης στον τύπο ι- Ι (y) = χ (σελίδα 23 στο παραπάνω βιβλιαράκι) εξακολουθεί να θεωρεί για την συνάρτηση ι- Ι ανεξάρτητη μεταβλητή το χ και εξαρτημένη μεταβλητή το y , που είναι απαράδεκτο λάθος, γιατί όπως είπα­με και παραπάνω, στην αντίστροφη συνάρτηση οι ρόλοι των μεταβλητών χ και y έχουν εναλλαχθεί (μετά τα αλλάζουμε). Ίσως αυτό να είναι η αιτία όλων των τραγελαφικών που μας λέει

6 . ΣΧΟΛ Ι Ο Δεν είναι αλήθεια, όπως ισχυρίζεται ο κ. Πετράκης, ότι η ακαδημαϊκή κοινότητα έχει αυτή την άποψη . Το πανεπιστήμιο Drexel που βρίσκεται στην Αμερική παρέχει μια υπηρε­σία μέσω internet. Έχει φτιάξει ένα forum το οποίο ονομάζεται Ask Dr. Math. Όπως το λέει και το όνομά του σ' αυτό το forum μπορεί ο καθένας να υποβάλει οποιαδήποτε μαθηματική απορία . Υπάρχει μια ομάδα εθελοντών μαθηματικών οι οποίοι απαντούν στις απορίες αυτές. Ο κ. Νίκος Φωτιάδης διαπίστωσε ότι στο παρελθόν είχε γίνει ερώτηση στο forum αν οι f και /-' τέμνο­νται μόνο πάνω στην y = χ είτε είναι γνησίως αύξουσες είτε γνησίως φθίνουσες και κάποιος από τους εθελοντές μαθηματικούς (με το όνομα Gaff) απάντησε καταφατικά. Έστειλε λοιπόν ο κ. Φωτιάδης μια σχετική ερώτηση σ ' αυτό το forum ζητώντας εξηγήσεις και πήρε άμεσα την απάντησή τους. Την απάντηση την έστειλε ο Doctor Peterson ένας από τους κύριους συνεργάτες του forum. Ανακάλεσε πλήρως την λάθος απάντηση που είχε δοθεί στο παρελθόν.

Η μετάφραση τη ς απάντησης (του Doctor Peterson) είναι η εξJ1 ς : «Στα αρχεία μας (εννοεί στο sίte τους) βάζουμε απαντήσεις μας οι οποίες νομίζουμε ότι αξίζουν να τις μοιραστούμε με άλ­λους. Ορισμένες φορές κάποιοι μας γράφουν όταν βρίσκουν κάποιο λάθος μας και τότε το διορ­θώνουμε. Είμαστε απλά άνθρωποι και είμαστε ευγνώμονες όταν μας διορθώνουν. Προφανώς αυτή η απάντηση (εννοεί του/της Gajj) την οποία δεν την έχουμε στο αρχείο μας, βρίσκεται σε ένα sίte που δεν ελέγχουμε και άρα δεν μπορούμε να το διορθώσουμε. Αυτό είναι ατυχία. (Η λανθασμένη απάντηση Gaff είναι δημοσιευμένη σε ελληνικό sίte) . Είναι απλά μια λάθος απάντηση και από τότε έχουμε απαντήσει σωστά όταν ερωτηθήκαμε για κάτι παρόμοιο. Δεν είναι αλήθεια ότι μια συνάρ­τηση τέμνει την aντίστροφή της μόνο πάνω στην y = χ και το παράδειγμά σου της f (χ) = -χ3 το αποδεικνύει. Υπάρχουν βεβαίως πολλά ακόμη παραδείγματα, σε αυτά περιλαμβάνονται συναρτή-

..

. . . . . . . -

ΜΑθΗΜΑΠ<Α ΜΟΝΠΛΑ Π Η Δ Ε Υ i Ε Ρ Ο β λ θ Μ Ι Α Ε Κ Π Α Ι Δ Σ Υ Σ Η ... .....

σεις που είναι aντίστροφες του εαυτού τους όπως οι

f(x) = !Ξ ή f(x) = c - χ , αλλά και πιο ενδιαφέ­χ ρουσες περιπτώσεις όπως η συνάρτηση f(x) = 2 ln(5 - x) . Η σωστή δήλωση είναι ότι η f τέμνει την ι-ι σε κάθε χ για το οποίο ισχύει f (χ) = χ , αλλά μπορεί να τέμνονται και σε άλλα σημεία. Υποθέτω ότι έχεις την απόδειξη της πρότα­σης ότι κάθε γνησίως αύξουσα συνάρτηση τέμνει την aντίστροφή της μόνο πάνω στην y = χ . Δεν νο-μίζω ότι αυτό είναι δύσκολο να αποδειχθεί. Προφα­νώς, δεν είναι αλήθεια για μια γνησίως φθίνουσα συνάρτηση. Ένας μαθηματικός ξέρει να μην εμπι­στεύεται ότι ο οποιοσδήποτε λέει χωρίς απόδειξη. Το αντιπαράδειγμά σου δείχνει ότι αυτός που ισχυ­ρίζεται το αντίθετο δεν μπορεί να έχει απόδειξη. Έτσι μπορείς να αγνοήσεις (ή να διορθώσεις) αυτόν τον επιπόλαιο ισχυρισμό». Τα κείμενα της ερώτη­σης και της απάντησης (στα αγγλικά) βρίσκονται στην παρακάτω διεύθυνση .

http://mathforum.org/dr.cgi/update.pl?status=ansd&option=continue&id=822540nfotiad

ΕΥ ΚΛΕΙΔΗΣ Β ' 69 τ. l/74

Ε u κλεiί δ η ς n ρ οτ ε ii νε �

«Η καρδιά των μαθηματικών είναι τα προβλήματα και οι λύσεις και ο κύριος λόγος ύπαρξης του μαθηματικού είναι να λύνει προβλήματω). Ρ. R. HALMOS

Επιμέλεια: Γ. ΤΡΙΑΝΤΟΣ, Ν. ΑΝΤΩΝΟΠΟΥ ΛΟΣ, ΘΑΝ. ΚΥΡΙΑΚΟΠΟΥ ΛΟΣ

ΑΣ Κ Η Σ Η 1 24 (τΕΥΧΟΥΣ 64)

Σε ισοσκελές τραπέζιο ΑΒΓ Δ οι μη παράλληλες

πλευρές ΒΓ και ΑΔ είναι ίσες με την μικρή βάση Γ Δ και σχηματίζουν με την μεγάλη βάση ΑΒ

γωνίες 20° . Ά ν Ε είναι εσωτερικό σημείο της

ΑΒ τέτοιο, ώστε ΑΕ= Γ Δ και Ρ το κοινό σημείο Λ

των ΒΓ , ΑΔ να αποδείξετε ότι ΑΡΕ = 30° .

(Προτάθηκε από τον Πολιτικό Μηχανικό κ. Ι Ω­Λ :\: \ Η Α :\ Δ Ρ Η - Α θήνα)

ΛΥΣΗ lη (από τον συνάδελφο Γιάννη Σταματο­

γιάννη - λροσιά Αττι κής)

Ρ 2ο�

�Γ �3� Α ι � Ε Θ Β

Σύμφωνα με την υπόθεση, το τετράπλευρο ΑΔΓΕ εί­

ναι ρόμβος με διαγωνίους ΑΓ, ΔΕ που διχοτομούνται

κάθετα και διχοτομούν τις γωνίες του. Από το σημείο

Ρ φέρουμε ευθεία κάθετη στην ΑΓ που τέμνει την ΑΓ

στο Μ και την ΑΒ στο Θ. Το τρίγωνο ΑΡΘ είναι ι­

σοσκελές με κορυφή το Α, αφού το ύψος του ΑΝ εί­

ναι και διχοτόμος της γωνίας Α. Συνεπώς, το Ν είναι

το μέσον της ΡΘ με αποτέλεσμα το τρίγωνο ΓΡΘ να

είναι επίσης ισοσκελές ( αφού η ΓΝ είναι διάμεσος

και ύψος του). Άρα είναι ΓΡ=ΓΘ και

ΓΡΘΛ

�eΛ

Ρ Ι 8οο -60ο 60ο . . Θ � = 1 · • = , οποτε το τριγωνο ΓΡ •

2

είναι ισόπλευρο με αποτέλεσμα ΡΓ=ΡΘ. Επειδή

ΔΡ=ΡΓ είναι και ΔΡ=ΡΘ. Από το ορθογώνιο τρίγωνο

ΑΡΝ έχουμε: ω + φ = 80° ( 1 ) Τ α τρίγωνα Τ ΔΓ και

ΤΕΓ είναι ίσα ( αφού έχουν την ΤΓ κοινή πλευρά,

ΔΓ=ΕΓ και ΔrΤ = ErT = Ι 0° ) .

Λ Λ Λ Άρα Τ ΔΓ = ΤΕΓ => Τ ΔΓ = ω .Επειδή ΔΡ=ΡΘ είναι

Ρ ΔΘ = ΡeΔ => ω + 20° = φ (2). Από τις σχέσεις

( I ),(2) => ω + 20° = 80° - ω => 2ω = 60° => ω = 30° .

Λ ΥΣΗ 2" (Από την συνάδελφο Ι ωάννα Γιαννα­

κοπούλου - Λαμία)

Λ Λ Επειδή ΑΡ// ΕΓ είναι ΑΡΕ = ΡΕΓ = χ .

Α ν ΔΗ .l ΑΒ , ΓΖ .l ΑΒ , τότε ισχύει ότι:

ΑΒ = Γ Δ + 2 · ΖΒ => ΑΒ - Γ Δ = 2 · ΖΒ = 2ασυν20° ( 1 ) .

Επειδή ΔΓ // ΑΒ , από την ομοιότητα των τριγώνων

Ρ ΔΓ και Ρ ΑΒ, παίρνουμε :

ΡΓ ΔΓ ΡΓ - = - => ---ΡΒ ΑΒ ΡΒ - ΡΓ

ΔΓ ΡΓ ΔΓ --- => - = --

ΑΒ - ΔΓ ΒΓ 2 · ΖΒ

< Ι > ΡΓ α α =>- = => ΡΓ = (2)

α 2ασυν20° 2συν20° Ρ

Δ:tJ?r �==ι� Α Η Ε Ζ Β

Από τον νόμο των ημιτόνων στο τρίγωνο

ΡΓ ΕΓ ΕΓ ημ(l40° - χ) ΡΕΓ, έχουμε : - =>-

ημχ ημ(ΕΡΓ) ΡΓ ημχ

(2) α =>>----α

2 · συν20°

ημ(140° χ) . ....::....:.....___-___.:. =>2ημχσυν20° =ημ(140° -χ)

ημχ

=> 2ημχσυν20° = ημ(40° + χ)

=> 2ημχσυν20° = ημ40° συνχ + ημχσυν40°

=> ημχ(2συν20° - συν40° ) = ημ40° συνχ

συνχ 2συν20° - συν40° => -- = ----...,..---

ημχ ημ40°

ΕΥΚΛΕΙΔΗΣ Β' 69 τ. Ι/75

---------------0 Ευκλείδης προτείνει . . . ---------------1 ο J3 ο ο 2 . - · συν40 + 2- ημ40 -συν40 2 2 = J3 => χ = 300

ημ40°

Λύσεις έστειλαν επίσης οι συνάδελφοι Α ντιuνης Ι ω ανν ίδης - Λ{φ ισα, Ροδ<Jλψος '\1πόρης - Λfιφ-

� �Σ Κ. Ηi Σ Η � 2 5 (τΕΥΧΟΥΣ 64) Με πλευρές τις πλευρές τριγώνου ΑΒΓ κατα­σκευάζουμε τετράγωνα εξωτερικά του ΑΒΓ και

ονομάζουμε Α1 , Β 1 , Γ1 τα κέντρα τους. Επανα­

λαμβάνουμε την ίδια διαδικασία στο τρίγωνο

Α1 Β1 Γ1 την οποία συνεχίζουμε επ άπειρον. Να

εξετάσετε αν υπάρχει το όριο της ακολουθίας

των τριγώνων ΑνΒνΓν και αν είναι ισόπλευρο

τρίγωνο. (Προτάθηκε από τον συνάδελφο Π Ω Ρ­ΓΟ Τ:t: .�\ � 1'\ Κ ! Λ Η - ΑΓΡΙΝΙΟ)

Στην εκφώνηση του προβλήματος μπαίνει η ορο-

Άρα

α + γ , α + β β ι > -- και τελος γ ι > -- . 2 2

Σχήμα 1

Όμοια

Έτσι λοιπόν έχουμε αι + β ι + γ ι > α + β + γ δηλαδή

δημιουργείται μια ακολουθία περιμέτρων (Πv) των Δ Δ Δ

τριγώνων ΑΒΓ, Αι Βr Γι , . . . , " Βν Γν , · · · που είναι

λογία «όριο ακολουθίας τριγώνων», οπότε για τους γνήσια αυξουσα με ελάχιστο στοιχείο το

αναγνώστες δημιουργείται ένας προβληματισμός Πο = α+ β + γ. Δηλαδή έχουμε:

για το τι ακριβώς εννοείται, αφού δεν έχει προη- O<fiσ <Π1 <� < ... <f\ < . . . ( 1 ) . Προκύπτει εύκολα γουμένως οριστεί η έννοια όριο ακολουθίας τρι- ότι: ΑΒι · ΑΓι = ΒΓι · ΒΑι = ΓΑι · ΓΒι = -Ε0(Ε0 είναι το γώνων. Στην περίπτωση που εννοείται, να εξετα-

Δ

σθεί αν οι ακολουθίες (αν) , (βv) , (γv) των μηκών εμβαδόν του ΑΒΓ) . Πράγματι: Δ

των πλευρών των τριγώνων Αν Β ν Γ v συγκλίνουν

(έχουν όριο στο JR. ) μπορούμε να δώσουμε την λύση που ακολουθεί.

Θεωρούμε Β1 Γι = αι . Τότε προκύπτει ότι:

(r1K >� και ΒιΑ ��) ή (Γι Κ � � και Β ι Α > %} {Κ} = ΑΒ n Γ1 Β 1 και {Λ} = ΑΓ n Γ1 Βι

γ β Αυτό επειδή αν ΓιΚ =- και ΒιΛ=- , θα είχαμε 2 2

- - β · γ ( ) β · γ ΑΒι · ΑΓι =-συν 90° + Α = --ημΑ = -Ε0 2 2

β2 2 2 - 2 (- - )2 + γ Επομένως: αι =ΓιΒι = ΑΒι -ΑΓι =-2- + 2Ε0

2 2 Κυκλικά παίρνουμε: β� = α + γ + 2Ε0 και τέλος

2

β2 2 γ� =

+α + 2Ε0 Γενικεύοντας εύκολα ( επαγωγι-2

κή κατασκευή) έχουμε την σχέση β2 2 ν- Ι ; γ ν-Ι + 2Εν-ι (2) (Εν είναι το εμβαδό του

Δ

ΑΒ/1 ΑΓ που σημαίνει ότι δεν θα υπήρχε τρίγωνο. αντίστοιχου τριγώνου Αν Β ν Γ ν ) . Παρατηρούμε ό-

ΕΥΚΛΕΙΔΗΣ Β' 69 τ. l/76

-------------- 0 Ευκλείδης προτείνει . . . -------------­

Ιβ2 -α2 1 =! Ιβ2 -α2 I =J.. Iβ2 -α2 I = =J.. Iβ2 -α2 1 ν ν 2 ν-1 ν-1 22 ν-2 ν-2 "' 2ν

Όμοια παίρνουμε: και

λαδή R = Ο που είναι άτοπο αφού R > Ο . Άρα οι

ακολουθίες (αν), (βν), (γv) δεν συγκλίνουν.

Λύσεις έστειλαν επίσης οι συνάδελφοι Γι {ιννης Στιφ ιι.­τογ ι{η·νης - Π είι κη, Ροδόλφος ' l πiψ η ς - λ ι'ιφνη κ ιι.ι

φ υ σ ι κ {1 ο ίδιος ο Γ ιώργος Τσαπακίδ η ς- Α γ ρ ίν ιο ) .

ΛΣ Κ Η Σ Η 1 2 6 (τΕΥΧΟΥΣ 64)

Ιγ: -α: Ι = ;ν Ιγ2 -α2 Ι . Οι προηγούμενες σχέσεις μας Έστω Ρ( χ) ένα πολυώνυμο με ακέραιους συντε-

οδηγούν στο συμπέρασμα: Αν τουλάχιστον μια

από τις ακολουθίες (αν), (βv) , (γν) συγκλίνει σε ένα

πραγματικό αριθμό, έστω R τότε και οι άλλες θα

έχουν το ίδιο όριο καθότι έχουμε ακολουθίες θετι-

' ' 1 ο ' ' κων ορων και 2v � , που σημαινει οτι η ακο-

λουθία (Πν) έχει όριο και μάλιστα Π ν � 3R . Αλλά

Π0 = α + β + γ > Ο => R > Ο .

Αν

ί ··,,

γ/ υν ' '-.�ν ·� I .,"� I

Β ν αν Μ ν αν Γ ν 2 2

Σχήμα 2

Έστω ότι: βν 2. γ ν (Σχήμα 2). Τότε β: -γ: = 2χναν (χν το μέτρο της προβολής της διαμέσου

μ ν = Αν Μ ν στην πλευρά αν = ΒνΓ ν ) από το 2°

Θεώρημα διαμέσου. Αφού αν '* Ο θα έχουμε

χ = βν - γ ν � Ο και υ2 =β2 - Υ + � �R2 -!:__ = 2 2 ( )2 2

ν 2αν ν ν '"ν 2 4

3f2 fJ3 = - => υ � --. 4 ν 2

Επομεν' ως Ε = ανυν � f fJ3 = f2J3 . Από ν 2 4 4 τη

f2 + f2 f 2J3 σχέση (2) προκύπτει ότι f2 = + 2 -- δη-

2 4

λεστές. Α ν για τους ακέραιους κ, λ με κ ::f:. λ ι­

σχύει Ρ(κ)Ρ(λ) =-(κ-λ)2 , να αποδείξετε ότι

Ρ( κ) + Ρ(λ) = Ο . (Προτάθηκε από τον συνάδελφο

� Ι ΚΟ ΑΝΤΩΝΟΠΟΥΛΟ - Ίλιον) ΛΥΣΗ ι " : (Απ6 τον ίδιο)

Θεωρούμε το τριώνυμο : f(x) = (x - Ρ(κ))(χ -

Ρ(λ))

κ - λ λ-κ

= χ 2 _ Ρ ( κ) - Ρ(λ) χ + _Ρ(.::......κ.:.._)Ρ-'-(λ....:...) κ - λ -(κ - λ) 2

Ρ(κ) - Ρ(λ) , = χ 2 - χ + 1 = χ - - μ χ + Ι κ - λ

·ο Ρ(κ) - Ρ(λ) z δ , λ , που = μ , μ ε , ιοτι το π ο υωνυμο κ - λ

έχει ακέραιους συντελεστές και για κάθε θετικό

ν λ'' ακέραιο ν ο αριθμός

κ -είναι ακέραιος. Το

κ - λ

τριώνυμο έχει ρίζες τους ρητούς αριθμούς

Ρ(κ) Ρ(λ) , , 'ζ -- και -- οι οποιες πρεπει να ταυτι ονται κ - λ λ - κ

μ ± Jμz - 4 με τους αριθμούς , οπότε ο αριθμός

2

J μ 2 - 4 πρέπει να είναι ακέραιος. Σ την περίπτωση

αυτή : Ι. Αν μ άρτιος, τότε οι αριθμοί

? Γ2----; μ- - 4 , νι μ - 4 είναι άρτιοι, οπότε οι αριθμοί

μ ± � ' ' ειναι ακεραιοι. 2

11 . Αν μ περιττός, τότε οι αριθμοί μ2 - 4 ,Jμ2 - 4

μ ± Jμ2 - 4 είναι περιττοί, οπότε οι αριθμοί είναι

2

ΕΥΚΛΕΙΔΗΣ Β ' 69 τ. Ι/77

--------------0 Ευκλείδης προτείνει . . . --------------ακέραιοι. Επομένως οι ρίζες του τριωνύμου είναι αριθμοί ακέραιοι και επειδή το γινόμενό τους είναι

I θα ισχύει Ρ(κ) = Ρ(λ) = ι ή Ρ( κ) = Ρ(λ) = - I ' κ - λ λ - κ κ - λ λ - κ

οπότε σε κάθε περίπτωση έχουμε Ρ( κ) + Ρ(λ) = Ο

Λ ΥΣΗ 2η : (από τον συνάδελφο Γιώργο Τρ ιάντο

- Αθήνα) Οι ταυτότητες των διαιρέσεων Ρ( χ ) : (χ - κ) , Ρ( χ ) : (χ - λ) είναι αντιστοίχως:

Ρ(χ ) = (χ - κ)Π(χ) + Ρ(κ) ( ι ) και

Ρ( χ ) = (χ - λ)π(χ ) + Ρ(λ) (2) . Από τις ( 1 ) ,(2) για

χ = λ , χ = κ παίρνουμε αντιστοίχως:

Ρ(λ) - Ρ( κ) = (λ - κ)Π(λ) (3) και

Ρ(λ) - Ρ(κ) = (λ - κ)π(κ) (4)

Από τις σχέσεις (3 ) ,(4) συνάγεται ότι: Π(λ) = π(κ) = p ε Ζ (5) και Ρ(λ) -Ρ(κ) = (λ -κ)p (6)

Με βάση την σχέση (6) και την υπόθεση έχουμε: [Ρ(λ) + Ρ(κ)]2 = [Ρ(λ) - Ρ(κ)]2 + 4Ρ(λ)Ρ(κ) ? 2 2 ? ? (7) = (λ - κγ p - 4(λ - κ) = (λ - κ)- (p- - 4)

Αρκεί να δειχθεί ότι: p2 - 4 = Ο . Η ισότητα

Ρ( κ )Ρ(λ) = -(κ - λ )2 , λόγω της ( 6) γράφεται

Ρ(κ) [Ρ(κ) + (λ - κ)p] = -(λ - κ)2 <=> (Ρ(κ))2 + p(λ - κ)Ρ(κ) + (λ - κ)2 = 0 (8)

Επειδή Ρ( κ) ε Ζ , η (8) δέχεται ακέραιη ρίζα οπό-

τε η διακρίνουσά της είναι τέλειο τετράγωνο ακε­ραίου και επειδή είναι Δ = (λ- κ)2 p2 - 4(λ- κ)2 =

= (λ - κ)2 (p2 - 4) , υπάρχει m ε Ζ τέτοιο, ώστε

p2 - 4 = m2 <=> p2 - m2 = 4 <=> (\Ρ \ + \m\)( \p \ - \m\) = 4 ισότητα που οδηγεί στα δυνατά ενδεχόμενα: { \ Ρ \ + \m\ = 4 { \ Ρ \ + \m\ = 2 ,

\P \ - \m\ = ι (9) ' \ P \ - \m\ = 2 (1 0) . Οι σχε-

σεις (9) δίνουν \Ρ \ = � � Ζ και απορρίπτονται, ενώ 2 οι σχέσεις ( 1 0) δίνουν \Ρ \ = 2 , \m\ = Ο που είναι τι-μές δεκτές και ικανοποιούν την p2 - 4 = Ο . Λύσεις έστειλαν επίσης οι συνάδελφοι Γιώ ργος Τσα-

πακίδης - Αγρ ίν ιο, Αντώνης Ι ωανν ίδης - Λ ά ρ ισα,

Ι ωάννα Γ ιαννακοποίιλου - Λαμία, Ροδόλφος Μπόρης

- Δάφνη, Γιώργος Αποστολόπουλος - Μ εσολ{ιγγι.

ΑΣ ΚΗΣΗ 1 27 (τΕΥΧΟΥΣ 65) Αν α,β είναι θετικοί

πραγματικοί αριθμοί να αποδειχθεί ότι:

(8α2 + β2 )2 > 1 6α3β . (προτάθηκε από τον συνά­

δελφο Θανάση Κυριακόπουλο - Αθήνα)

Λ ΥΣΗ ι Η : (από τον τριτοετή φοιτητή των Μαθη­ματικών Δημήτριο Κουκάκη - Κιλκίς)

Παρατηρούμε ότι: 8α2 + β2 = 4α2 + 2α2 + 2α2 + β2

και 4α2 ;t. 2α2 • Από την ανισότητα του Cauchy για

τους θετικούς αριθμούς 4α2 , 2α2 , 2α2 , β2 έχουμε:

4α2 + 2α2 + 2α2 + β2 > 4�4α2 · 2α2 · 2α2 · β2

� 8α2 + β2 > 4�ι 6α6β2 � 8α2 + β2 > 8� > 4� � (8α2 + β2 )2 > 1 6α3β

Λ ΥΣΗ 2" : (Από τον συνάδελφο Γιώργο Λποστο­

λόπουλο - Μεσολόγγι) Η αποδεικτέα ισοδυναμεί με : 64α4 + 16α2β2 +β4 - 16α3β > Ο και με διαίρεση δια

α4 > 0 , 64 + 1 6(�Υ + (�)4 - 1 6� > 0 ( Ι ) . Θέτουμε α α α

� = k > Ο , και η αποδεικτέα ισοδυναμεί με την α k4 + 1 6k2 - 1 6k + 64 > Ο (2) . Έχουμε :

k4 + ι6k2 - 1 6k + 64= k4 + ι 6(k2 - k) +64

4 2 ι ι = k + ι6(k -k +- --) + 64 = 4 4

4 I z ι 4 I z = k + 1 6[(k -2) -4] +64= k + 1 6(k -2) + 60 > 0

Λύσεις έστειλαν επίσης οι συνάδελφοι: l:τέλιος Π ετρο­

λέκας - Δ ρ απι:τσώνα, Δη μήτρ ιος Καραβότας - Αχαία

, ΓιιίJργος Τσαπακίδης - Αγρίνιο, Γιώργος Ν ι κητά­

κης - Σητεία Κρήτης, Ροδ{ιλφος Μπόρης - Δ άφνη ,

Αθανάσιος Καλ{ικος - Π ατήσια, Ι ωάννα Γιαννακο­

ποίιλου - Λ α μ ία, Γι{ιννης Σταματογιάννης - Δροσιά

Αττικής, Γιώ ργος Μ:λη στάθη ς - Κ άτω Π ατήσια, Βα­

σίλης Ν ι κολ{ικης - Θες\ν ίκη, Η λίας Κ ων/νος - Αλι­

βέρι, ο Χη μικός κ. Δη μήτριος Καρβελάς - Πείι κη και

ο πολιτικός Μηχανικός κ. Ι ωάννη ς Ανδρή ς - Αθήνα.

ΕΥΚΛΕΙΔΗΣ Β' 69 τ. l/78

---------------0 Ευκλείδης προτείνει . . . ---------------ΑΣΚΗΣΗ 1 28 (τΕΥΧΟΥΣ 65) Δίνεται η εξίσωση

αχ2 + βχ + γ = Ο (1), όπου α, β, γ ε R με lαr l > Ο

(2) και Ια i - IP I + lr l < Ο (3) Να αποδειχθεί ότι η

εξίσωση (1) έχει το πολύ μία ακέραιη ρίζα. (προ­

τάθηκε από τον συνάδελφο Θανάση Κυρ ιακόπου­

λο - Αθι1να)

ΛΥΣΗ (από τον συνάδελφο Γιώργο Τσυ.πακίδη ­

Αγρίνιο) Έστω ότι η εξίσωση ( 1 ) έχει δύο ακέραι-

ες ρίζες ρ 1 , ρ2 • Επειδή Ιαγ l > Ο , είναι α * Ο , γ * Ο .

'Ετσι, ρ 1 ρ2 = 1 * Ο , οπότε ρ 1 * Ο , ρ2 * Ο . Άρα, θα α

είναι και I ρ ι 1 2: ι , Ιρ 2 1 2: ι ( 4 ) . Έχουμε:

Ι αΙ - Ιβ l + lr l < ο � � - � + b1 < ο Ια l lα l lα l � � - �-�� + �� � < Ο � ι - lρ ι + ρ2 l + iρ ι ρ2 I < Ο (5)

Όμως, ι - l ρ ι l - lρ 2 1 + 1ρ ι l l ρ 2 1 :ς ι - l ρ ι + ρ2 1 + 1 ρ ιρ z i < Ο (λόγω της (5)) , οπότε 1 - lρ ι 1 - Ι ρ 2 1 + l ρ ι l l ρz l < Ο � � σ - lρι Ιχι - Ιρ2 1 ) < ο , άτοπο, λόγω των σχέσεων < 4).

Άρα, η εξίσωση ( ι ) έχει το πολύ μία ακέραιη ρίζα. Λύσεις έστειλαν επίσης οι συνάδελφοι Γιώργος Απο­

στολόπουί.ος - Vl εσολόγγι, Στέλιος Π ετρολέκας -

Δραπετσ<όνα, Γιώργος Ν ικητάκης - Σητεία Κρήτης,

Ροδόλφος Μ πόρης - Δάφνη, Αθανάσιος Καλάκος -

Π ατήσια, Λουκάς Χυτή ρης - Κέρκυρα, Ιωάννα Γιαν­

νακοπούλου - Λαμία, Γιάννης Σταματογιάννης -

Μ 1 Μ 3 , Μ 2 Μ 4 και Κ το κέντρο βάρους του τρι-

' Ν β θ ' λ ' ΟΚ Π ' ' γωνου. α ρε ει ο ογος -- . ροτεινεται απο ΟΔ

τον Χημικό Δη μήτριο Καρβελά - Π είJκη.

1 44. Αν α, β, γ είναι μη αρνητικοί πραγματικοί α-

ριθμοί με α + β + γ = ι , να αποδειχθεί ότι ισχύει:

β β 9αβγ + 1 Π , ,

α + γ + γα :ς . ροτεινεται απο τον συ-4 νάδελφο Γιιόργο Αποστολόπουλο - Μεσολόγγι.

145 . Αν είναι Ι το έγκεντρο, Ο το περίκεντρο, Η το

ορθόκεντρο οξυγωνίου σκαληνού τριγώνου ΑΒΓ

και θ το μέτρο της γωνίας ΙΗΟ, να δειχθεί ότι:

1 - S + 2ι - 8Ρ , , συνθ = , οπου ειναι 2J(l - 8P)(l - S + ι - 2Ρ) S = συν Α + συν Β + συν Γ ,

ι = συνΑσυνΒ + συνΒσυνΓ + συνΓσυνΑ ,

Ρ = συνΑσυνΒσυνΓ . Προτείνεται από τον συνά­

δελφο Γιώργο Τριάντο - Αθήνα.

1 46. Σε τυχαίο τρίγωνο ΑΒΓ χωρίζουμε τις πλευ­

ρές του ΑΒ και ΑΓ με τα σημεία Ε, Ζ και Η, Θ α­

ντιστοίχως σε τρια ίσα μέρη (ΑΖ=ΖΕ=ΕΒ και

ΑΘ=ΘΗ=ΗΓ) . Η ΒΘ τέμνει τις ΓΕ και ΓΖ στα ση­

μεία Κ,Λ αντιστοίχως και η ΒΗ τέμνει τις ΓΕ, ΓΖ

στα σημεία Ν,Μ αντιστοίχως. Να αποδειχθεί ότι:

α) ΚΜ//ΒΓ, ΚΜ = � ΒΓ . β) (ΚΛΜ) =�(Ι<ΛΜΝ) 7 7

Προτείνεται Γ. Αποστολόπουλο - Μεσολόγγι

Δροσιά Αττικής, Γιώργος Δεληστάθης - Κάτω Π α- 1 47. Να δείξετε ότι ο αριθμός τήσια, Βασίλης Νικολάκης - Θες\νίκη, Ηλίας

Κων/νος - Αλιβέρι, ο τριτοετής φοιτητής των Μαθημα­

τικών Δημήτριος Κουκάκης - Κιλκίς, και ο Πολιτικός

Μηχανικός κ. Ιωάννης Ανδρής - Αθήνα.

Π ΡΟΤ Ε Ι ΝΟΜ ΕΝΑ Θ Ε Μ ΑΤΑ

1 43. Δίνεται τρίγωνο ΑΒΓ και τυχαίο σημείο Δ στο

εσωτερικό της πλευράς ΑΒ. Ονομάζουμε

� 6J3 + ι Ο - � 6J3 - 1 Ο είναι ρητός Προτείνεται

από τον συνάδελφο Γ. Η λιόπουλο - Καλαμάτα

1 48. Έστω α ε (Ο , ι ) και συνάρτηση f : IR � IR με

f ( χ ) - f ( αχ) l im f ( χ) = Ο και lim = Ο . Να δειχθεί χ�ο χ �ο χ ότι lim

f ( χ ) = Ο (Προτείνεται από τον συνάδελφο χ�Ο Χ

αντιστοίχως, Ο το σημείο τομής των Γ. Η λιόπουλο - Καλαμάτα

ΕΥΚΛΕΙΔΗΣ Β ' 69 τ. l/79

Τα Μαθηματικά μας διασκεδάζουν Τα μαθηματικά αν και είναι επιστήμη που απαιτεί αυστηρή διατύπωση, έχουν τη μαγεία να αποσπούν το ενδια­

φέρον όλων των ανθρώπων. Επινοήσεις σε προβλήματα ή ασκήσεις με κατάλληλο τρόπο διατυπωμένα εξάπτουν το πνεύμα, διεγείρουν τη φαντασία και κεντρίζουν την περιέργεια. Πρώτοι οι Αρχαίοι Έλληνες όπως ο Διόφα­ντος, ο Ζιίνωνας κ. ά. μας δίδαξαν αυτά τα μαθηματικά. Στη στήλη αυτή θα παρουσιάζουμε θέματα τα οποία δεν απαιτούν ιδιαίτερες μαθηματικές γνώσεις αλλά μας διασκεδάζουν με την εκφώνησή τους ή τη λύση τους και είναι μια ευχάριστη και συναρπαστική ασχολία .

Λίγα λόγια για το Παράδοξο Το παράδοξο είναι κάθε συμπέρασμα που αρχικά

φαίνεται παράλογο αλλά υποστηρίζεται από κάποιο επι­χείρημα. Είναι ιστορίες που προκαλούν απορία και σύγ­χυση αλλά ταυτόχρονα μας συναρπάζουν. Είναι γνωστά τα παράδοξα του Ζήνωνα αλλά και του Ευβουλίδη .

Ας δούμε μερικά α) του σωρού : Έχετε μπροστά σας ένα σωρό από άμμο. Ας υποθέσουμε ότι ο σωρός έχει 1 . 000.000 κόκκους. Α ν αφαιρέσετε έναν κόκκο θα έχετε και πάλι σωρό, αν αφαιρέσετε δύο κόκκους θα έχετε και πάλι σωρό, αν αφαιρέσετε τρείς κόκκους θα έχετε και πάλι σωρό. Ακολουθώντας τον συλλογισμό αυτό θα φτάσετε στο παράλογο συμπέρασμα ένας μόνο κόκκος να αποτελεί σωρό.

β) του Αχιλλέα και της χελώνας : Σε διαγωνισμό στο τρέξιμο στα ι 00 μέτρα ο Αχιλλέας που τρέχει δέκα φορές ποιο γρήγορα από τη χελώνα ξεκινά Ι Ο μέτρα ποιο πίσω από την αφετηρία της χελώνας. Όταν ο Αχιλ­λέας φτάσει στο σημείο εκκίνησης της χελώνας, η χε­λώνα θα έχει προχωρήσει Ι μέτρο, όταν ο Αχιλλέας διανύσει το ένα μέτρο, η χελώνα θα έχει προχωρήσει ι ο εκατοστά, κ.ο.κ. συνεπώς δεν μπορεί ο Αχιλλέας να νι­κήσει τη χελώνα. γ) το παράδοξο του βράχου : Όταν λέμε ότι κάποιος είναι παντοδύναμος σημαίνει ότι μπορεί να κάνει τα πάντα. Ο Θεός ας πούμε είναι πα­ντοδύναμος. Ερώτηση « Μπορεί ο θεός να φτιάξει έναν βράχο που να μη μπορεί να τον σηκώσει;» Α ν δεν μπο­ρεί ο θεός να φτιάξει έναν βράχο που να μη μπορεί να τον σηκώσει τότε υπάρχει κάτι που δεν μπορεί να το κάνει, άρα δεν είναι παντοδύναμος. Αν μπορεί ο θεός να φτιάξει έναν βράχο που να μη μπορεί να τον σηκώσει τότε πάλι δεν είναι παντοδύναμος. δ) η υπέροχη κόμη : Ένας νέος έχει υπέροχη κόμη ( Ι 00.000 υπέροχες τρίχες τόσες νομίζω, ρωτήστε εσείς ένα γιατρό). Μια τρίχα έ­πεσε ε δεν έγινε και φαλακρός, έχει και πάλι υπέροχη κόμη . Δυο τρίχες έπεσαν πάλι έχει υπέροχη κόμη, τρείς τρίχες έπεσαν παραμένει υπέροχη κόμη, ν τρίχες έπεσαν

Επιμέλεια : Παναγιώτης Χριστόπουλος

υπέροχη κόμη, έτσι αν και έμεινε μια μόνο τρίχα δεν εί­ναι φαλακρός! έχει υπέροχη κόμη ! ! ! Θα σας πρότεινα να διαβάσετε το βιβλίο «Το παράδοξο» της Doris Οιίη καθηγήτριας στο Τορόντο του Καναδά.

Πρόβλημα Σε ένα τηλεοπτικό παιχνίδι πρέπει να διαλέξετε ένα φάκελο ανάμεσα σε δυο σφραγισμένους που ο ένας περιέχει 5 .000 Ευρώ και ο άλλος 1 0 .000 Ευ­ρώ. Επιλέξατε στην τύχη τον Α αλλά σας δίνετε την τε­λευταία στιγμή η δυνατότητα να τον αλλάξετε με τον Β . Θ α το κάνετε;

Η Εγκυκλοπαίδεια Ο Μιχάλης αγόρασε μια εγκυ­κλοπαίδεια που έχει 8 τόμους. Τοποθέτησε τους τόμους της εγκυκλοπαίδειας στη βιβλιοθήκη του κανονικά από τα αριστερά προς τα δεξιά ι 0\ 2°\ . . . 8°ς τόμος. Κάθε τόμος έχει πάχος 5 εκατοστά. Τι πάχος έχουμε από την πρώτη σελίδα του πρώτου τόμου μέχρι την τελευταία του τελευταίου;( τα εξώφυλλα δεν έχουν πάχος) .

Οι κηπουροί Ο Α κηπουρός είχε 8 στρέμματα κή­που για σκάψιμο. Ο Β κηπουρός είχε 6 στρέμματα κή­που για σκάψιμο αλλά τους βοήθησε και ένας εργάτης στον οποίο έδωσαν 2 Ι Ο Ευρώ.

Εργάστηκαν και οι τρείς μαζί για το σκάψιμο. Πό­σα χρήματα έδωσε ο Α και πόσα ο Β κηπουρός στον

Τα Ψάρια Κάποιος αγόρασε ψάρια (μπαρ-μπούνια, γόπες ,κοκωβιοί) και έδωσε Ι 00 Ευρώ. Πόσα πήρε από κάθε είδος; (τα μπαρμπούνια προς 5 Ευρώ, οι γόπες προς Ι Ευρώ και οι κοκωβιοί 0,05 Ευρώ).Λύστε το ίδιο με Ι 20 ψάρια και Ι 20 Ευρώ (κόστος 2€,3€, 0.5€)

ΕΥΚΛ ΕΙΔΗΣ Β' 69 τ. Ι /80

Τα Μαθη ματικά μας διασκεδάζουν -------------

4 ψή φιος αριθμός Είναι τέλειο τετράγωνο , τα 2 πρώτα ψηφία είναι

ίσα μεταξύ τους, τα 2 τελευταία επίσης ίσα μεταξύ τους. Ποιος είναι ο αριθμός;

Ένα ευχάριστο παιχνίδι Πόσες μέρες την εβδο­μάδα θα ήθελες να μην εργάζεσαι ;

Πολλαπλασίασε αυτόν τον αριθμό επί 2 . Πρόσθεσε και 5 και τώρα τον αριθμό αυτόν πολλαπλασίασε επί 50. Αν τα γενέθλια σου για φέτο; πέρασαν πρόσθεσε 1 758 αν όzι πρόσθεσε 1 75 7 . :\πό τον αριθμό που σα; έχει προ":ύwει αφαιρέστε το έτο; γέ\..,,σή; σα;.

Τώρα έχετε έ\·α τριψήφιο αριθμό. Το πρώτο ψηφίο είναι πόσε; μέρε; την εβδομάδα δεν θέλετε να εργάζεστε.

Τ α άλλα δυο ψηφία αντιστοιχούν φυσικά . . . στην ηλικία σας ! ! !

Το Λάδι Ο Πέτρος και ο Νίκος θέλουν να μοιρά­σουν το λάδι που έχουν σε δυο δοχεία 1 3 λίτρα και 7 λί­τρα. Στη διάθεσή τους υπάρχει άδειο δοχείο 1 9 λίτρων. Πώς θα γίνει το μοίρασμα;

Οι κόρες του Γιώργου

Καλημέρα Γιώργο Καλημέρα Δημήτρη Τι κάνουν οι κόρες σου; Καλά είναι: Πόσο ϊpόνων είναι: Το γινόμενο των ηλιt-..,ών του; είναι 64. Δεν είναι αρκετή η πληροφορία. Α ναι η μεγάλη είναι ξανθιά και η μικρή είχε σήμε­

ρα γενέθλια. Εντάξει τώρα ξέρω. Η Π ΡΟ Σ Θ Ε Σ Ι Σ Ξέρετε τι λέει ο Κ.Π. Καβάφης;

Απαντήσεις στα μαθηματικά μας διασκεδά�οuν. � ' Ψ' ' : Αν ο Α έχει χ Ευρώ τότε ο Β έχει τα μισά (χ/2) ή διπλάσια (2χ) Ευρώ. Έτσι αν αλλάξετε φάκελο η προσδοκώμενη ωφελιμότη­τα είναι Υ2.χ/2+ Υ2 .2χ = 1 ,25χ ενώ ο Α έχει προσδοκώ­μενη ωφελιμότητα χ. Έτσι φαίνεται ότι πρέπει να αλλά­ξετε φάκελο. Αν όμως αλλάξετε το φάκελο και πάρετε τον Β με τον ίδιο συλλογισμό πρέπει να επιστρέψετε στον Α. (Παράδοξο) Επομένως δεν αλλάζει τίποτα αν κάνουμε αλλαγή φακέλου. Η . , , , ,, , , ; ·τ , , i ,) : ω : Μεταξύ της πρώτης σελίδας του πρώτου τόμου και της τελευταίας του 8ου έχουμε 6 τό­μους δηλαδή 30 εκατοστά. '· ' ' ··. η ·: ' ' ; ·, · ' · Έσκαψαν 1 4 στρέμματα αλλά αν και οι ίδιοι ήταν εργάτες θα πλήρωναν 630 Ευρώ Άρα το I στρέμμα κοστίζει; 630/1 4=45Ευρώ Ο Α θα πληρώσει 630/ 1 4χ8=360-2 1 0= 1 50 Ο Β θα πληρώσει 630/ 1 4χ6=270-2 1 0=60 ' , ·: 1 9 μπαρμπούνια χ5Ευρώ=95, I γόπες χ Ι Ευρώ = 1 , 8 0 κοκωβιοί χ0,05 Ευρώ =4

fΈπιστnuρν�

82, 84, 86, 88 , 90, 92, 94 3, 6, 9, 1 2, 1 5 , 1 8 , 2 1 3 5 , 30, 25, 20, 1 5 , 1 0, 5 ; , . , , , • t •.< " ' ' ' ' > · - 7744

Δοχεία 16 βήματα 1 9 ο 7 1 9 1 2 1 2 5 5 1 8 1 8 1 1 1 1 4 4 1 7 1 7 1 0 ι 3 1 3 ι 3 ι ι 8 8 1 3 ο 2 2 9 9 ι 3 ο 3 3 7 7 0 0 7 0 7 2 2 0 7 0 7 3 3 0 7 ' ) ι ' "' i " "' :ο • , , ; : · · Το 64 είναι 2 .2 .2 .2 .2 .2 και α-φού δεν είναι δίδυμες ή τρίδυμες είναι 2, 4, 8 ετών. 1 1 ϊ : , . . , ,; , ; , '" ..: : Αν ευτυχής ή δυστυχής είμαι δεν εξετάζω. Πλην ένα πράγμα με χαράν στο νου πάντα βάζω που στην μεγάλη πρόσθεση (την πρόσθεση των που μισώ) που έχει τόσους αριθμούς, δεν είμαι εγώ εκεί απ' ταις πολλαίς μονάδες μιά . . Μες στ' ολικό ποσό δεν aριθμήθηκα. Κι αυτή η χαρά μ' αρκεί. (Ευχαριστούμε τον κ Παπα­

σταυρίδη Σταύρο που μας την έστειλε.)

.ι.tι-; πιrm.r- 1� με θέμcχ: 'Συμποσιο

'Ύο Έρyο του Κ. ΚαραfJεοδωρrί

ιαιι οι Σύyχρονεs Προεκτάσειs ταιJ'

Κωνσταντίνος ΚφaθεοδωρΠ με θ!μ<ο �το ΈptQ m.• Κ ΙUφ<ι/kοδωρn ,.... ιvΣV�Jψoonιtoειs ·wtΓ

-ΔΗΜΟΣ

ΠΕΡΙΣΊΈΡ10Υ piti!N!QjψΙmtAjjKΙ!iii§iiW

110 111 g_ � � ΟΚΤΩΒΡΙΟΥ

11873-19501 Ο Κ. Κορσθεοδωρι\ ιmpξι ο κορυφοίο( Έλληνος /"Ιοθηιο1'Jiόι;

mς: ώγ�ρονnς εnoJr'κ;

Το σrιμονnιι:ό εΙϊΙσrψονικό του έργο σνaφέρεαι σε πολλούς τομεlς των t"οβφατικωv και mς �κfιι;.

Υηι'φfε ιιιcιικός Κοθnyn'Ιής των !lονεοοmμίων·

Ανόβεροο, Μη:ιέολοοu, Γιι.:ιίτιyιο:εν, 8εραλίvου. Αθηνών,

Ε. t1 Πολυπ:χνε/οu κιι Μονάχου_

Εκ.λtχθnιο:ε μέλος οολλtiΝ Ακοδψων, μεrοξύ ΠιΙν οrιοΚ.ιν ιnς; Γ1ρωσιιο:ής Ακaδnμbι; τοu βερολiνοu,

Tl"'( &:ιuορικ.ι'ις Αι<:οδnμiας Εnισmμών, fl'l( Ακοδnμος Αθnνών κ.ό

ΕΥΚΛΕΙΔΗΣ Β ' 69 τ.l/81

> 3 ω

(JJ' ·W .-< '3

U· ·IB111"'MMMM Ιt1.30-ΙΙ,30 'E..q:Q>Σ�vdφi.,.. c:ιnQ ιc Δ!μ:� lkpaoιρloι.ι

. ,...._. Χοφtιοιμ>�[� Π ρώτη Ιuνcδρ iασπ

ιι,10-Ι2d10 ΙΙ.. Β•ΙΙrsdι.Π.W.ιαν<ίαΜσ>Φ;αu. βcιJοι;οοι/Ιι\Ιωδnμα (rοιηnμ.>�� ·L� � dι!rιg.rossιιon� l•taltwrrι;NJkκ

ο lkr � � c.r..� ::ι_ :-=.;.:�:'i:""Κ:=::''ow ::ι_ ι2....οο-ιωο Γ.rlοvκλlδnο;.[&/tr.\iμ.,οοριαΓΙaλοιιι:χνι\a Ο <Δ.<Ι'ψ<>�ιi; crnοll'μ(l>'l'ς "'"m-ριώοο <ιJ<'iσt'ροφιιiίνΙΜ α._ IUI).Iz.ι.G �:;..::"�:=ew�<>λ<M,.;r,c -δ �::::.•οιι Κ �μtΟ<Jαοο..,.. Q_ 1 2..411-l :t-.00 Ιιfιχ..ΔΩWψ(.βJιο.<> Ε"""iδα:<>n. I<> ΓενικόΛ""ι:ι� C �=ς�;.,ΚΗtJf101Jrοδωρtivοο ιιw&υιφD/kιθμιο '"""""""

< Ν

...J < cz: z u < ­w l­ι:::ι <

:Ξ W w :Ξ: ι­w < Ξ :Ξ σ α: Q..

από r ιs εκδόσε ιs <<ΔΙΟΦΑΝΤΟΣ>>

Γιώργος Μ. Μιχαηλίδης Γιώργος Μ. Μιχαηλfδης

ΜΑθΗΜΑΤΙΚΑ r ΛΥΚΕΙΟΥ

ΤΟΜΟΣ Β' θετική - Τεχνολογική Κατεύθυνση

ΟΛΟΚΛΗΡΩΜΑΤΑ

8 ΘtωQ(α a Meθιtbf'ιλoyCα • ΙΙαοαtηQήσεις - Σχολια 8 :\υ��Ενcι ΙlαL!αhεί;••ιαtιι • ερωτψrει; ΚαtανόηΟ1j;- • λσκtiυει; - et,ιαtcL

Γιώργος Μ. Μιχαηλfδης

ΜΑθΗΜΑΤΙΚΑ ΓΛΥΚΕΙΟΥ

θετική - Τεχνολογική Κατεύθυνση

sξs�άσsις 100& - 1008

Πιθανι2 etματα

--,-�--�-

�'

Γιώργος Μ. Μιχαηλ!δης

w >< Ω.) w E _,J Q.) CL. ::c; 2: � o a.. (,) ·-

CI) w ._ α: :;: w ·-

c z: .... :::;) : :z w

Octavian Ν. Stanasila (Proi. UιιiY. - -

ΜΑθΗΜΑΤΙΚΑ Γ ΛΥΚΕΙΟΥ

Θετική - Τεχνολογική Κατεύθυνση

Μ Ι ΓΑΔΙ ΚΟΙ ΑΡΙΘΜΟΙ

απt) το Λ ι7' ειος . . . τ" J_

••

Γιωpγος Μ Μιχαηλfδης

cι:: "'

1-cι:: α:: :ε (;) w ....ι .... .... cι:: < :ε !:: ...ι cι:: ­c:, ι:r:ι ϊ= : (/) (;) ϊ= α: cι:: Ω.. 1- cι:: (I) -

ι α: - (;) - w 1- ::ι: � �-::ι .....

Octavian Ν. Stanasila (Prof. UM. P--

ΔωΦΑΝΤΟΣ ΕΚΔΟΣΕΙΣ